download scanned papers here - Paper 1    Paper II


UPSC PRELIMS 2021 PAPER I -- ANSWER KEYS

Exam Analysis HOME       2021 - Paper I - English       Paper II - E       Paper I - Hindi       Paper II - H      


विहंगम दृष्टि

Exam Analysis HOME       2021 - Paper I - English       Paper II - E       Paper I - Hindi       Paper II - H      

  • मूल तत्त्व: UPSC सिविल सेवा प्रारंभिक परीक्षा 2021 का आयोजन देश के 77 शहरों में 10 अक्टूबर, 2021 को किया गया था। इस वर्ष भी लगभग 10 लाख उम्मीदवारों ने परीक्षा के लिए पंजीकरण कराया था। लगभग 5-5.5 लाख उम्मीदवारों के परीक्षा में बैठने का अनुमान था। इस साल सीटों की संख्या कम कर दी गई है। पिछले साल यह 796 थी। इस साल यह 712 है।

  • 2021 एक कठिन पेपर था, हालांकि 2020 में उतना कठिन नहीं था (पिछले सात वर्षों में सबसे कठिन)। प्रयास की एक अच्छी दर होती – पेपर I – 65-70 प्रश्न; पेपर II – 27+ प्रश्न

  • प्रीलिम्स 2021 कठिन था, लेकिन 2020 जितना नहीं जो किसी भी छात्र के लिए बड़ी चुनौती थी - जिन्होंने या तो वर्ष 2019 या पहले के वर्षों के आधार पर एक निर्धारित पैटर्न की उम्मीद की थी

  • अब छात्र जानते हैं कि उन्हें किसी भी पैटर्न के लिए तैयार रहना होगा

  • सबक (एक बार फिर) : लंबी अवधि की तपस्या, गहराई में जाएं, विवेक, एवं अभ्यास। चूंकि यह एक सापेक्ष परीक्षा है, अंततः वे लोग जिन्होंने अपना दिमाग शांत रखा और अपना सर्वश्रेष्ठ प्रदर्शन किया, वे बढ़िया करेंगे

  • विषयवार वर्णन:
    1. सिविल सेवा परीक्षा से अपेक्षित कठिनाई स्तर के समान
    2. 2020 के पेपर से थोड़ा अधिक कठिन; संतुलित प्रश्नपत्र
    3. भारतीय इतिहास, अर्थशास्त्र, राजनीति, पर्यावरण, भारतीय और विश्व भूगोल, विज्ञान और प्रौद्योगिकी, अंतर्राष्ट्रीय संबंध, करंट अफेयर्स आदि जैसे सभी विषयों के प्रश्न
    4. इतिहास से अधिकतम प्रश्न जो प्राचीन, मध्यकालीन और आधुनिक का समामेलन था
    5. अर्थव्यवस्था - कुल 15 प्रश्न। 2 या 3 आसान थे और सरल उन्मूलन विधियों द्वारा हल किए गए थे। कठिनाई का स्तर पिछले साल की तुलना में कम था। स्थैतिक से भी कुछ प्रश्न।
    6. इतिहास - यूपीएससी हर साल और गहराई में जा रहा है। भूगोल से जुड़े कई पहलुओं और इतिहास को जोड़ने वाले प्रश्न पूछे गए हैं। मध्यकालीन इतिहास के प्रश्नों का भार बढ़ा (9 प्रश्न)। कठिनाई स्तर - कठिन
    7. राजव्यवस्था - कुछ सीधी, कुछ तथ्यात्मक, कुछ वैचारिक, कुछ करेंट अफेयर्स से जुड़ी। राजनीति एनसीईआरटी से सीधे प्रश्न (कक्षा 11 और 12। उदाहरण के लिए: संवैधानिक सरकार, 1950 में भारत की संवैधानिक स्थिति, देवी लाल, भारत के संघीय चरित्र को इंगित करने वाली आवश्यक विशेषताएं)
    8. भूगोल – तथ्यात्मक प्रश्न, अनुप्रयुक्त प्रश्न, प्रत्यक्ष कई प्रश्न, पिछले वर्ष की तुलना में आसान
    9. पर्यावरण - पारंपरिक ज्ञान की ओर झुकाव वाले प्रश्न। संघ, सहजीवी संबंधों आदि की बुनियादी समझ। प्रवृत्ति समान है जैसे कि यूएनईपी, यूएनएफसीसीसी आदि जैसे बड़े संगठनों पर अधिक ध्यान देना।
    10. करेंट अफेयर्स – पिछले 6-12 महीने के प्रश्न, कोविद -19 महामारी, भारत-चीन संबंध, खेल से 3 प्रश्न पिछले 20 वर्षों में पहली बार पूछे गए थे! (हमारी वर्गीकरण और ब्रेक-अप तालिका में, इनमें से कई सामान्य विज्ञान, जीव विज्ञान, विज्ञान और प्रौद्योगिकी या पर्यावरण और पारिस्थितिकी के अंतर्गत पाए जा सकते हैं, क्योंकि वे मूल रूप से उन विषयों से आते हैं)
  • कट-ऑफ: अधिकांश कट-ऑफ पिछले वर्ष की तुलना में थोड़ा अधिक होने की उम्मीद कर रहे हैं, भले ही रिक्तियों की संख्या कम हो। कुछ मोटे अनुमान हो सकते हैं - सामान्य 98.46± 5; ईडब्ल्यूएस 84 ± 5; ओबीसी 96 ± 5; एससी 83 ± 5; एसटी 80 ± 5.

विषयवार वर्गीकरण

Exam Analysis HOME       2021 - Paper I - English       Paper II - E       Paper I - Hindi       Paper II - H      


विस्तृत प्रश्नवार समाधान (Set A)

Exam Analysis HOME       2021 - Paper I - English       Paper II - E       Paper I - Hindi       Paper II - H      

1. निम्नलिखित कथनों पर विचार कीजिए।

  1. केन्द्र सरकार द्वारा भारतीय रिजर्व बैंक (आर. बी० आर०) के गवर्नर नियुक्ति की जाती है।
  2. भारतीय संविधान के कतिपय प्रावधान केन सरकार को जनहित में आर० बी० आइ. को निदेश देने का अधिकार देते हैं।
  3. आर०पी० आई० का गवर्नर अपना अधिकार (पावर) आर०बी० आइ. अधिनिया से प्राप करता है?

उपर्युक्त कथनों में से कौन-से सही है?

  1. केवल 1 और 2
  2. केवल 2 और 3
  3. केवल 1 और 3
  4. 1, 2 और 3

उत्तर (c) कथन 1 एवं 3 सही हैं, लेकिन 2 नहीं है। याद रखें कि आरबीआई का संपूर्ण ढांचा 1935 में, आरबीआई अधिनियम, 1934 के अनुसार अस्तित्व में आया था।

आरबीआई-गवर्नर की नियुक्ति भारत सरकार द्वारा ही की जाती है। अतः कथन 1 सही है।

आरबीआई, एक संवैधानिक निकाय नहीं है, क्योंकि इसे भारतीय रिजर्व बैंक अधिनियम, 1934 के तहत स्थापित किया गया था। आरबीआई की स्वतंत्रता संवैधानिक नहीं है। हालांकि स्थापना के समय निजी स्वामित्व वाली आरबीआई, 1949 में राष्ट्रीयकरण के बाद से, पूरी तरह से भारत सरकार के स्वामित्व में है।

सातवीं अनुसूची (अनुच्छेद 246) के तहत भारतीय संविधान संघ एवं राज्य स्तर पर सरकार के संबंधित कार्यों एवं वित्तीय संसाधनों को निर्धारित करता है एवं इसमें तीन सूचियां शामिल हैं। सूची (i) संघ सूची (97 आइटम) में, इसमें आरबीआई (कार्य-रक्षा, परमाणु ऊर्जा एवं खनिज संसाधन, विदेश मामले, राजनयिक संबंध, रेलवे, एयरवेज, डाक एवं टेलीग्राफ, संघ का सार्वजनिक ऋण, मुद्रा एवं सिक्के, आरबीआई, बैंकिंग, बीमा, स्टॉक एक्सचेंज, आदि) का उल्लेख है। लेकिन इसमें यह विवरण नहीं है कि शक्ति का प्रयोग कैसे किया जाना है।

लेकिन आरबीआई अधिनियम, 1934 के अनुसार, अध्याय 2, खंड 7, ‘प्रबंधन’ के अनुसार - (1) केंद्र सरकार समय-समय पर बैंक के गवर्नर से परामर्श करने के बाद बैंक को जनहित में आवश्यक निर्देश दे सकती है। अतः कथन 2 सही नहीं है।

गवर्नर वास्तव में आरबीआई अधिनियम, 1934 से अपनी शक्तियां प्राप्त करता है।

अक्टूबर 2018 में, सरकार ने आरबीआई अधिनियम के तहत पहले कभी इस्तेमाल नहीं की गई शक्तियों को लागू किया, जिससे वह सार्वजनिक हित के मामलों पर केंद्रीय बैंक के गवर्नर को निर्देश जारी कर सके, एक घटना जिसने आरबीआई एवं सरकार के बीच चल रही झड़प को एक नया मोड़ दिया था।आरबीआई अधिनियम की धारा 7 सरकार को कुछ मुद्दों पर कार्रवाई करने के लिए राज्यपाल से परामर्श करने एवं निर्देश देने का अधिकार देती है, जिसे सरकार गंभीर एवं जनहित में मानती है। स्वतंत्र भारत में अब तक इस धारा का प्रयोग कभी नहीं हुआ था। इसका उपयोग1991 एवं 2008 के आर्थिक संकट के समय भी नही किया गया था।कथन 3 सही है।



2.भारत में नियोजित अनियम मजदूरों के सन्दर्भ में, निम्नलिखित कथनो पर विचार कीजिये:

  1. सभी अनियत मजदुर नियत कार्य-समय एवं समयोपरि भुगतान के हक़दार है।
  2. सभी अनियत मजदुर नियमित कार्य-समय एवं समयोपरि भुगतान के हक़दार है।
  3. सरकार अधिसूचना के द्वारा यह विनिर्दिष्ट कर सकती है की कोई प्रतिष्ठान या ओधोग केवल अपने बैंक खातों के माध्यम से मजदूरी का भुगतान करेग।

उपर्युक्त कथनों में से कौन - से सही हैं ?

  1. केवल 1 और 2
  2. केवल 2 और 3
  3. केवल 1 और 3
  4. 1, 2 और 3

उत्तर (d) ईपीएफ में नियमों के अनुसार, जिस कर्मचारी का वेतन नौकरी शुरू करते समय 15,000 रुपये प्रति माह से अधिक है, वह कर्मचारी भविष्य निधि का सदस्य बनने के लिए पात्र नहीं है एवं उसे अपात्र कर्मचारी कहा जाता है। 15,000 रुपये प्रति माह से कम आहरण करने वाले कर्मचारियों को अनिवार्य रूप से ईपीएफ का सदस्य बनना होता है। फिर, 2020 में, सुप्रीम कोर्ट ने माना कि जहां तक सामाजिक सुरक्षा लाभों का संबंध है, कोई नियोक्ता किसी अनियत श्रमिक कर्मचारी एवं स्थायी कर्मचारियों के बीच अंतर नहीं कर सकता।

अतः कथन 1 सही है। इसलिए विकल्प (b) गलत है।

श्रम कानूनों में सुधारों ने अब अनियत श्रमिकों के लिए भी समयोपरि (ओवरटाइम) भुगतान को अनिवार्य कर दिया है। अतः कथन 2 भी सही है। कथन 3 सही है।


3. आर्थिक मंदी के समय, निम्नलिखित में से कौन-सा कदम उठाये जाने की सर्वाधिक सम्भावना होती है?

  1. कर की दरों में कटौती के साथ-साथ ब्याज दर में वृद्धि करना
  2. सार्वजनिक परियोजनाओं पर व्यय में वृद्धि करना
  3. कर की दरों में वृद्धि के साथ-साथ ब्याज दर में वृद्धि करना
  4. सार्वजनिक परियोजनाओं पर व्यय में कमी करना

उत्तर (b) आर्थिक मंदी के दौरान सरकार आर्थिक गतिविधियों को आगे बढ़ाना चाहती है। ऐसा करने का एक अच्छा तरीका यह होता है कि सार्वजनिक परियोजनाओं पर व्यय में वृद्धि कि जाए। विकल्प (a) गलत है क्योंकि ब्याज दरों में वृद्धि ऋण की लेने गति का धीमा कर देंगी। विकल्प (c) गलत है क्योंकि कर की दरों में वृद्धि निवेश के माहौल को खराब कर देंगी। विकल्प (d) गलत है क्योंकि इससे आर्थिक सुधार धीमा हो जाएगा।


4.निम्नलिखित कथनों पर विचार कीजिए।

अन्य बातें अपरिवर्तित रहने पर भी किसी वस्तु के लिए बाजार मांग बढ़ सकती है, यदि

  1. इसकी स्थानापन्न वस्तु की कीमत में वृद्धि हो
  2. इसकी पूरक वस्तु की कीमत में वृद्धि हो
  3. वस्तु घटिया किस्म की है और उपभोक्ताओं की आय में वृद्धि होती है
  4. इसकी कीमत घटती है

उपर्युक्त कथनों में से कौन-से सही है?

  1. केवल 1 और 4
  2. 2, 3 और 4
  3. 1, 3 और 4
  4. 1, 2 और 3

उत्तर (a) एक स्थानापन्न उत्पाद बाजार में किसी दूसरे उत्पाद के समान उद्देश्य को पूरा करता है। एक के अधिक उपलब्ध होने का अर्थ है दूसरे की कम मांग, यानी एक नकारात्मक सहसंबंध। एक उत्पाद की खपत दूसरे उत्पाद की जगह ले लेती है। इसलिए किसी वस्तु स्थानापन्न उत्पाद के महंगा होने की दशा में उस वस्तु की मांग बढ़ेगी।

एक पूरक वस्तु किसी अन्य उत्पाद को अधिक उपयोगी बनाती है एवं इसे उस उत्पाद साथ मेँ उपयोग किया जाता है। उदाहरण के लिए, अनाज एवं दूध, एक उत्पाद महंगा हुआ तो दूसरे पर नकारात्मक प्रभाव पड़ता है।

एक घटिया वस्तु - परिभाषा के अनुसार - वह वस्तु है जिसकी मांग लोगों की आय बढ़ने पर घट जाती है!

स्पष्ट रूप से, 1 एवं 4 से वस्तु की मांग में वृद्धि होगी, लेकिन 2 एवं 3 में ऐसा नहीं होगा।


5.भारत में 'शहरी सहकारी बैंकों' के संदर्भ में, निम्नलिखित कथनों पर विचार कीजिए :

  1. राज्य सरकारों द्वारा स्थापित स्थानीय मंडलों द्वारा उनका पर्यवेक्षण एवं विनियमन किया जाता है।
  2. वे इक्विटी शेयर और अधिमान शेयर जारी कर
  3. उन्हें वर्ष 1966 में एक संशोधन के द्वारा बैंककारी विनियमन अधिनियम, 1949 के कार्य-क्षेत्र में लाया गया था।

उपर्युक्त कथनों मैं से कौन -सा / कौन -से सही /हैं ?

  1. केवल 1
  2. केवल 2 और 3
  3. केवल 1 और 3
  4. 1, 2 और 3

उत्तर (b) भारतीय रिजर्व बैंक की वेबसाइट के अनुसार, पूंजीगत निधियों में वृद्धि के लिए, ‘शहरी सहकारी बैंकों’ को

  1. उनके उप-नियमों के प्रावधानों के अनुसार, उनके संचालन के क्षेत्र में व्यक्तियों को सदस्यों के रूप में नामांकित करके इक्विटी शेयर जारी करके, इक्विटी शेयर पूंजी जुटाने एवं
  2. मौजूदा सदस्यों को अतिरिक्त इक्विटी शेयर जारी करने, की अनुमति है।

शहरी सहकारी बैंकों को अधिमान शेयर जारी करके भी अपनी पूंजी बढ़ाने की अनुमति है। इसलिए कथन 2 सही है, एवं विकल्प (a) तथा (c) गलत हैं।

अतः कथन 3 निश्चित रूप से सही है।

शहरी सहकारी बैंकों के विनियमन के संबंध में, आरबीआई सूचित करता है कि वह ‘बैंकिंग विनियमन अधिनियम, 1949 के प्रावधानों के तहत शहरी सहकारी बैंकों के बैंकिंग कार्यों को विनियमित एवं पर्यवेक्षित करता है। रिजर्व बैंक के भीतर, एक अलग विभाग-शहरी बैंक विभाग (अर्बन बैंक्स डिर्पाटमेंट) को यह कार्य सौंपा गया है। शहरी बैंक विभाग अन्य नियामकों जैसे सहकारी समितियों के रजिस्ट्रार एवं सहकारी समितियों के केंद्रीय रजिस्ट्रार के साथ निकट समन्वय में कार्य करता है।

अतः कथन 1 सही नहीं है।


6.भारतीय सरकारी बॉन्ड प्रतिफल निम्नलिखित में से किससे/किनसे प्रभावित होता है/होते हैं?

  1. यूनाइटेड स्टेट्स फेडरल रिज़र्व की कार्रवाई
  2. भारतीय रिज़र्व बैंक की कार्रवाई
  3. मुद्रास्फीति एवं अल्पावधि ब्याज दर

नीचे दिए गए कूट का प्रयोग कर सही उत्तर चुनिए।

  1. केवल 1 और 2
  2. केवल 2
  3. केवल 3
  4. 1, 2 और 3

उत्तर (d) भारत सरकार के बॉन्ड प्रतिफल को तीनों कार्यवाहियाँ प्रभावित करती है।

बॉन्ड प्रतिफल का अर्थ है कि बॉन्ड में निवेश करने पर निवेशक को मिलने वाले वापसी से है।

उपज की गणना के लिए, वार्षिक कूपन दर को बांड के वर्तमान बाजार मूल्य से विभाजित किया जाता है।

बॉन्ड प्रतिफल में बदलाव (वृद्धि या गिरावट) का कारण अर्थव्यवस्था में अच्छे या बुरे परिणाम हो सकते हैं। यह रुपये के कमजोर होने से उत्पन्न मुद्रास्फीति एवं इस कारण अधिक महंगा तेल आयात होने पर, या खपत-आधारित मुद्रास्फीति जहां लॉकडाउन के दौरान कम उत्पादन के कारण मांग में वृद्धि या आपूर्ति के झटके आए हो, या अपेक्षित उच्च डिफॉल्ट जोखिम एवं शायद कम क्रेडिट रेटिंग के कारण, या रिस्क इक्विटी के लिए लोगों द्वारा रिस्क फ्री बांड बेचने से, हो सकता है। आमतौर पर यह इनमें से किसी एक के साथ-साथ सरकारी बॉन्ड की समग्र आपूर्ति-मांग (ऐसा कुछ जिसे आरबीआई नियंत्रित कर सकता है) का मिश्रण होता है।

यदि, यूएस ट्रेजरी यील्ड में वृद्धि जारी रहती है, मुद्रास्फीति बढ़ती रहती है, एवं अर्थव्यवस्था में सुधार केंद्रीय बैंकों को नीतिगत रुख बदलने के लिए प्रेरित करता है, तो इससे बड़े पैमाने पर विदेशी फंड का बहिर्वाह हो सकता है, जिससे भारत के शेयर बाजार में प्रभाव पड़ सकता है।


7. निम्नलिखित पर विचार कीजिए :

  1. विदेशी मुद्रा संपरिवर्तनीय बॉन्द
  2. कुछ शर्तों के साथ विदेशी संस्थान
  3. वैश्विक निक्षेपागार (डिपॉजिटरी) प्राप्तियाँ
  4. अनिवासी विदेशी जमा

उपर्युक्त में से किसे /कीन्हे विदेशी प्रत्यक्ष निवेश में सम्मिलित किया जा सकता है/किये जा सकते हैं ?

  1. 1, 2 और 2
  2. केवल 3
  3. 2 और 4
  4. 1 और 4

उत्तर (d) विदेशी मुद्रा संपरिवर्तनीय बांड (एफसीसीबी) किसी भारतीय कंपनी में निवेशित बांड हैं। चूंकि ये बांड समयावधि में इक्विटी शेयरों में परिवर्तनीय हैं, इसलिए वे ‘प्रत्यक्ष विदेशी निवेश’ नीतिके अंतर्गत आते हैं। अतः 1 ‘प्रत्यक्ष विदेशी निवेश’ का हिस्सा है। विकल्प (b) एवं (c) खारिज किए जा सकते है।

अब, 3 निश्चित रूप से ‘प्रत्यक्ष विदेशी निवेश’ नहीं है, बल्कि ‘विदेशी पोर्टफोलियो निवेश’ है, इसलिए विकल्प (a) को खारिज किया जा सकता है।

‘अनिवासी वस्तु’ का अर्थ है ‘भारत के बाहर निवासी व्यक्ति’ जैसा कि फेमा के तहत परिभाषित किया गया है। ‘अनिवासी भारतीय’ (एनआरआई) का अर्थ भारत के बाहर निवास करने वाले किसी व्यक्ति से है जो भारत का नागरिक है। अनिवासी विदेशी जमा को प्रत्यक्ष विदेशी निवेश माना जाता है। इसलिए, (d)।


8.निम्नलिखित कथनों पर विचार कीजिए:

किसी मुद्रा के अवमूल्यन का प्रभाव यह है कि अनिवार्य रूप से

  1. विदेशी बाजारों में घरेलू नियाती की प्रतिस्पर्धात्मकता को बढ़ाता है
  2. घरेलू मुद्रा के विदेशी मूल्य को बढ़ाता है
  3. व्यापार संतुलन में सुधार लाता है

उपर्युक्त कथनों में से कौन-सा/कौन-से सही है हैं?:

  1. केवल 1
  2. 1 और 2
  3. केवल 3
  4. 2 और 3

उत्तर (a) अवमूल्यन का अर्थ है मुद्रा के मूल्य में गिरावट। इसके मुख्य प्रभाव हैं

  1. विदेशी ग्राहकों के लिए निर्यात सस्ता होता है,
  2. आयात अधिक महंगा होता है, एवं
  3. अल्पावधि में, अवमूल्यन से मुद्रास्फीति में वद्धि, उच्च विकास एवं निर्यात की मांग में बढ़ोतरी होती है। इसलिए कथन 1 सही है एवं विकल्प (c) तथा (d)  को खारिज किया जा सकता है।

अवमूल्यन से वास्तव में घरेलू मुद्रा का विदेशी मूल्यघटता है, इसलिए 2 सत्य नहीं है।

केवल विकल्प (a) संभव है।


9.भारत में काले धन के सृजन के निम्नलिखित प्रभावों में से कौन-सा भारत सरकार की चिन्ता का प्रमुख कारण है?

  1. स्थावर संपदा के क्रय और विलासितायुक्त आवास में निवेश के लिए संसाधनों का अपयोजन
  2. अनुत्पादक गतिविधियों में निवेश और जवाहरात, गहने, सोना इत्यादि का क्रय
  3. राजनीतिक दलों को बड़े चंदे एवं क्षेत्रवाद का विकास
  4. कर अपवंचन के कारण राजकोष में राजस्व की हानि

उत्तर (d) चारों विकल्प सही लगते हैं, लेकिन (a) तथा (b) सरकार को उतना प्रभावित नहीं करते जितना (d) निश्चित रूप से करता है। सरकार के राजस्व में गिरावट से उसकी आय पर सीधा असर पड़ता है। विकल्प (c) चिंता का विषय नहीं है क्योंकि सभी राजनीतिक दलों को बड़े चंदे प्राप्त होते है,‘क्षेत्रवाद’ शब्द का प्रयोग (c) को भी गलत बनाता है।


10.निम्नलिखित में से कौन-सा अपने प्रभाव में सर्वाधिक मुद्रास्फीतिकारक हो सकता है?

  1. सार्वजनिक ऋण की चुकौती
  2. बजट घाटे के वित्तीयन के लिए जनता से उधार लेना
  3. बजट घाटे के वित्तीयन के लिए बैंकों से उधार लेना
  4. बजट घाटे के वित्तीयन के लिए नई मुद्रा का सृजन करना

उत्तर (d) विकल्प (d), यानी बजट घाटे को पूरा करने के लिए नए पैसे के निर्माण से मुद्रा नोटों की आपूर्ति में वृद्धि होगी, जो मुद्रास्फीति का एक कारण है। जब बाजार में पैसे की आपूर्ति बढ़ जाती है, तो ग्राहक की मांग अपने आप बढ़ जाती है, जिससे कीमतों में वृद्धि होती है।

बजट घाटा तब होता है जब कुल व्यय कुल राजस्व से अधिक हो जाता है। घाटे का वित्तपोषण बजट घाटे को दूर करने के लिए धन उत्पन्न करने का एक उपकरण है। यह तीन तरीकों से किया जा सकता है, पहला नया पैसा बनाकर, दूसरा आंतरिक स्रोतों से उधार लेकर (जैसे आरबीआई, बॉन्ड आदि जारी करके) एवं तीसरा बाहरी स्रोतों से उधार लेकर (उदा. डब्ल्यूबी, आईएमएफ आदि)।



11.निम्नलिखित में से किससे किसी अर्थव्य्वश्था में मुद्रा गुणक में वृद्धि होती है?

  1. बैंकों में आरक्षित नकदी निधि अनुपात में वृद्धि
  2. बैंकों के सांविधिक चलनिधि अनुपात में वृद्धि
  3. लोगों की बैंकिंग आदतों में वृद्धि
  4. देश की जनसंख्या में वृद्धि

उत्तर (c) ‘मुद्रा गुणक’ से तात्पर्य है कि कुल धन आपूर्ति में कैसे एक प्रारंभिक जमा राशि, अंत में एक बड़ी राशि बन सकती है।

उदाहरण के लिए, यदि वाणिज्यिक बैंक 1 करोड़ रुपये की जमा राशि प्राप्त करते हैं एवं इससे 100 करोड़ रुपये की अंतिम धन आपूर्ति होती है, तो मुद्रा गुणक 10 है।

यदि अधिक लोग बैंकिंग शुरू करेंगे तो क्रेडिट बढ़ेगा एवं मुद्रा गुणक भी।

जहां तक आरक्षित नकदी निधि अनुपात में वृद्धि का संबंध है, यह जितना कम होता है, उतनी ही अधिक यह धन आपूर्ति में वृद्धि लाता है, एवं ग्राहकों की अधिक जमा राशि, बैंकों द्वारा ऋण के रूप में आगे दी जाती है। तो उच्च आरक्षित नकदी निधि अनुपात का अर्थहै कम मुद्रा गुणक। सांविधिक चलनिधि अनुपात के लिए भी यही स्थिति है।


12.भारतीय अर्थव्यवस्था के सन्दर्भ मे, माँग-प्रेरित मुद्रास्फीति या उसमें वृद्धि निम्नलिखित किन कारणों से होती है?

  1. विस्तारकारी नीतियाँ
  2. राजकोषीय प्रोत्साहन
  3. मद्रास्फीति सूचकांकन मजदूरी (इनफ्लेशन इंडेक्सिंग वेजेस)
  4. उच्च क्रय शक्ति
  5. बढ़ती ब्याज दर

  6. नीचे दिए गए कूट का प्रयोग कर सही उत्तर चुनिए।:

  1. केवल 1, 2 और 4
  2. केवल 3, 4 और 5
  3. केवल 1, 2, 3 और 5
  4. 1, 2, 3, 4 और 5

उत्तर (a) राजकोषीय प्रोत्साहन लोगों के हाथों में पैसा डालता है। इससे मांग-प्रेरित मुद्रास्फीति को बढ़ावा होना निश्चित है। अतः विकल्प (b) गलत है। इसी प्रकार 4 निश्चित रूप से सही है, इसलिए विकल्प (c) गलत है।

5 पर ध्यान देने पर - बढ़ती ब्याज दरें- इससे अधिक मांग (अधिक ऋण के माध्यम से) उत्पन्न नहीं होगी, अपितु मांग कम होगी।

इसलिए, विकल्प (d) को खारित किया जा सकता है। इसलिए (a) सही है!

शार्टकट- यदि आप यह पहचान पाते हैं कि 5 गलत है, तो तीन विकल्प (b), (c), (d) तुरंत खारिज हो जाते हैं।


13.भारत के संदर्भ में, निम्नलिखित कथनो पर विचार कीजिये :

  1. खुदरा निवेशक डीमैट खातों के माध्यम से प्राथमिक बाजार में राजकोष बिल (ट्रेजरी बिल) और 'भारत सरकार के ऋण बॉन्ड में निवेश कर सकते हैं।
  2. बातचीत से तय लेनदेन प्रणाली-ऑर्डर मिलान (निगोशिएटेड डीलिंग सिस्टम-ऑर्डर मैचिंग)' भारतीय रिजर्व बैंक का सरकारी प्रतिभूति व्यापारिक मंच है।
  3. सेंट्रल डिपोजिटरी सर्विसेज लिमिटेड' का भारतीय रिजर्व बैंक एवं बम्बई स्टॉक एक्सचेंज द्वारा संयुक्त रूप से प्रवर्तन किया जाता है।

उपर्युक्त कथनों में से कौन-सा/कौन-से सही है/हैं?

  1. केवल 1
  2. 1 और 2
  3. केवल 3
  4. 2 और 3

उत्तर (b) 1 एवं 2 दोनों सही हैं। खुदरा निवेशकों को राजकोष बिल (टी या ट्रेज़री -बिल)एवं सरकारी प्रतिभूतियों के प्राथमिक बाजार में निवेश करने की अनुमति है।

‘बातचीत से तय लेनदेन प्रणाली (नेगोशिएटेड डीलिंग सिस्टमया एनडीएस) भारतीय रिजर्व बैंक द्वारा संचालित एक इलेक्ट्रॉनिक ट्रेडिंग प्लेटफॉर्म है जो सरकारी प्रतिभूतियों एवं अन्य मुद्रा बाजार उपकरणों को जारी एवं विनिमय करने की अनुमति देता है। इसे स्थापित करने का लक्ष्य मैनुअल कागजी कार्रवाई में कटौती एवं सभी प्रतिभागियों के लिए पारदर्शिता बढ़ाना है।

सेंट्रल डिपॉजीटरी सर्विसेस लिमीटेड (सीडीएसएल) को शुरू में एशिया का नवीनतम स्टॉक एक्सचेंज- बम्बई स्टॉक एक्सचेंज लिमिटेड (बीएसई लिमिटेड) द्वारा बढ़ावा दिया गया था। अन्य शेयरधारकों में एचडीएफसी बैंक लिमिटेड, स्टैंडर्ड चार्टर्ड बैंक एवं केनरा बैंक शामिल हैं। इसमें आरबीआई शामिल नहीं था। अतः, 3 गलत है एवं (c) तथा (d) को खारिज किया जा सकता है।


14.वॉटरक्रेडिर' के संदर्भ में निम्नलिखित कथनों पर विचार कीजिए:

  1. यह जल एवं स्वच्छता क्षेत्र में कार्य के लिए सूक्ष्म वित्त साधनों (माइक्रोफाइनेंस टूल्स) को लागू करता है।
  2. यह एक वैश्विक पहल है जिसे विश्व स्वास्थ्य संगठन और विश्व बैंक के तत्वावधान में प्रारंभ किया गया है।
  3. इसका उद्देश्य निर्धन व्यक्तियों को सहायिकी के बिना अपनी जल-संबंधी आवश्यकताओं को पूरा करने के लिए समर्थ बनाना है।

उपर्युक्त कथनों में से कौन-से सही है?

  1. केवल 1 और 2
  2. केवल 2 और 3
  3. केवल 1 और 3
  4. 1, 2 और 3

उत्तर (c) water.org,एक वैश्विक गैर-लाभकारी संगठन है जो विश्व में पानी को सुरक्षित, सुलभ एवं किफायती बनाने एवं स्वच्छता के क्षेत्र में काम कर रहा है। यह विश्व स्वास्थ्य संगठन या विश्व बैंक से संबद्ध नहीं है। (इसलिए तीन विकल्पों (a), (b) एवं (d) को सीधे ही खारिज किया जा सकता है।)

सुरक्षित पानी एवं स्वच्छता की राह में एक प्रमुख बाधा किफायती वित्तपोषण है। वाटरक्रेडिट इनिशिएटिवऋण कार्यक्रम इस बाधा से सीधे निपटता है। वाटरक्रेडिट, घरेलू पानी एवं शौचालय संबंधित समस्याओं के समाधान प्राप्त करने में लगे लोगों एवं संस्थाओं को किफायती वित्तपोषण एवं विशेषज्ञ संसाधनों तक पहुंच बनाने में मदद करता है।

वाटरक्रेडिट, जल तथा स्वच्छता क्षेत्र में, माइक्रोफाइनेंस उपकरणों को लाने वाला पहलाउ संगठन है। वाटरक्रेडिट उन लोगों के लिए छोटे ऋण लाने में मदद करता है जिन्हें घरेलू पानी एवं शौचालय समाधान को वास्तविकता बनाने के लिए किफायती वित्तपोषण एवं विशेषज्ञ संसाधनों तक पहुंच की आवश्यकता होती है।

शॉर्टकट - यदि आप यह पहचान सकते हैं कि 2 गलत है, तो तीन विकल्प (a), (b) एवं (d) तुरंत खारिज हो जाते हैं।


15.भारत में, 'अतिम उधारदात (लैंडर ऑफ़ लास्ट रिसार्ट) के रूप में केंद्रीय बैंक के कार्य में सामान्यतः निम्नलिखित में से क्या सम्मिलित है /हैं ?

  1. अन्य स्त्रोतों से प्राप्ति में विफल होने पर व्यापार एवं उद्योग निकायों को ऋण ऋण प्रदान करना
  2. अस्थायी संकट के समय बैंकों के लिए चलनिधि उपलब्ध कराना
  3. बजटीय घाटों के वित्तीयन के लिए सरकारों को ऋण देना

नीचे दिए गए कूट का प्रयोग कर सही उत्तर चनिया

  1. 1 और 2
  2. केवल 2
  3. 2 और 3
  4. केवल 3

उत्तर (b) भारतीय रिजर्व बैंक स्पष्ट रूप से ‘बैंकों के बैंकर’ के रूप में, ‘अंतिम उधारदाता’ के रूप में भी कार्य करता है।

यह किसी सॉल्वेंट बैंक जो अस्थायी तरलता संकटों का सामना कर रहा की मदद के लिए तब आगे आ सकता है, जब उसे कोई और बैंक ऋण देने को तैयार नहीं होता है।

रिजर्व बैंक, बैंक-जमाकर्ताओं के हितों की रक्षा करने एवं बैंक की संभावित विफलता, जो बदले में अन्य बैंकों एवं संस्थानों को भी प्रभावित कर वित्तीय अस्थिरता ला सकती है, को रोकने के लिए इस सुविधा का विस्तार करता है।

भारिबै केंद्र सरकार के बजटीय घाटे के वित्तीयन के लिए अंतिम उपाय के ऋणदाता के रूप में कार्य नहीं करता है।


16.निम्नलिखित में से किसके अंगीकरण को प्रोत्साहित करने के लिए 'R2 व्यवहार संहिता (R2 कोड ऑफ प्रैक्टिसेज) साधन उपलब्ध करती है?

  1. इलेक्ट्रॉनिकी पुनर्चक्रण उद्योग में पर्यावरणीय दृष्टि से विश्वसनीय व्यवहार
  2. रामसर कन्वेंशन के अंतर्गत अंतर्राष्ट्रीय महत्त्व की आर्द्रभूमि का पारिस्थितिक प्रबंधन
  3. निम्नीकृत भूमि पर कृषि फसलों की खेती का संधारणीय व्यवहार
  4. प्राकृतिक संसाधनों के दोहन में 'पर्यावरणीय प्रभाव आकलन

उत्तर (a) आर 2 का अर्थ रिस्पाँसिबल रीसाइक्लिंग है।ये सस्टेनेबल इलेक्ट्रॉनिक्स रीसाइक्लिंग इंटरनेशनल (एसईआरआई) द्वारा विशेष रूप से इलेक्ट्रॉनिक्स रीसाइक्लिंग उद्योग के लिए बनाये गए मानक है। इन मानकों को फिर अमेरिकन नेशनल स्टैंडर्ड इंस्टीट्युट नेशनल एक्रिडिशन बोर्ड (एएनएबी) द्वारा मान्यता दी गई एवं 2008 में जारी किया गया।


17.ताम्र प्रगलन संयंत्रों के बारे में चिन्ता का कारण क्या है?

  1. वे पर्यावरण में कार्बन मोनोक्साइड को घातक मात्राओं में निर्मुक्त कर सकते हैं।
  2. ताम्रमल (कॉपर स्लैग) पर्यावरण में कुछ भारी धातुओं के निक्षालन (लीचिंग) का कारण बन सकता है।
  3. वे सल्फर डाइऑक्साइड को एक प्रदूषक के रूप में निर्मुक्त कर सकते हैं।

नीचे दिए गए कूट का प्रयोग कर सही उत्तर चुनिए।

  1. केवल 1 और 2
  2. केवल 2 और 3
  3. केवल 1 और 3
  4. 1, 2 और 3

उत्तर (b) कथन 1, एक प्रमुख चिंता का विषय नहीं है, इसलिए तीन विकल्पों (a), (c) एवं (d) को खारिज किया जा सकता है।

ताम्र प्रगलन की मूल प्रतिक्रिया Cu2O + CO → 2Cu + CO2 तांबा एवं वाष्पशील कार्बन डाइऑक्साइड पैदा करती है।

प्रगलन का अर्थ धातु वाले अणुओं को अलग करना है। इसे मौलिक धातु एवं कुछ अन्य तत्वों में तोड़ा जाता है। अणु को अपचयन की प्रक्रिया द्वारा ‘अपचयित’ किया जाता हैं (ज्यादातर कार्बन मोनोऑक्साइड (सीओ) जैसे कुछ अपचयन एजेंट के साथ प्रतिक्रिया द्वारा)।

खनन से प्राप्त अयस्कों से आवश्यक खनिज प्राप्त करने के लिए उन्हें संसाधित किया जाता है। धातु अयस्कों के मामले में, इन खनिज सांद्रों को आमतौर पर अयस्क खनिजों में अन्य तत्वों से धातु को अलग करने के लिए आगे संसाधित करने की आवश्यकता होती है। प्रगलन, धातु को पिघलाने के लिए उच्च तापमान पर धातु को अशुद्धियों से अलग करने की प्रक्रिया है (एक ठोस अपशिष्ट उत्पाद के साथ जिसे ताँबें के प्रकरण में ‘ताम्रमल’ कहा जाता है)।

प्रगलन की प्रक्रिया से प्राप्त होने वाले मुख्य प्रदुषक दूषित वायु, अपशिष्ट जल एवं धातुमल जैसे प्रक्रिया अपशिष्ट हैं। यह वातावरण में तांबा, चांदी, लोहा, कोबाल्ट एवं सेलेनियम जैसी जहरीली धातुओं को छोड़ता है। प्रगलक, गैसीय सल्फर डाइऑक्साइड भी छोड़ते हैं, जो अम्लीय वर्षा में योगदान एवं पानी को अम्लीकृत करते है।

सल्फाइड अयस्कों के गलाने से सल्फर डाइऑक्साइड गैस निकलती है, जो वातावरण में रासायनिक रूप से प्रतिक्रिया करके सल्फ्यूरिक एसिड धुंध बनाती है। जैसे ही यह अम्लीय वर्षा पृथ्वी पर गिरती है, यह मिट्टी, नदियों एवं झीलों की अम्लता को बढ़ाती है, जिससे वनस्पतियों एवं मछलियों एवं वन्यजीव आबादी के स्वास्थ्य को नुकसान पहुंचता है।

केवल एल्यूमीनियम उद्योग में होने वाली प्रगलन की प्रक्रिया से लगभग 17 प्रतिशत ग्रीनहाउस गैसों का उत्सर्जन होता है एवं यह कुल वैश्विक कार्बन डाई ऑक्सईड उत्सर्जन का 1 प्रतिशत है।

शॉर्टकट- यदि आप पहचान सकते हैं कि 1 गलत है, तो तीन विकल्प (a), (c) एवं (d) तुरंत खारिज हो जाते हैं।


18.भट्टी तेल (फर्नेस ऑयल) के संदर्भ में, निम्नलिखित कथनों पर विचार कीजिए:

  1. यह तेल परिष्करणियों (रिफाइनरी) का एक उत्पाद हैं।
  2. कुछ उद्योग इसका उपयोग ऊर्जा (पावर) उत्पादन के लिए करते हैं।
  3. इसके उपयोग से पर्यावरण में गंधक का उत्सर्जन होता है।

उपर्युक्त कथनों में से कौन-से सही हैं?

  1. केवल 1 और 2
  2. केवल 2 और 3
  3. केवल 1 और 3
  4. 1, 2 और 3

उत्तर (d) भट्टी तेल या फर्नेस ऑयल पेट्रोलियम के आसवन से प्राप्त कच्चे तेल का उप-उत्पाद है। अतिव्यापी गुणों के कारण इसे डीजल के साथ भ्रमित किया जा सकता है, लेकिन वे अलग होते हैं।

इसमें मुख्य रूप से कच्चे तेल के आसवन के अवशेष होते हैं। इसका उपयोग मुख्य रूप से बिजली संयंत्रों, जहाजों एवं औद्योगिक संयंत्रों में भाप बॉयलरों के लिए किया जाता है। वाणिज्यिक ईंधन तेलों को आम तौर पर वांछित चिपचिपाहट एवं फ्लैश प्वाइंट बनाने के लिए अन्य पेट्रोलियम अंशों के साथ मिश्रित किया जाता है।

अवशिष्ट ईंधन तेल (भट्ठी का तेल) कम उपयोगी होता है क्योंकि यह इतना चिपचिपा होता है कि उपयोग करने से पहले इसे एक विशेष हीटिंग सिस्टम से गर्म करना पड़ता है एवं इसमें अपेक्षाकृत अधिक मात्रा में प्रदूषक, विशेष रूप से गंधक (सल्फर), जो दहन पर सल्फर डाइऑक्साइड बनाते हैं, हो सकते हैं।


19.ब्लू कार्बन क्या हैं?

  1. महासागरों और तटीय पारिस्थितिक तंत्रों द्वारा परग्रहीत कार्बन
  2. वन जैव मात्रा (बायोमास) और कृषि मृदा में प्रच्छादित कार्बन
  3. पेट्रोलियम और प्राकृतिक गैस में अंतर्विष्ट कार्बन
  4. वायुमंडल में विद्यमान कार्बन

उत्तर (a) ‘ब्लू कार्बन’, विश्व के महासागरों एवं तटीय पारिस्थितिक तंत्र में प्रगृहित कार्बन है। मानवीय गतिविधियाँ कार्बन डाइऑक्साइड का उत्सर्जन करती हैं, जिसमें वायुमंडलीय कार्बन होता है। गैसें दुनिया की जलवायु को नकारात्मक रूप से बदल रही हैं। लेकिन समुद्र एवं समुद्र-तट इस कार्बन के प्रगृहित करने के माध्यम से हमारे वातावरण पर ग्रीनहाउस गैसों के प्रभाव को कम करने का एक प्राकृतिक तरीका प्रदान करते हैं।

समुद्री तटों पर पाई जाने वाली समुद्री-घास, मैंग्रोव एवं नमकीन-दलदल कार्बन प्रगृहक के रूप में कार्य करते हुए कार्बन को ‘कैप्चर एंड होल्ड’ करते हैं। ये तटीय प्रणालियाँ, हालांकि ग्रह के जंगलों की तुलना में आकार में बहुत छोटी हैं, किंतु कार्बन को बहुत तेज दर से अलग करती हैं, एवं लाखों वर्षों तक ऐसा करना जारी रख सकती हैं। इन पारिस्थितिक तंत्रों द्वारा ग्रहण किया गया अधिकांश कार्बन जमीन के नीचे जमा हो जाता है। तटीय मिट्टी में पाया जाने वाला कार्बन अक्सर हजारों साल पुराना होता है!


20.प्रकृति में, निम्नलिखित में से किस जीव का/किन जीवों के मृदाविहीन सतह पर जीवित पाए जाने की सर्वाधिक संभावना है?

  1. फर्न
  2. लाइकेन
  3. मॉस
  4. छत्रक (मशरूम)

नीचे दिए गए कूट का प्रयोग कर सही उत्तर चुनिए।

  1. केवल 1 और 4
  2. केवल 2
  3. 2 और 3
  4. 1, 3 और 4

उत्तर (c) लाइकेन, एक जटिल जीवन रूप है जो दो अलग-अलग जीवों, कवक तथा शैवाल की सहजीवी साझेदारी है। इस प्रक्रिया में प्रमुख भागीदार कवक है।

लाइकेन किसी भी अबाधित सतह पर उगते हैं - छाल, लकड़ी, काई, चट्टान, मिट्टी, पीट, कांच, धातु, प्लास्टिक एवं यहां तक कि कपड़े। लाइकेन अपनी ‘थालि’ के किसी भी हिस्से से पानी को अवशोषित कर सकते हैं एवं उन्हें जड़ों की आवश्यकता नहीं होती है। लाइकेन पौधों को नुकसान नहीं पहुंचाते हैं या छाल की नमी को नहीं लूटते हैं।

अतः 2 सही है, एवं विकल्प (a) तथा (d) खारिज किए जा सकते हैं।

मॉस, तने एवं पत्तों वाले नोन-फ्लॉवरिंग पौधे होते हैं जो स्पोर पैदा करते हैं, लेकिन इनमें वास्तविक जड़ें नहीं होती हैं। मॉस, एवं उन्ही के तरह के पौधों लिवरवॉर्ट्स एवं हॉर्नवॉर्ट्स को प्लांट किंगडम में ब्रायोफाइटा (ब्रायोफाइट्स) के रूप में वर्गीकृत किया गया है। मॉस, नम मिट्टी, पेड़ की छाल, चट्टानों, कंक्रीट, या लगभग किसी भी अन्य स्थिर सतह पर उग सकते हैं।

फर्न को अप्रत्यक्ष धूप, नम मिट्टी एवं आर्द्र वातावरण की आवश्यकता होती है। इसलिए सही उत्तर (c) है।



21.निम्नलिखित में से किसका उपयोग प्राकृतिक मच्छर प्रतिकर्षी तैयार करने में किया जाता है?

  1. कांग्रेस घास
  2. एलिफैंट घास
  3. लेमन घास
  4. नट घास

उत्तर (c) लेमनग्रास का उपयोग प्राकृतिक मच्छर प्रतिकर्षी तैयार करने में किया जाता है।

कांग्रेस घास- इसका वैज्ञानिक नाम, पार्थेनियम हिस्टेरोफोरस है। यह घास 40 से अधिक उष्णकटिबंधीय एवं उपोष्ण कटिबंधीय देशों में पाई जाती है तथा इसे विश्व की 100 सबसे खराब आक्रामक प्रजातियों की सूची में रखा गया है। भारत में अब यह लगभग सभी राज्यों में फैल चुकी है, एक अनुमान के अनुसार, इसके आक्रमण से देश में कई फसलों में 40% तक की उपज का नुकसान हुआ है एवं चारा उत्पादन में 90% की गिरावट आई है।

एलिफैंट घास-इसका वैज्ञानिक नाम, पेनिसेटम पुरपुरम शुमाच है। यह एक प्रमुख उष्णकटिबंधीय घास है। यह सबसे अधिक उपज देने वाली उष्णकटिबंधीय घासों में से एक है। यह एक बहुत ही बहुगुणीय प्रजाति है जिसे कई प्रकार की परिस्थितियों एवं प्रणालियों जैसे शुष्क या नम, छोटे या बडे़ पर कृषि पैमाने के तहत उगाया जा सकता है।

लेमनग्रास पौधेके पत्तों का उपयोग तेल एवं दवा बनाने के लिए किया जाता है। इसका उपयोग पाचन तंत्र की ऐंठन, पेट दर्द, उच्च रक्तचाप, ऐंठन, दर्द, उल्टी, खांसी, जोड़ों में दर्द (गठिया), बुखार, सामान्य सर्दी एवं थकावट के इलाज के लिए किया जाता है। इसका उपयोग कीटाणुओं को मारने एवं हल्के कसैले के रूप में भी किया जाता है। लेमनग्रास का पौधा एक उत्कृष्ट प्रतिकर्षी भी माना जाता है।

नट घास या नागरमोथा (साइपरस रोटंडस) का उपयोग कई बीमारियों के उपचार एवं रोकथाम में किया जाता है। पौधे के प्रकंद का उपयोग दस्त, आंतों के परजीवी, अपच एवं आंत्र विकारों के इलाज के लिए किया जाता है।


22.जीवों के निम्नलिखित प्रकारों पर विचार कीजिए :

  1. कॉपिपोड
  2. साइनोबैक्टीरिया
  3. डायटम
  4. फोरैमिनिफेरा

उपर्युक्त में से कौन-से जीव महासागरों की आहार श्रृंखलाओं में प्राथमिक उत्पादक हैं?

  1. 1 और 2
  2. 2 और 3
  3. 3 और 4
  4. 1 और 4

उत्तर (b) महासागरों में, प्राथमिक उत्पादक - बैक्टीरिया, फाइटोप्लांकटन एवं शैवाल - सबसे छोटा पोषण स्तर बनाते हैं, जो जलीय खाद्यजाल का आधार है। वे भोजन की आवश्यकता के बिना अपनी ऊर्जा का संश्लेषण करते हैं। कई, कार्बोहाइड्रेट बनाने के लिए सूर्य की ऊर्जा के उपयोग से प्रकाश संश्लेषण करते हैं।

कॉपिपोड विश्व महासागर में प्रमुख द्वितीयक उत्पादक हैं।

जैसा कि उपर कहा गया है, जलीय वातावरण में, साइनोबैक्टीरिया महत्वपूर्ण प्राथमिक उत्पादक हैं।

डायटम समुद्र में प्रमुख प्राथमिक उत्पादकों में से एक हैं, एवं प्रकाश संश्लेषण करने वाले शैवाल हैं।

अतः 2 एवं 3 (सायनोबैक्टीरिया एवं डायटम) सही हैं। इसलिए, (a), (c) एवं (d)गलत है।

शॉर्टकट - एक बार जब आप 2 या 3 या दोनों में से किसी एक को जान लेते हैं, तो चीजें आसान हो जाती हैं।


23.निम्नलिखित प्राणियों पर विचार कीजिए:

  1. जाहक (हेज्हॉग)
  2. शैलमूषक (मारमॉट)
  3. वज्रशल्क (पैंगोलिन)

उपर्युक्त में से कौन-सा/कौन-से जीव परभक्षियों द्वारा पकड़े जाने की संभावना को कम करने के लिए, स्वयं को लपेटकर अपने सुभेद्य अंगों की रक्षा करता है/ करते हैं?

  1. 1 और 2
  2. केवल 2
  3. केवल 3
  4. 1 और 3

उत्तर (d) हेज्हॉग एक स्तन पायी है, जिसके शरीर पर काँटे पाए जाते हैं। हमला होने या खतरा होने की दशा में, अपने नाजुक अंतरंग सुभेद्य अंगों की रक्षा के लिए यह अपने सिर एवं पैरों को अपने शरीर में घुमाता है एवं एक गेंद का आकार ले लेता है। अतः 1 सही है एवं विकल्प (b) एवं (c) सही उत्तर नही हो सकते।

शैल मूषक अर्थात मारमॉट (जीनस मर्मोटा) मुख्य रूप से उत्तरी अमेरिका एवं यूरेशिया में पाए जाने वाली विशाल ग्राउंड स्क्विीरल की 14 प्रजातियों में से एक है। ये कृंतक बड़े एवं भारी होते हैं। शिकारी को देखने पर, पीले-बेल वाले मारमॉट क्षेत्र में दूसरे मारमॉटों को चेतावनी देने के लिए सीटी बजाते हैं, जिसके बाद यह आमतौर पर पास के चट्टान के ढेर में छिप जाते है एवं छिपे रहते हैं जब तक कि खतरा टल नही जाता।

खतरा होने पर वज्रशल्क (पैंगोलिन) तुरंत अपने शरीर को लपेटकर एक तंग गेंद में बदल लेता हैं एवं अपने सुभेद्य अंगों की रक्षा के लिए अपनी तेज-नुकीली पूंछ का उपयोग करता है। अतः विकल्प (d) सही है।


24.'वनों पर न्यूयॉर्क घोषणा (न्यूयॉर्क डिक्लेरेशन ऑन फॉरेस्ट्स)' के संदर्भ में, निम्नलिखित में से कौन-से कथन सही हैं?

  1. 2014 में, संयुक्त राष्ट्र जलवायु शिखर सम्मेलन में पहली बार इसका समर्थन किया गया था।
  2. इसमें वन के ह्रास को रोकने के लिए एक वैश्विक समय-रेखा का समर्थन किया गया।
  3. यह वैध रूप से बाध्यकारी अंतर्राष्ट्रीय घोषणा है।
  4. यह सरकारों, बड़ी कंपनियों और देशीय समुदायों द्वारा समर्थित है।
  5. भारत, इसके प्रारंभ के समय, हस्ताक्षरकर्ताओं में से एक था।

नीचे दिए गए कूट का प्रयोग कर सही उत्तर चुनिए।

  1. 1, 2 और 4
  2. 1, 3 और 5
  3. 3 और 4
  4. 2 और 5

उत्तर (a) ‘वनों पर न्यूयॉर्क घोषणा’ एक स्वैच्छिक घोषणा है, अनिवार्य नहीं। अतः विकल्प (b) एवं (c) को अस्वीकार किया जा सकता हैं।

‘वनों पर न्यूयॉर्क घोषणा’ को 2014 के जलवायु शिखर सम्मेलन में 150 से अधिक सरकारों, कंपनियों, स्वदेशी लोगों एवं नागरिक समाज संगठनों द्वारा समर्थन दिया गया था, जो घोषणा के दस लक्ष्यों को प्राप्त करने एवं इसके साथ की कार्रवाई एजेंडा का पालन करने के लिए प्रतिबद्ध थे।

इसके दस लक्ष्यों में 2030 तक प्राकृतिक वन हानि को रोकना, 350 मिलियन हेक्टेयर की गिरावट वाले परिदृश्य एवं वनभूमि को बहाल करना, शासन में सुधार, वन वित्त में वृद्धि, एवं 2020 के बाद के वैश्विक जलवायु समझौते के हिस्से के रूप में वनों की कटाई एवं वन क्षरण से उत्सर्जन को कम करना शामिल है।

भारत इस घोषणा का हस्ताक्षरकर्ता राष्ट्र नहीं है। अतः 5 गलत है, इसलिए विकल्प (d) भी सही नही हो सकता।

शॉर्टकट - सभी 5 तथ्य जानना जरूरी नहीं है। यदि आप किन्हीं दो को जानते हैं, तो भी इस प्रश्न को हल कर सकते थे।


25.तंत्रिका अपहास (न्यूरोडीजेनेरेटिव) समस्याओं के लिए उत्तरदायी माने जाने वाले मैग्नेटाइट कण पर्यावरणीय प्रदूषकों के रूप में निम्नलिखित में से किनसे उत्पन्न होते है?

  1. मोटरगाड़ी के ब्रेक
  2. मोटरगाड़ी के इंजन
  3. घरों में प्रयोग होने वाले माइक्रोवेव स्टोव
  4. बिजली संयंत्र
  5. टेलीफोन लाइन

नीचे दिए गए कूट का प्रयोग कर सही उत्तर चुनिए।

  1. केवल 1, 2, 3 और 5
  2. केवल 1, 2 और 4
  3. केवल 3, 4 और 5
  4. 1, 2, 3, 4 और 5

उत्तर (b) मैग्नेटाइट एक खनिज एवं लौह के मुख्य अयस्कों में से एक है, जिसका रासायनिक सूत्र Fe3O4 है।

मैग्नेटाइट नैनोस्फीयर, दहन एवं/या घर्षण-व्युत्पन्न उष्मा द्वारा बनते हैं एवं शहरी वायुजनित पार्टिक्यूलेट मैटर में (पीएम) में विपुल मात्रा में पाए जाते हैं। चूंकि कई वायवीय मैग्नेटाइट प्रदूषण कण 200 एनएम व्यास से छोटे होते हैं, वे सीधे घ्राण तंत्रिका के माध्यम से एवं क्षतिग्रस्त घ्राण इकाई को पार करके मस्तिष्क में प्रवेश कर सकते हैं। नैनोस्केल मैग्नेटाइट बाहरी चुंबकीय क्षेत्रों के प्रति अनुक्रियाशील एवं मस्तिष्क के लिए विषाक्त होते है।

वाहन ब्रेक सिस्टम, सड़क किनारे वायवीय मैग्नेटाइट का प्रमुख स्रोत हैं। डीजल एवं पेट्रोल-इंजन निकास पीएम में मैग्नेटाइट सांद्रता समान है। वाहन ब्रेक सिस्टम से निकलने वाली उच्च मैग्नेटाइट सामग्री मानव स्वास्थ्य के लिए जोखिम का कारण बन सकती है।

अतः 1 एवं 2 सही हैं, एवं विकल्प (c) गलत है।

घरेलू माइक्रोवेव स्टोव से उत्पन्न मैग्नेटाइट की कोई जानकारी प्राप्त नहीं है। अतः 3 गलत है एवं विकल्पों; (a), (c) एवं (d) को खारिज किया जा सकता हैं।


26.निम्नलिखित में से कौन-सा जीव निस्यंदक भोजी (फिल्टर फीडर) है?

  1. अशल्क मीन (कैटफिश)
  2. अष्टभुज (ऑक्टोपस)
  3. सीप (ऑयस्टर)
  4. हवासिल (पेलिकन)

उत्तर (c) सीप (ऑयस्टर) निस्पंदक भोजी (फिल्टर फीडर) हैं। वे अपने शरीर के माध्यम से बड़ी मात्रा में पानी पंप करके खाते हैं। सिलिया द्वारा सीप के गलफड़ों के माध्यम से पानी पंप किया जाता है। गलफड़ों के बलगम में प्लैंकटन, शैवाल एवं अन्य कण फंस जाते हैं। वहां से इन कणों को खाने के लिए सीप के मुंह एवं अन्नप्रणाली में ले जाया जाता है एवं फिर पचने के लिए पेट में।

अशल्क मीन (कैटफिश) एवं अष्टभुज (ऑक्टोपस) को सीधे तौर पर खारिज किया जा सकता है, क्योंकि दोनों सामान्य भोजी हैं। अधिकांश कैटफिश वास्तव में बॉटम फीडर हैं।

पेलिकन मुख्य रूप से मछली खाते हैं, लेकिन वे अवसरवादी फीडर हैं एवं क्रस्टेशियंस, टैडपोल एवं कछुए सहित विभिन्न प्रकार के जलीय जानवरों को खाते हैं। जब भोजन पकड़ा जाता है, तो पेलिकन इसे अपने बिल में तब तक घुमाता रहता है जब तक कि शिकार का सिर आमतौर पर पेलिकन के गले की ओर नहीं आ जाता।


27.निम्नलिखित जैव भूरासायनिक चक्रों में से किसमें, चट्टानों का अपक्षय चक्र में प्रवेश करने वाले पोषक तत्त्व के निर्मुक्त होने का मुख्य स्रोत है?

  1. कार्बन चक्र
  2. नाइट्रोजन चक्र
  3. फॉस्फोरस चक्र
  4. सल्फर चक्र

उत्तर (c) या (d) जैव-भू-रासायनिक चक्र एक प्राकृतिक मार्ग है जिसके द्वारा जीवित पदार्थ के आवश्यक तत्व परिचालित होते हैं।

कार्बन-चक्र उस प्रक्रिया का वर्णन करता है जिसमें कार्बन परमाणु लगातार वायुमंडल से पृथ्वी तक एवं फिर वापस वायुमंडल में जाते हैं। जीवों के मरने, ज्वालामुखीयों के फटने, आग लगने, जीवाश्म ईंधन के जलने, एवं कई अन्य तंत्रों के माध्यम से कार्बन पुनः वायुमंडल में छोड़ दिया जाता है। अतः (a) गलत है।

नाइट्रोजन-चक्र एक जैव-भू-रासायनिक प्रक्रिया है जिसके माध्यम से नाइट्रोजन को कई रूपों में परिवर्तित किया जाता है, जो लगातार वातावरण से मिट्टी, जीव एवं पुनः वायुमंडल में जाता है। नाइट्रोजन चक्र में पाँच चरण होते हैं- स्थिरीकरण, नाइट्रीकरण, स्वांगिकरण, अमोनीकरण एवं विनाइट्रीकरण। अतः विकल्प (b) गलत है।

फॉस्फोरस-चक्र वह प्रक्रिया है जिसके द्वारा फॉस्फोरस स्थलमंडल, जलमंडल एवं जीवमंडल से होकर गुजरता है। फास्फोरस पौधे एवं जानवरों के विकास के साथ-साथ मिट्टी में रहने वाले रोगाणुओं के स्वास्थ्य के लिए आवश्यक है, लेकिन समय के साथ मिट्टी से धीरे-धीरे समाप्त हो जाते है। पृथ्वी पर अधिकांश फास्फोरस चट्टान एवं तलछटी जमा में है, जहाँ से यह अपक्षय, लीचिंग एवं खनन द्वारा छोड़ा जाता है।

गंधक (सल्फर)-चक्र भूमंडल एवं जीवमंडल के माध्यम से सल्फर की गति का वर्णन करता है। अपक्षय के माध्यम से चट्टानों से सल्फर निकलता है, एवं फिर रोगाणुओं एवं पौधों द्वारा आत्मसात किया जाता है। फिर इसे खाद्य श्रृंखला में पारित किया जाता है एवं पौधों एवं जानवरों द्वारा आत्मसात किया जाता है, एवं जब वे विघटित हो जाते हैं तो उसे छोड़ दिया जाता है।


28.निम्नलिखित में से कौन-से जीव अपरदाहारी (डेट्राइटिनोर) हैं?

  1. केंचुआ
  2. जेलीफिश
  3. सहस्रपादी (मिलीपीड)
  4. समुद्री घोड़ा (सीहॉर्स)
  5. काष्ठ यूका (वुडलाइस)

नीचे दिए गए कूट का प्रयोग कर सही उत्तर चुनिए।

  1. केवल 1, 2 और 4
  2. केवल 2, 3, 4 और
  3. केवल 1, 3 और 5
  4. 1, 2, 3, 4 और 5

उत्तर (c) अपरदाहारी (डिट्रिटिवोर) एक ऐसा जीव है जो मृत या सड़ने वाले पौधों या जानवरों से भोजन प्राप्त करता है। यह एक परपोषी जीव है, जो अपरद द्वारा अपना पोषण करता है।

अक्सर अकशेरुकी कीड़े जैसे कि घुन, भृंग, तितलियाँ एवं मक्खियाँ, घोंघे, मिट्टी में रहने वाले केंचुए, मिलीपेड एवं वुडलाइस, अपरदाहारी होते है।

मछली, जेलीफिश एवं क्रस्टेशियंस सामान्य द्वितीयक उपभोक्ता हैं। जेलीफिश फाइटोप्लांकटन एवं जोप्लांकटन को खाती है।

शॉर्टकट - तीन विकल्पों (a), (b) एवं (d) में कथन 2 हैं जो सही नही है, अतः इन विकल्पों को अस्वीकार किया जा सकता है।


29.यू० एन० ई० पी० द्वारा समर्थित 'कॉमन कार्बन मेट्रिक' को किसलिए विकसित किया गया है?

  1. संपूर्ण विश्व में निर्माण कार्यों के कार्बन पदचिह्न का आकलन करने के लिए
  2. कार्बन उत्सर्जन व्यापार में विश्वभर की वाणिज्यिक कृषि संस्थाओं के प्रवेश हेतु अधिकार देने के लिए
  3. सरकारों को अपने देशों द्वारा किए गए समग्र कार्बन पदचिह्न के आकलन हेतु अधिकार देने के लिए
  4. किसी इकाई समय (यूनिट टाइम) में विश्व में जीवाश्मी ईंधनों के उपयोग से उत्पन्न होने वाले समग्र कार्बन पदचिह्न के आकलन के लिए

उत्तर (a) कॉमन कार्बन मीट्रिक विशिष्ट जलवायु क्षेत्रों के भवनों के प्रकार के संचालन से जुड़े जीएचजी उत्सर्जन के लिए माप, रिपोर्टिंग एवं सत्यापन को परिभाषित करने के लिए उपयोग की जाने वाली गणना है। इसमें वेटिंग या बेंचमार्किंग जैसे मापों की मूल्य-आधारित व्याख्या शामिल नहीं है।

यूएनईपी के अनुसार, कॉमन कार्बन मीट्रिक का उद्देश्य एक ऐसे निर्माण क्षेत्र जो विश्व की 40% ऊर्जा खपत का प्रतिनिधित्व करता है एवं जिसकी वैश्विक ग्रीनहाउस गैस (जीएचजी) उत्सर्जन में हिस्सेदारी एक तिहाई है, को सुसंगत एवं तुलनीय तरीके से कटौती को मापने, रिपोर्ट करने एवं सत्यापित करने के लिए एक ऐसा उपकरण प्रदान करने की है जो अब तक उपलब्ध नही है।


30.निम्नलिखित समूहों में से किनमें ऐसी जातियाँ होती हैं, जो अन्य जीवों के साथ सहजीवी संबंध बना सकती हैं?

  1. नाइडेरिया
  2. कवक (फंजाई)
  3. आदिजंतु (प्रोटोजोआ)

नीचे दिए गए कूट का प्रयोग कर सही उत्तर चुनिए।

  1. केवल 1 और 2
  2. केवल 2 और 3
  3. केवल 1 और 3
  4. 1, 2 और 3

उत्तर (d) सहजीवन दो (या अधिक) विभिन्न प्रजातियों के सदस्यों के मध्य घनिष्ठ पारिस्थितिक संबंध है।

नाइडेरिया एवं डाईनोफ्लैगलेट शैवाल के बीच संबंध को ‘सहजीवी’ कहा जाता है। (नाइडेरिया किंगडम एनिमिलीया के तहत एक फाइलम है जिसमें मीठे पानी एवं समुद्री वातावरण दोनों में पाए जाने वाले जलीय जानवरों की 11,000 से अधिक प्रजातियां शामिल हैं)

कवक से जुड़े दो सामान्य पारस्परिक संबंध माइकोराइजा एवं लाइकेन हैं।

आदिजंतु (प्रोटोजोआ) में सहजीवन ज्यादातर एक प्रोटोजोआ एवं एककोशिकीय सहजीवन (बैक्टीरिया, सायनोबैक्टीरिया या/एवं एककोशिकीय शैवाल) या प्रोटोजोआ एवं एक बहुकोशिकीय जीव (जुगाली करने वाले, लोअर टर्माईट्स, लकड़ी खाने वाले तिलचट्टे, पौधे) के बीच पाया जाता है।


31.मुरैना के समीप स्थित चौंसठ योगिनी मंदिर के संदर्भ में, निम्नलिखित कथनों पर विचार कीजिए :

  1. यह कच्छपघात राजवंश के शासनकाल में निर्मित एक वृत्ताकार मंदिर है।
  2. यह भारत में निर्मित एकमात्र वृत्ताकार मंदिर है।
  3. इसका उद्देश्य इस क्षेत्र में वैष्णव पूजा-पद्धति को प्रोत्साहन देना था।
  4. इसके डिजाइन से यह लोकप्रिय धारणा बनी कि यह भारतीय संसद भवन के लिए प्रेरणा-स्रोत रहा था।

उपर्युक्त कथनों में से कौन-से सही हैं?

  1. 1 और 2
  2. केवल 2 और 3
  3. 1 केवल 4
  4. 2, 3 और 4

उत्तर (c) चौसठ योगिनी मंदिर, मितावली, भारत में सबसे दुर्लभ हिंदू मंदिरों में से एक है। इसकी योजना वृत्ताकार है एवं यह चौसठ योगिनी के तांत्रिक संप्रदाय को समर्पित है। अधिकांश हिंदू मंदिर वर्ग या आयताकार योजना पर बनाए गए हैं।

भारत का संसद भवन वृत्ताकार चैसठ योगिनी मंदिर से प्रेरित था।

अतः, कथन 4 सही है एवं विकल्प (a) तथा (b) गलत हैं। चूंकि यह एकमात्र गोलाकार मंदिर नहीं है, इसलिए 2 गलत है एवं विकल्प (c) सही है।

1323 इस्वी (विक्रम संवत 1383) के एक शिलालेख के अनुसार, मंदिर का निर्माण कच्छपघात राजा देवपाल (1055-1075 इस्वी) द्वारा किया गया था। ऐसा कहा जाता है कि यह मंदिर सूर्य के गोचर के आधार पर ज्योतिष एवं गणित में शिक्षा प्रदान करने का स्थान था।

यह बाहरी रूप से गोलाकार है जिसकी त्रिज्या 170 फीट है एवं इसके आंतरिक भाग में 64 छोटे कक्ष हैं। मुख्य केंद्रीय मंदिर के भीतर स्लैब कवरिंग हैं जिनमें वर्षा जल को एक बड़े भूमिगत भंडारण में निकालने के लिए छिद्र हैं। छत से पाइप लाइन बारिश के पानी को भंडारण तक ले जाती है। कक्ष एवं मुख्य मंदिर सपाट शीर्ष पर हैं, लेकिन ऐसा माना जाता है कि शुरू में प्रत्येक के ऊपर एक शिखर था।

मूल रूप से 64 सहायक मंदिरों में रखी गई 64 योगिनियाँ अब गायब हैं, प्रत्येक कक्ष में एक शिवलिंग भी स्थापित है। केंद्रीय मंदिर में एक शिवलिंग भी है।


32.निम्नलिखित में से कौन-सा प्राचीन नगर अपने उन्नत जल संचयन और प्रबंधन प्रणाली के लिए सुप्रसिद्ध है, जहाँ बाँधों की श्रृंखला का निर्माण किया गया था और संबद्ध जलाशयों में नहर के माध्यम से जल को प्रवाहित किया जाता था?

  1. धौलावीरा
  2. कालीबंगा
  3. राखीगढ़ी
  4. रोपड़

उत्तर (a) यह एक अत्यंत सरल एवं सीधा प्रश्न था।

धोलावीरा शहर का विन्यास, अपने सुनहरे दिनों के दौरान, नियोजित एवं अलग-अलग शहरी आवासीय क्षेत्रों के साथ नियोजित शहर का एक उदाहरण है, जो संभवतः अलग-अलग व्यावसायिक गतिविधियों एवं एक स्तरीकृत समाज पर आधारित है। जल दोहन प्रणालियों, जल निकासी प्रणालियों के साथ-साथ वास्तुशिल्प एवं तकनीकी रूप से विकसित सुविधाओं में तकनीकी प्रगति स्थानीय सामग्रियों के डिजाइन, निष्पादन एवं प्रभावी दोहन में परिलक्षित होती है।

हड़प्पा के पूर्ववर्ती शहरों के विपरीत जो आम तौर पर नदियों एवं पानी के बारहमासी स्रोतों के पास स्थित थे, धोलावीरा खादिर द्वीप में स्थित था जो विभिन्न खनिज एवं कच्चे माल के स्रोतों (तांबा, खोल, एगेट-कारेलियन, स्टीटाइट, सीसा, बैंडेड चूना पत्थर) का दोहन करने के साथ मगन (आधुनिक ओमान प्रायद्वीप) एवं मेसोपोटामिया क्षेत्रों में आंतरिक एवं बाहरी व्यापार की सुविधा के लिए रणनीतिक था।


33.सत्रहवीं शताब्दी के पहले चतुर्थांश में, निम्नलिखित में से कहाँ इंग्लिश ईस्ट इंडिया कंपनी का कारखाना/ . के कारखाने स्थित था/थे?

  1. भरूच
  2. चिकाकोल
  3. त्रिचिनोपोली

नीचे दिए गए कूट का प्रयोग कर सही उत्तर चुनिए।

  1. केवल 1
  2. 1 और 2
  3. केवल 3
  4. 2 और 3

उत्तर (a) 1623 तक, इंग्लिश ईस्ट इंडिया कंपनी ने सूरत, भरूच, अहमदाबाद, आगरा एवं मसूलीपट्टम में कारखाने स्थापित कर लिए थे। 1625 में, सूरत में ईस्ट इंडिया कंपनी के अधिकारियों ने अपने कारखाने को मजबूत करने का प्रयास किया, लेकिन मुगल साम्राज्य के स्थानीय अधिकारियों द्वारा अंग्रेजी कारखाने के प्रमुखों को तुरंत कैद कर लिया गया।

अंग्रेजों ने 1611 में मसूलीपट्टम में दक्षिण में अपना पहला कारखाना खोला। लेकिन उन्होंने जल्द ही अपनी गतिविधि का केंद्र मद्रास में स्थानांतरित कर दिया, जिसका पट्टा उन्हें 1639 में स्थानीय राजा द्वारा दिया गया था।


34.गुप्त वंश के पतन से लेकर आरंभिक सातवीं शताब्दी में हर्षवर्धन के उत्थान तक उत्तर भारत में निम्नलिखित में से किन राज्यों का शासन था?

  1. मगध के गुप्त
  2. मालवा के परमार
  3. थानेसर के पुष्यभूति
  4. कन्नौज के मौखरि
  5. देवगिरि के यादव
  6. वल्लभी के मैत्रक

नीचे दिए गए कूट का प्रयोग कर सही उत्तर चुनिए।

  1. 1, 2 और 5
  2. 1, 3, 4 और 6
  3. 2, 3 और 4
  4. 5 और 6

उत्तर (b) गुप्त साम्राज्य के पतन के बाद, उत्तरी भारत में चार साम्राज्य थे

  1. मगध के गुप्त
  2. पुष्यभूति
  3. मौखरि, एवं
  4. मैत्रक।

अतः 1 एवं 3 सही है। इसलिए (a), (c) एवं (d) गलत हैं। अतः सही उत्तर (b) है।

यह भी ध्यान दें कि देवगिरी के यादव मध्यकालीन युग के थे। अतः 5 गलत है एवं इसलिए विकल्प (a) एवं (d) गलत हैं।


35. पुर्तगाली लेखक नूनिज के अनुसार, विजयनगर साम्राज्य - में महिलाएं निम्नलिखित में से किन क्षेत्रों में निपुण थीं?

  1. कुश्ती
  2. ज्योतिषशास्त्र
  3. लेखाकरण
  4. भविष्यवाणी

नीचे दिए गए कूट का प्रयोग कर सही उत्तर चुनिए।

  1. केवल 1, 2 और 3
  2. केवल 1, 3 और 4
  3. केवल 2 और 4
  4. 1, 2, 3 और 4

उत्तर (d) विजयनगर साम्राज्य की स्थापना हरिहर एवं बुक्का ने की थी एवं शासन काल 1336 इस्वी से 1646 इस्वी तक था।

इस साम्राज्य में महिलाओं ने उच्च स्थान प्राप्त किया एवं साम्राज्य के राजनीतिक, सामाजिक एवं साहित्यिक जीवन में सक्रिय रही। उन्हें कुश्ती में, अपराध एवं रक्षा के विभिन्न हथियारों के उपयोग में, संगीत एवं ललित कलाओं में शिक्षित एवं प्रशिक्षित किया गया था। कुछ महिलाओं ने उच्च कोटि की शिक्षा भी प्राप्त की। नूनिज़ लिखते हैं कि राजाओं के पास महिला ज्योतिषी, क्लर्क, लेखाकार, गार्ड एवं पहलवान थे।

स्त्रोत - https://www.thehindubusinessline.com/news/variety/Alpha-women-of-Hampi/article20396779.ece

विजयनगर समाज में महिलाओं का स्थान सम्मानजनक था। कुशल संगीतकारों एवं नर्तकियों के अलावा, महिलाएं ज्योतिषी, भविष्यवक्ता, न्यायाधीश, लेखक एवं प्रख्यात साहित्यकार के रूप में उत्कृष्ट थीं, एवं कई प्रशासन एवं व्यापार के मामलों में सक्रिय रूप से शामिल थीं।


36.आंध्र प्रदेश में मदनपल्ली के संदर्भ में, निम्नलिखित में से कौन-सा कथन सही है?

  1. पिंगलि वेंकैया ने यहाँ भारतीय राष्ट्रीय ध्वज तिरंगे का डिजाइन किया।
  2. पट्टाभि सीतारमैया ने यहाँ से आंध्र क्षेत्र में भारत छोड़ो आंदोलन का नेतृत्व किया।
  3. रवीन्द्रनाथ टैगोर ने यहाँ राष्ट्रगान का बांग्ला से अंग्रेजी में अनुवाद किया।
  4. मैडम ब्लावत्स्की तथा कर्नल ऑलकाट ने सबसे पहले यहाँ थियोसोफिकल सोसाइटि का मुख्यालय स्थापित किया।

उत्तर (c) रवींद्रनाथ टैगोर ने मदनपल्ली में ‘जन गण मन’ का बंगाली से अंग्रेजी में अनुवाद किया एवं इसे संगीतबद्ध भी किया।

भारतीय राष्ट्रगान को 1911 की शुरुआत में लिखा गया था एवं उसी वर्ष 27 दिसंबर को कलकत्ता में भारतीय राष्ट्रीय कांग्रेस के वार्षिक सत्र में गाया गया था। यह वास्तव में बेसेंट थियोसोफिकल कॉलेज, मदनपल्ली था, जहां श्री टैगोर फरवरी 1919 में कुछ दिनों के लिए रुके थे कि जब राष्ट्रगान की परिचित धुन सेट की गई थी।

यह शिक्षाविद् डॉ. जेम्स हेनरी कजिन्स की पत्नी मार्गरेट कजिन्स थीं, जिन्होंने ‘जन गण मन’ के लिए धुन तैयार की थी।

डॉ. जे.एच. कजिन्स उस समय मदनपल्ले कॉलेज के प्राचार्य थे, जिसकी स्थापना डॉ. एनी बेसेंट ने की थी।


37.निम्नलिखित युग्मों पर विचार कीजिए:

(ऐतिहासिक स्थान)            (ख्याति का कारण)

  1. बुर्जहोम      :       शैलकृत देव मंदिर
  2. चंद्रकेतुगढ़      :       टेराकोटा कला
  3. गणेश्वर      :       ताम्र कलाकृतियाँ

उपर्युक्त युग्मों में से कौन-सा/कौन-से सही सुमेलित

  1. केवल 1
  2. 1 और 2
  3. केवल 3
  4. 2 और 3

उत्तर (d) गणेश्वर-जोधपुरा ताम्रपाषाण संस्कृति की खोज पहली बार 1970 के दशक में क्रमशः जोधपुर एवं गणेश्वर में खुदाई के बाद हुई थी। गणेश्वर के खुदाई स्थल से बहुत सी तांबे की वस्तुएं मिलीं जैसे कि तीर, भाला, छेनी, मछली के हुक, रेजर ब्लेड, अंगूठियां, हेयरपिन, चूड़ियां, सुरमा की छड़ें, डबल स्पाइरल हेड पिन जैसे अन्य आभूषण। अतः 3 सही है एवं विकल्प (a) तथा (b) गलत हैं।

कोलकाता से लगभग 50 किमी उत्तर-पूर्व में, पश्चिम बंगाल के बेराचंपा के छोटे से गाँव के पास, चंद्रकेतुगढ़ का 2,300 साल पुराना स्थल है, जो सबसे अद्भुत टेराकोटा मूर्तियों से भरा है।

मोहनजोदड़ो के खंडहरों की खोज करने वाले पुरातत्वविद् राखलदास बनर्जी ने चंद्रकेतुगढ़ का दौरा किया एवं टेराकोटा कलाकृतियों की एक विशाल भंडार पाया। अतः 2 भी सही है, एवं सही उत्तर (d) है।

यदि आप 1 के बारे में नहीं जानते हैं तो भी इस प्रश्न को हल किया जा सकता है।


38.निम्नलिखित कथनों पर विचार कीजिए:

  1. इल्तुतमिश के शासनकाल में, चंगेज़ खान भगोड़े ख्वारिज्म युवराज की खोज में सिंधु नदी तक पहुँचा था।
  2. मुहम्मद बिन तुग़लक के शासनकाल में, तैमूर ने मुल्तान पर अधिकार किया था और सिंधु नदी पार की थी।
  3. विजयनगर साम्राज्य के देव राय द्वितीय के शासनकाल में, वास्को द गामा केरल के तट पर पहुंचा था।

उपर्युक्त कथनों में से कौन-सा/कौन-से सही है/हैं?

  1. केवल 1
  2. 1 और 2
  3. केवल 3
  4. 2 और 3

उत्तर (a) 1221 में, महान मंगोल विजेता चंगेज खान लगभग 50,000 सैनिकों के साथ सिंधु नदी के पश्चिमी तट पर विजयी हुए। उन्होंने ख्वारिज़्म के शाह जलालुद्धीन को हराया था। पंजाब के कालाबाग शहर के पास हुई इस लड़ाई में जलालुद्धीन की सेना नष्ट हो गई। उस समय उत्तरी भारत पर शम्सुद्दिन इल्तुतमिश का शासन था। अतः 1 सही है इसलिए विकल्प (c) एवं (d) गलत हैं।

दिल्ली की लड़ाई-सन् 1398 में अपनी राजधानी समरकंद से मध्य एशिया पर शासन करने वाले शासक मंगोल-तुर्की योद्धा तैमूर ने दक्षिण में भारत पर हमला करने का निश्चय किया। एक धर्मनिष्ठ मुस्लिम, तैमूर ने आरोप लगाया कि दिल्ली के उनके सह-धर्मवादी सुल्तान नसीरुद्दीन महमूद (तुगलक नहीं) अपनी हिंदू प्रजा के प्रति बहुत उदार थे। अतः, 2 गलत है इसलिए सही उत्तर (a) है।



39.निम्नलिखित कथनों पर विचार कीजिए:

  1. संत फ्रांसिस जेवियर, जेसुइट संघ (ऑर्डर) के संस्थापक सदस्यों में से एक थे।
  2. संत फ्रांसिस जेवियर की मृत्यु गोवा में हुई तथा यहाँ उन्हें समर्पित एक गिरजाघर है।
  3. गोवा में प्रति वर्ष संत फ्रांसिस जेवियर के भोज 1 का अनुष्ठान किया जाता है।

उपर्युक्त कथनों में से कौन-से सही हैं?

  1. केवल 1 और 2
  2. केवल 2 और 3
  3. केवल 1 और 3
  4. 1, 2 और 3

उत्तर (c) संत फ्रांसिस ज़ेवियर -किंगडम ऑफ नवार (स्पेन में) में जन्मे, संत फ्रांसिस ज़ेवियर ने एशिया में एक मिशन का नेतृत्व किया, मुख्य रूप से उस समय के पुर्तगाली साम्राज्य में एवं ज्यादातर भारत में प्रचार कार्य में प्रभावशाली थे।

फ्रांसिस जेवियर 6 मई, 1542 को पूर्व में पुर्तगाली गतिविधि के केंद्र गोवा आए। अगले तीन वर्षों में उन्होंने भारत के दक्षिणपूर्वी तट पर साधारण, गरीब मोती मछुआरों, परवों के बीच बिताया। फ्रांसिस ने उनके बीच धर्म के प्रवचन देने एवं पुष्टि करने के लिए गांव-गांव की अथक यात्रा की। उनकी स्पष्ट अच्छाई एवं उनके दृढ़ विश्वास के बल ने मौखिक संचार की सीमाओं पर विजय प्राप्त की।

उसे पता चल गया था कि जापान के धर्म परिवर्तन का रास्ता चीन से होकर जाता है, जापानी ज्ञान की तलाश में चीन की ओर देखते थे। हालाँकि, वे कभी चीन नहीं पहुँच सके। 3 दिसंबर, 1552 को, चीन में जहाँ उस समय विदेशीयों का प्रवेश वर्जित था सुरक्षित प्रवेश करने के प्रयास में फ्रांसिस की सैन्सियन द्वीप (चीनी तट से दूर शांगचुआन) पर बुखार से मृत्यु हो गई।

अतः 2 गलत है, इसलिए विकल्प (a), (b) एवं (d) गलत हैं।

शॉर्टकट - यदि आप जानते हैं कि 2 गलत है, उत्तर सीधे (c) प्राप्त किया जा सकता था।


40.प्राचीन भारत के इतिहास के संदर्भ में, निम्नलिखित कथनों में से कौन-सा/कौन-से सही है/हैं?

  1. मिताक्षरा ऊँची जाति की सिविल विधि थी और दायभाग निम्न जाति की सिविल विधि थी।
  2. मिताक्षरा व्यवस्था में, पुत्र अपने पिता के जीवनकाल में ही संपत्ति पर अधिकार का दावा कर सकते थे, जबकि दायभाग व्यवस्था में पिता की मृत्यु के उपरांत ही पुत्र संपत्ति पर अधिकार का दावा कर सकते थे।
  3. मिताक्षरा व्यवस्था किसी परिवार के केवल पुरुष सदस्यों के संपत्ति-संबंधी मामलों पर विचार करती है, जबकि दायभाग व्यवस्था किसी परिवार के पुरुष एवं महिला सदस्यों, दोनों के संपत्ति-संबंधी मामलों पर विचार करती है।

नीचे दिए गए कूट का प्रयोग कर सही उत्तर चुनिए।

  1. 1 और 2
  2. केवल 2
  3. 1 और 3
  4. केवल 3

उत्तर (b) दायभाग एवं मिताक्षरा कानून के दो संकुल हैं जो भारतीय कानून के तहत हिंदू अविभाजित परिवार के उत्तराधिकार कानून को नियंत्रित करते हैं।

मिताक्षरा लॉ संकुल - एक संयुक्त परिवार परिवार के केवल पुरुष सदस्य को संदर्भित करता है एवं उसके बेटे, पोते एवं परपोते को शामिल करता है। उनका संयुक्त परिवार में सामूहिक रूप से सह-स्वामित्व/सहदायिकी है। इस प्रकार, जन्म से एक पुत्र संयुक्त परिवार की पैतृक संपत्ति में रुचि प्राप्त करता है।

दायभाग लॉ संकुल- बेटे का जन्म से कोई स्वतः स्वामित्व नहीं होता है, किंतु वह अपने पिता के निधन पर इसे प्राप्त करता है। अतः 2 निश्चित रूप से सही है। इसलिए, विकल्प (c) एवं (d) गलत हैं।

मिताक्षरा व्यवस्था में पत्नी विभाजन की मांग नहीं कर सकती। हालाँकि, उसे अपने पति एवं अपने बेटों के बीच हुए किसी भी विभाजन में हिस्सेदारी का अधिकार है। दायाभाग के तहत यह अधिकार महिलाओं के लिए मौजूद नहीं है क्योंकि बेटे विभाजन की मांग नहीं कर सकते क्योंकि पिता पूर्ण मालिक है। दोनों ही व्यवस्थाओं में पुत्रों के किसी भी विभाजन में माता पुत्र के बराबर हिस्से की हकदार होती है। इसी तरह, जब कोई बेटा बंटवारे से पहले मर जाता है एवं माँ को अपना उत्तराधिकारी छोड़ देता है, तब बाकी बेटों के बीच विभाजन के समय माँ अपने मृत बेटे के हिस्से के साथ-साथ अपने अधिकार में हिस्से की भी हकदार होती है।


41.प्राचीन भारत के इतिहास के संदर्भ में, भवभूति, हस्तिमल्ल तथा क्षेमेश्वर क्यों प्रसिद्ध थे?

  1. जैन साधु
  2. नाटककार
  3. मंदिर वास्तुकार
  4. दार्शनिक

उत्तर (b) भवभूति भारत के 8वीं शताब्दी के विद्वान थे, जो संस्कृत में लिखे गए अपने नाटकों एवं कविताओं के लिए विख्यात थे।

उनके नाटकों को कालिदास के कार्यों के समान माना जाता है। ऐसा लगता है कि भवभूति कन्नौज के राजा यशोवर्मन के दरबारी कवि थे एवं राजा, जिन्होंने कश्मीर के राजा ललितादित्य मुक्तापाद के साथ युद्ध किया था एवं उनसे हार गए थे, के दल में जाने जाते थे।

हस्तिमल्ला 13वीं सदी के होयसल साम्राज्य के कन्नड़ कवि एवं लेखक थे। उन्होंने पूर्वपुराण लिखा।

क्षेमेश्वर (990 इस्वी -1070 इस्वी) भारत में कश्मीर के 11वीं सदी के संस्कृत कवि थे।

शॉर्टकट - यदि आप इन तीनों में से किसी एक का पेशा जानते हैं, तो प्रश्न हल किया जा सकता है।


42.निम्नलिखित कथनों पर विचार कीजिए:

  1. 1919 के मांटेग्यू-चेम्सफ़ोर्ड सुधारों में, 21 वर्ष से अधिक आयु की सभी महिलाओं के लिए मताधिकार की संस्तुति की गई।
  2. 1935 के गवर्नमेंट ऑफ इंडिया ऐक्ट में विधानमंडल में महिलाओं के लिए आरक्षित स्थानों का प्रावधान किया गया।

उपर्युक्त कथनों में से कौन-सा/कौन-से सही है/हैं?

  1. केवल 1
  2. केवल 2
  3. 1 और 2 दोनों
  4. न तो 1 और न ही 2

उत्तर (b) 1918 में, जब ब्रिटेन ने महिला संपत्ति धारकों को सीमित मताधिकार दिया, तो यह कानून साम्राज्य के अन्य हिस्सों में ब्रिटिश नागरिकों पर लागू नहीं हुआ।

महिलाओं द्वारा प्रस्तुत याचिकाओं के बावजूद, मोंटेग्यू-चेम्सफोर्ड सुधारों में महिलाओं की मांगों को नजरअंदाज कर दिया गया। यद्यपि उन्हें मतदान का अधिकार नहीं दिया गया था, न ही चुनाव में खड़े होने का अधिकार, फिर भी भारत सरकार अधिनियम 1919 ने प्रांतीय परिषदों को यह निर्धारित करने की अनुमति दी थी कि महिलाएं मतदान कर सकती हैं, बशर्ते कि वे कड़े संपत्ति, आय या शैक्षिक स्तर को पूरा करती हों। अतः 1 गलत है एवं इसलिए विकल्प (b) तथा (c) गलत हैं।

1919 एवं 1929 के बीच, सभी ब्रिटिश प्रांतों के साथ-साथ अधिकांश रियासतों ने महिलाओं को मताधिकार दिया गया एवं कुछ मामलों में, उन्हें स्थानीय चुनावों में खड़े होने की अनुमति भी दी गई।

तीन गोलमेजों सम्मेलनों की रिपोर्ट ब्रिटिश संसद की संयुक्त समिति को भेजी गई थी, जिसमें मतदान की आयु को कम करके 21 करने की सिफारिश की गई थी, लेकिन संपत्ति एवं साक्षरता प्रतिबंधों को बनाए रखने के साथ-साथ उनकी वैवाहिक स्थिति पर महिलाओं की योग्यता को आधार बनाया गया था। इसने प्रांतीय विधानसभाओं में महिलाओं एवं जातीय समूहों के लिए विशेष कोटा भी प्रदान किया। इन प्रावधानों को भारत सरकार अधिनियम 1935 में शामिल किया गया था। हालांकि इसने चुनावी पात्रता को बढ़ाया, फिर भी अधिनियम ने भारत में केवल 2.5% महिलाओं को वोट देने की अनुमति दी। अतः कथन 2 सही है एवं विकल्प (b) सही उत्तर है।


43.भारतीय इतिहास में 8 अगस्त, 1942 के संदर्भ में, निम्नलिखित कथनों में से कौन-सा सही है?

  1. ए० आइ० सी० सी० द्वारा भारत छोड़ो प्रस्ताव अंगीकार किया गया।
  2. वायसराय की एक्जेक्यूटिव काउंसिल का विस्तार अधिक संख्या में भारतीयों को सम्मिलित करने के लिए किया गया।
  3. सात प्रांतों में कांग्रेस मंत्रिमंडलों'ने त्यागपत्र दिया।
  4. क्रिप्स ने प्रस्ताव रखा कि द्वितीय विश्व युद्ध समाप्त होते ही संपूर्ण डोमिनियन स्टेटस वाले भारतीय संघ की स्थापना की जाएगी।

उत्तर (a) अगस्त 8, 1942 को, महात्मा गांधी ने मुंबई (तब बॉम्बे) में ब्रिटिश शासन से स्वतंत्रता के लिए भारत छोड़ो आंदोलन शुरू किया। भारत छोड़ो आंदोलन, जिसे अगस्त आंदोलन के रूप में भी जाना जाता है, गांधी द्वारा सत्याग्रह (स्वतंत्रता) के लिए शुरू किया गया एक सविनय अवज्ञा आंदोलन था।

8 अगस्त, 1942 को अखिल भारतीय कांग्रेस कमेटी (एआईसीसी) के बॉम्बे सत्र में भारत छोड़ो प्रस्ताव पारित किया गया था।

उस दिन बॉम्बे के गोवालिया टैंक से दिए अपने भाषण में श्री गांधी ने भारतीयों से अहिंसक सविनय अवज्ञा का पालन करने के लिए कहा। उन्होंने जनता से एक स्वतंत्र राष्ट्र के रूप में कार्य करने के लिए कहा।


44.इनमें से कौन अंग्रेजी में अनूदित प्राचीन भारतीय धार्मिक गीतिकाव्य–'सॉन्स फ्रॉम प्रिजन' से संबद्ध हैं?

  1. बाल गंगाधर तिलक
  2. जवाहरलाल नेहरू
  3. मोहनदास करमचंद गाँधी
  4. सरोजिनी नायडू

उत्तर (c) इसके लेखक महात्मा गांधी थे।


45.मध्यकालीन भारत के संदर्भ में, निम्नलिखित में से कौन-सा आकार की दृष्टि से आरोही क्रम में सही अनुक्रम है?

  1. परगना-सरकार-सूबा
  2. सरकार—परगना-सूबा
  3. सूबा सरकार—परगना
  4. परगना-सूबा सरकार

उत्तर (a) ध्यान दें कि ‘आरोही’ क्रम पूछा जाता है।

एक सूबा मुगल साम्राज्य में एक प्रांत (राज्य) के लिए शब्द था। सूबे के राज्यपाल/शासक को सूबेदार के रूप में जाना जाता था। 1572-1580 के वर्षों के अपने प्रशासनिक सुधारों के दौरान सम्राट अकबर द्वारा सूबों की स्थापना की गई थी।

पूरे सूबे को सरकार (जिलों) में विभाजित किया गया था, प्रत्येक की देखभाल एक सैन्य अधिकारी द्वारा की जाती थी जिसे प्रमुख शिकदार कहा जाता था। प्रत्येक सरकार को परगना नामक छोटी इकाइयों में विभाजित किया गया था। प्रत्येक परगना में एक शिकदार (सैन्य अधिकारी), अमीन या मुंसिफ (नागरिक न्यायाधीश), एक फोत्तदार (कोषाध्यक्ष) एवं दो कारकुन (क्लर्क या लेखक) थे।


46.इनमें से कौन सेक्रेटरी के रूप में हिन्दू फीमेल स्कूल से संबद्ध थे/थीं, जो बाद में बेथ्यून फीमेल स्कूल के नाम से जाना जाने लगा ?

  1. एनी बेसेंट
  2. देवेन्द्रनाथ टैगोर
  3. ईश्वरचंद्र विद्यासागर
  4. सरोजिनी नायडू

उत्तर (c) हिंदू फीमेल स्कूल की उत्पत्ति जॉन इलियट ड्रिंकवाटर बेथ्यून (1801-1851) से संबंधित है। जो 1849 में हिंदू फीमेल स्कूल के रूप में जो शुरू हुआ, एवं बाद में 1856 में इसका नाम बदलकर बेथ्यून स्कूल कर दिया गया।तब स्कूल की प्रबंध समिति का गठन किया गया एवं महिला मुक्ति के समर्थक पंडित ईश्वर चंद्र विद्यासागर को सचिव बनाया गया।

ईश्वर चंद्र विद्यासागर (1820-1891) हिंदू विधवा पुनर्विवाह के लिए सबसे प्रमुख प्रचारक थे, उन्होंने कड़े विरोध के बावजूद विधान परिषद में याचिका दायर की, एवं लॉर्ड डलहौजी ने व्यक्तिगत रूप से बिल को अंतिम रूप दिया एवं हिंदू विधवा पुनर्विवाह अधिनियम, 1856 पारित किया गया।



47.औपनिवेशिक भारत के संदर्भ में, शाह नवाज़ खान, प्रेम कुमार सहगल और गुरबख्श सिंह ढिल्लों याद किए जाते हैं

  1. स्वदेशी और बहिष्कार आंदोलन के नेता के रूप
  2. 1946 की अंतरिम सरकार के सदस्यों के रूप में
  3. संविधान सभा में प्रारूप समिति के सदस्यों के रूप
  4. आज़ाद हिंद फौज (इंडियन नैशनल आर्मी) के अधिकारियों के रूप में

उत्तर (d) वे तीनो आज़ाद हिंद फौज (आईएनए) अधिकारी थे। यह एक आसान प्रश्न था!

नवंबर 1945 एवं मई 1946 के बीच, दिल्ली के लाल किले में सार्वजनिक रूप से लगभग दस कोर्ट-मार्शल आयोजित किए गए। इन्हें लाल किला ट्रायल भी कहा जाता था, एवं ये ब्रिटिश राज की एक बड़ी गलती थी।

ब्रिटिश-भारतीय सेना के कमांडर-इन-चीफ क्लॉड औचिनलेक ने आशा व्यक्त की कि लाल किले में सार्वजनिक परीक्षण करने से जनता की राय आईएनए के खिलाफ हो जाएगी। प्रेम सहगल, गुरुबख्श सिंह ढिल्लों एवं शाह नवाज खान की पहली एवं सबसे प्रसिद्ध संयुक्त कोर्ट-मार्शल - अंग्रेजों की उम्मीदों के विपरित निकली।

ये नेताजी सुभाष बोस के नेतृत्व में आईएनए के अधिकारी थे।


48.भारतीय इतिहास के संदर्भ में, निम्नलिखित कथनों में से कौन-सा/कौन-से सही है/हैं? /

  1. हैदराबाद राज्य से आरकोट की निज़ामत का उदय हुआ।
  2. विजयनगर साम्राज्य से मैसूर राज्य का उदय हुआ।
  3. रूहेलखंड राज्य का गठन, अहमद शाह दुर्रानी द्वारा अधिकृत राज्यक्षेत्र में से हुआ।

नीचे दिए गए कूट का प्रयोग कर सही उत्तर चुनिए।

  1. 1 और 2
  2. केवल 2
  3. 2 और 3
  4. केवल 3

उत्तर (a) रूहेलखंड राज्य 1721 में गिरते मुगल साम्राज्य के बाद उभरा एवं 1774 तक जब अंग्रेजों ने उस पर कब्जा कर लिया एवं इसके काफी इलाके को रामपुर रियासत में बदल दिया, अस्तित्व में रहा। प्राचीन बरहा राजवंश के वंशज, नवाब अली मोहम्मद खान, रूहेलखंड के पहले नवाब बने। उन्हें पहले चौदह वर्ष की आयु में विभिन्न अफगान प्रमुखों द्वारा अधिपति के रूप में चुना गया था। उन्होंने ढहते मुगल साम्राज्य से भविष्य के राज्य को तराश कर रोहिल्ला राजवंश की स्थापना की। अतः 3 गलत है। इसलिए विकल्प (c) एवं (d) गलत हैं।

कर्नाटक (आर्कोट) के नवाबों ने लगभग 1690 एवं 1855 के बीच दक्षिण भारत के कर्नाटक क्षेत्र पर शासन किया। यह हैदराबाद-दक्कन का उपनिवेश था। यही वह समय था जब मुगल साम्राज्य ने मराठा साम्राज्य के बढ़ते प्रभाव एवं बाद में ब्रिटिश राज के उदय का मार्ग प्रशस्त किया। अतः 1 सही है, एवं विकल्प (a) सही उत्तर है।


49.निम्नलिखित कथनों में से कौन-सा सही है?

  1. अजंता गुफाएँ, वाघोरा नदी की घाटी में स्थित हैं।
  2. साँची स्तूप, चंबल नदी की घाटी में स्थित है। से जाना जाने लगा।
  3. पांडू-लेणा गुफा देव मंदिर, नर्मदा नदी की घाटी में स्थित हैं।
  4. अमरावती स्तूप, गोदावरी नदी की घाटी में स्थित है।

उत्तर (a) अजंता गुफाएं बौद्ध रॉक-कट गुफा मंदिर एवं मठ हैं, जो उत्तर-मध्य महाराष्ट्र के अजंता गांव के पास स्थित हैं। ये उनकी भित्ती चित्रों के लिए माने जाते हैं। मंदिरों को औरंगाबाद से 105 किमी उत्तर-पूर्व में वाघोरा नदी घाटी में 70-फुट (20-मीटर) खड्ड के अंदरूनी हिस्से में ग्रेनाइट की चट्टानों से काट कर बनाया गया है। अतः (a) सही है।

सांची ऐतिहासिक स्थल, पश्चिम-मध्य मध्यप्रदेश राज्य, मध्य भारत, बेतवा नदी के पश्चिम में एवं विदिशा से लगभग 8 किमी दक्षिण पश्चिम में एक ऊंचे पठारी क्षेत्र में स्थित है।

बौद्ध स्मारक पांडू-लेणा गुफाएँ एवं त्रिरश्मी गुफाएं 24 रॉक कट गुफाओं का एक समूह हैं। यह महाराष्ट्र के नासिक शहर के करीब है।

अंग्रेजों को 1797 में मेजर कॉलिन मैकेंजी की यात्रा के बाद अमरावती में स्तूप के खंडहरों के बारे में पता चला। यह दक्षिण-पूर्व भारत में आंध्र प्रदेश के अमरावती जिले में कृष्णा नदी के दाहिने किनारे पर स्थित है।


50.निम्नलिखित कथनों पर विचार कीजिए :

  1. यूनिसेफ (UNICEF) द्वारा 21 फरवरी को अंतर्राष्ट्रीय मातृभाषा दिवस घोषित किया गया।
  2. पाकिस्तान की संविधान सभा में यह माँग रखी गई कि राष्ट्रभाषाओं में बांला को भी सम्मिलित किया जाए।

उपर्युक्त कथनों में से कौन-सा/कौन-से सही है/हैं?

  1. केवल 1
  2. केवल 2
  3. 1 और 2 दोनों
  4. न तो 1 और न ही 2

उत्तर (c) अंतर्राष्ट्रीय मातृभाषा दिवस मनाने का विचार बांग्लादेश की पहल थी। इसे 1999 के यूनेस्को आम सम्मेलन में अनुमोदित किया गया था एवं वर्ष 2000 से दुनिया भर में मनाया जा रहा है। यह प्रतिवर्ष 21 फरवरी को मनाया जाता है। अतः, कथन 1 सही है इसलिए विकल्प (b)तथा (d)गलत हैं।

पाकिस्तान की संविधान सभा पहली बार 11 अगस्त, 1947 को स्वतंत्रता की पूर्व संध्या एवं ब्रिटिश शासन की समाप्ति पर बुलाई गई थी।

1948 में, पाकिस्तान की तत्कालीन सरकार ने उर्दू को पाकिस्तान की एकमात्र राष्ट्रीय भाषा घोषित किया, भले ही बंगाली या बांग्ला पूर्वी पाकिस्तान एवं पश्चिमी पाकिस्तान को मिलाकर अधिकांश लोगों द्वारा बोली जाती थी। पूर्वी पाकिस्तान के लोगों ने विरोध किया, एवं मांग की कि उर्दू के अलावा बांग्ला को कम से कम राष्ट्रीय भाषाओं में से एक होना चाहिए। यह मांग 23 फरवरी 1948 को पूर्वी पाकिस्तान के धीरेंद्रनाथ दत्ता ने पाकिस्तान की संविधान सभा में उठाई थी। अतः, 2 भी सही है।



51.स्थायी कृषि (पर्माकल्चर), पारंपरिक रासायनिक कृषि से किस तरह भिन्न है?

  1. स्थायी कृषि एकधान्य कृषि पद्धति को हतोत्साहित करता है, किन्तु पारंपरिक रासायनिक कृषि में एकधान्य कृषि पद्धति की प्रधानता है।
  2. पारंपरिक रासायनिक कृषि के कारण मृदा की लवणता में वृद्धि हो सकती है, किन्तु इस तरह की परिघटना स्थायी कृषि में दृष्टिगोचर नहीं होती
  3. पारंपरिक रासायनिक कृषि अर्धशुष्क क्षेत्रों में आसानी से संभव है, किन्तु ऐसे क्षेत्रों में स्थायी कृषि इतनी आसानी से संभव नहीं है।
  4. मल्च बनाने (मल्चिंग) की प्रथा स्थायी कृषि में काफी महत्त्वपूर्ण है, किन्तु पारंपरिक रासायनिक कृषि में ऐसी प्रथा आवश्यक नहीं है।

नीचे दिए गए कूट का प्रयोग कर सही उत्तर चुनिए।

  1. 1 और 3
  2. 1, 2 और 4
  3. केवल 4
  4. 2 और 3

उत्तर (b) स्थायी कृषि (पर्माकल्चर) का सार हमारे घरों, बगीचों, समुदायों एवं व्यवसायों में पारिस्थितिक रूप से स्वस्थ जीवन जीने के तरीके का डिजाइन है। यह प्रकृति के साथ सहयोग करके एवं पृथ्वी एवं उसके लोगों की देखभाल करके बनाया जाता है।

स्थायी कृषि (पर्माकल्चर) भूमि प्रबंधन के लिए एक दृष्टिकोण है जो समृद्ध प्राकृतिक पारिस्थितिक तंत्र में पाई जाने वाली व्यवस्था को अपनाता है। पर्माकल्चर के सिद्धांत एकधान्य कृषि पद्धति को हतोत्साहित करते हैं, यह विभिन्न प्रकार के अनाज, फलों एवं सब्जियों को उगाने के अवसर को खोलता है, एवं भोजन की टोकरी को बड़ा करता है। अतः कथन 1 सत्य है। केवल संभावित विकल्प (a) एवं (b) हैं।

यह सिद्ध है कि पारंपरिक रासायनिक खेती में कीटनाशकों एवं उर्वरकों के उपयोग से मिट्टी की लवणता बढ़ जाती है, लेकिन स्थायी कृषि (पर्माकल्चर) के सिद्धांत ऐसी प्रथाओं के उपयोग पर रोक लगाते हैं। अतः कथन 2 भी सत्य है।

इसलिए, विकल्प (b) सही उत्तर है।


52.'ताड़ तेल (पाम ऑयल)' के संदर्भ में, निम्नलिखित कथनों पर विचार कीजिए :

  1. ताड़ तेल वृक्ष दक्षिण-पूर्व एशिया में प्राकृतिक रूप में पाया जाता है।
  2. ताड़ तेल लिपस्टिक और इत्र बनाने वाले कुछ का उद्योगों के लिए कच्चा माल है।
  3. ताड़ तेल का उपयोग जैव डीज़ल के उत्पादन में किया जा सकता है।

उपर्युक्त कथनों में से कौन-से सही हैं?

  1. केवल 1 और 2
  2. केवल 2 और 3
  3. केवल 1 और 3
  4. 1, 2 और 3

उत्तर (b). कथन 1 गलत है, क्योंकि ताड़ के तेल का पेड़ मूलतः अफ्रीका की वनस्पति है, लेकिन 100 साल पहले दक्षिण-पूर्व एशिया में एक सजावटी पेड़ की फसल के रूप में लाया गया था। आज, इंडोनेशिया एवं मलेशिया वैश्विक आपूर्ति का 85% से अधिक बनाते हैं, लेकिन 42 अन्य देश भी हैं जो ताड़ के तेल का उत्पादन करते हैं।

ताड़ के पेड़ (एलाइस गाइनेन्सिस जैक) जो पश्चिम अफ्रीका की वनस्पति है एवं एवं प्रचुर मात्रा में पानी के साथ उष्णकटिबंधीय जलवायु में सबसे अच्छा बढ़ता है से ताड़ का तेल प्राप्त होता है।

ताड़ के तेल के कुल उत्पादन का तीन-चैथाई भोजन, विशेष रूप से खाना पकाने के तेल एवं प्रसंस्कृत तेल तथा वसा के लिए उपयोग किया जाता है।

कथन 1 गलत है, इसलिए सही विकल्प (b) है।


53.सिंधु नदी प्रणाली के संदर्भ में, निम्नलिखित चार नदियों में से तीन नदियाँ इनमें से किसी एक नदी में मिलती हैं जो सीधे सिंधु नदी से मिलती है। निम्नलिखित में से वह नदी कौन-सी है, जो सिंधु नदी से सीधे मिलती है?

  1. चेनाब
  2. झेलम
  3. रावी
  4. सतलुज

उत्तर (a). नदियों एवं सहायक नदियों पर एक अपेक्षित प्रश्न! नीचे दिए गए चित्र से स्पष्ट है कि चेनाब नदी सीधे सिंधु नदी में मिलती है।

सिंधु से मिलने से पहले सतलुज, रावी एवं झेलम चिनाब में मिलती हैं।



54.भारत के संदर्भ में डीडवाना, कुचामन, सरगोल और खाटू किनके नाम हैं?

  1. हिमनद
  2. गरान (मैंग्रोव) क्षेत्र
  3. रामसर क्षेत्र
  4. लवण झील

. उत्तर (d). सभी झीलें हैं।

डीडवाना झील - स्थान - नागौर, राजस्थान, कुल क्षेत्रफल - 50 वर्ग किमी, प्रकार - साल्ट लेक

कुचामन झील - यह एक महाद्वीपीय खारे पानी की झील है जो कुचामन शहर, नागौर जिले, राजस्थान के पास स्थित है। यह 8.5 वर्ग किलोमीटर क्षेत्र में फैली है एवं 380 मीटर समोच्च रेखा से बंधी है।

सरगोल एवं खाटू भी खारे पानी की झीलें हैं।


55.निम्नलिखित नदियों पर विचार कीजिए :

  1. ब्राह्मणी
  2. नागावली
  3. सुवर्णरेखा
  4. वंशधारा

उपर्युक्त में से कौन-सी नदियाँ पूर्वी घाट से निकलती हैं?

  1. 1 और 2
  2. 2 और 4
  3. 3 और 4
  4. 1 और 3

. उत्तर (b). पूर्वी घाट उत्तरी ओडिशा से आंध्र प्रदेश के माध्यम से दक्षिण में तमिलनाडु तक कर्नाटक के कुछ हिस्सों में फैले हुई पर्वत श्रृंखलाएं हैं। प्रायद्वीपीय भारत की चार प्रमुख नदियां गोदावरी, महानदी, कृष्णा एवं कावेरी इनसे होकर गुजरती है। ये पर्वत श्रृंखलाएं बंगाल की खाड़ी के समानांतर चलती हैं। ये दक्षिण भारत के पूर्वी तटीय मैदानों की कई छोटी एवं मध्यम नदियों का स्रोत क्षेत्र है।

पूर्वी घाट से निकलने वाली नदियों में शामिल हैं- बैतरणी नदी, बुधबलंगा नदी, रुशिकुल्या नदी, वंशधारा नदी, पलार नदी, नागावली नदी, चंपावती नदी, गोस्थनी नदी, सारदा नदी, सबरी नदी, सिलेरू नदी, तमिलेरु गुंडलकम्मा नदी, पेनई यारू नदी, स्वर्णमुखी, कुंडू नदी, वेल्लार नदी, पेन्ना नदी।

ब्राह्मणी नदी ओडिशा में एक प्रमुख मौसमी नदी है, जो सांख एवं दक्षिण कोयल नदियों के संगम से बनती है। बैतरणी नदी के साथ मिलकर, यह धामरा में बंगाल की खाड़ी में मिलने से पहले एक बड़ा डेल्टा बनाती है।


56.निम्नलिखित कथनों पर विचार कीजिए:

  1. वैश्विक सागर आयोग (ग्लोबल ओशन कमीशन) अंतर्राष्ट्रीय जल-क्षेत्र में समुद्र-संस्तरीय (सीबेड) खोज और खनन के लिए लाइसेंस प्रदान करता
  2. भारत ने अंतर्राष्ट्रीय जल-क्षेत्र में समुद्र-संस्तरीय खनिज की खोज के लिए लाइसेंस प्राप्त किया है।
  3. 'दुर्लभ मृदा खनिज (रअर अर्थ मिनरल)' अंतर्राष्ट्रीय जल-क्षेत्र में समुद्र अधस्तल पर उपलब्ध है।

उपर्युक्त कथनों में से कौन-से सही हैं?

  1. केवल 1 और 2
  2. केवल 2 और 3
  3. केवल 1 और 3
  4. 1, 2 और 3

उत्तर (b). इंटरनेशनल सीबेड अथॉरिटी (आईएसए) किंग्स्टन, जमैका में स्थित एक अंतर सरकारी निकाय है, जिसे राष्ट्रीय अधिकार क्षेत्र की सीमाओं से परे अंतरराष्ट्रीय समुद्री क्षेत्र, जो विश्व के सभी समुद्रों में पाया जाता है, से सभी खनिज-संबंधी गतिविधियों को व्यवस्थित, विनियमित एवं नियंत्रित करने के लिए स्थापित किया गया था। यह समुद्र के कानून पर संयुक्त राष्ट्र सम्मेलन द्वारा स्थापित एक संगठन है। अतः कथन 1 सही नहीं है। तो केवल (b) ही उत्तर हो सकता है!

(वैश्विक सागर आयोग एक ऐसा निकाय था जो 2013 से कुछ वर्षों तक अस्तित्व में था)

2017 में, मध्य हिंद महासागर बेसिन (सीआईओबी) में समुद्र तल से पॉलीमेटेलिक नोड्यूल्स का पता लगाने के लिए भारत के विशेष अधिकारों को पांच साल के लिए बढ़ा दिया गया है। महत्वपूर्ण दुर्लभ पृथ्वी खनिजों के विशाल भंडार समुद्र तल में पाए जाते हैं। अतः कथन 2 एवं 3 सही हैं।


57.निम्नलिखित में से कौन-सी फसल, न्यूनतम जल-दक्ष (लीस्ट वॉटर-एफिशिएंट) फसल है?

  1. गन्ना
  2. सूरजमुखी
  3. बाजरा
  4. अरहर (रेड ग्राम)

उत्तर (a). एक किलो गन्ने के उत्पादन के लिए लगभग 210 लीटर पानी की आवश्यकता होती है।

बाजरा आश्चर्यजनक रूप से कम पानी की खपत करने वाली फसल है। ज्वार एवं बाजरा के लिए गन्ने एवं केले के लिए आवश्यक पानी के 25% एवं चावल के लिए आवश्यक पानी के 30% की आवश्यकता होती है।

सूरजमुखी को बाजरा एवं गन्ने की तुलना में बहुत कम पानी की आवश्यकता होती है।

अरहर के लिए, मानसून के दौरान सिंचाई की आवश्यकता नहीं होती है, लेकिन सूखे या लंबे समय तक सूखे की स्थिति में विकास की प्रजनन अवधि के दौरान, पूरक/जीवन रक्षक सिंचाई दी जानी चाहिए। एक अच्छी फसल उगाने के लिए कुल 20-25 सेंटीमीटर पानी ही पर्याप्त होता है।


58.निम्नलिखित कथनों पर विचार कीजिए:

  1. उष्णकटिबंधीय क्षेत्र में, व्यापारिक पवन के प्रभाव के कारण पूर्वी खंडों की तुलना में महासागरों के पश्चिमी खंड अधिक उष्ण होते हैं।
  2. शीतोष्ण क्षेत्र में, पश्चिमी पवन पश्चिमी खंडों की तुलना में महासागरों के पूर्वी खंडों को अधिक उष्ण बनाता है।

उपर्युक्त कथनों में से कौन-सा/कौन-से सही है/हैं?

  1. केवल 1
  2. केवल 2
  3. 1 और 2 दोनों
  4. न तो 1 और न ही

उत्तर (c). पृथ्वी की सतह (जमीन एवं पानी दोनों) में हवा के तापमान में अंतर से हवाएं चलती है, जिसमें गर्म हवा ठंडी हवा की तुलना में हल्की होती है। भूमध्य रेखा के पास, सूर्य समुद्र की सतह को गर्म करता है, जिससे सतह पर गर्म हवा ऊपर उठती है एवं उपोष्णकटिबंधीय उच्च दबाव प्रणालियों से भूमध्यरेखीय निम्न दबाव वाले गर्तों में बहने वाली व्यापारिक हवाओं द्वारा प्रतिस्थापित की जाती है। व्यापारिक हवाएँ दिनों तक लगातार चलती हैं एवं पृथ्वी पर सबसे लगातार चलने वाली पवनें हैं। जब व्यापारिक हवाएं गर्म उष्णकटिबंधीय पानी पर चलती हैं, तो वे नमी उठाती हैं एवं पहाड़ी क्षेत्रों में हवा की ओर झुकी ढलानों पर भारी वर्षा करती हैं, जो शुष्क हवा की नीचे की ओर गति के विपरीत होती है जो भूमि पर रेगिस्तानी क्षेत्रों का निर्माण करती है। क्योंकि कर्क रेखा एवं मकर रेखा के बीच का पृथ्वी का क्षेत्र, भूमध्य रेखा के दोनों ओर लगभग 23 डिग्री अक्षांश पर स्थित है, पृथ्वी के बाकी हिस्सों की तुलना में अधिक सौर ताप प्राप्त करता है, गर्म हवा बादल बनाती है एवं वहाँ लगभग हर दिन गरज के साथ बारिश होती है। ।

मौसम एवं जलवायु पर व्यापारिक हवाओं का प्रभाव अल नीनो, ला नीना एवं तूफान/चक्रवात के विकास के साथ देखा जाता है। प्रशांत के दोनों किनारों के बीच दबाव एवं तापमान में अंतर व्यापारिक हवाओं के कारण होता है, पूर्व से पश्चिम की ओर बहने वाली हवा पानी को धक्का देती है, जिससे पश्चिमी प्रशांत क्षेत्र में समुद्र का स्तर ऊंचा हो जाता है, एवं ठंडे पानी को सतह की ओर बढ़ा देता है, जिससे पूर्वी प्रशांत, पश्चिमी प्रशांत की तुलना में लगभग 14 डिग्री फारेनहाइट (7.7 डिग्री सेल्सियस) ठंडा हो जाता है।

अतः कथन 1 सही है।

मध्य अक्षांशों में, भंवर परिसंचरण महाद्वीपों के पूर्वी तटों (महासागर के पश्चिमी किनारों) के साथ ध्रुव की ओर बढ़ते हुए, गर्म पानी लाते हैं। इस प्रकार इन स्थानों के पूर्वी तटों का पानी उस अक्षांश के लिए अपेक्षा से अधिक गर्म होता है। महासागरीय घाटियों (महाद्वीपों के पश्चिमी तट) के पूर्वी किनारों पर वापसी प्रवाह उस अक्षांश के ध्रुव से ठंडा पानी लाता है। यह विश्लेषण केवल मध्य अक्षांशों पर लागू होता है। ध्रुवों के करीब विपरीत भंवर बनते हैं जो पूर्वी तटों को अपेक्षा से अधिक ठंडा बनाते हैं। (अलास्का की धारा की तुलना लैब्राडोर से करें, जो कनाडा के पूर्वी तट से दूर है।)

अतः कथन 2 सही है।


59.जलवायु-अनुकूल कृषि (क्लाइमेट-स्मार्ट एग्रीकल्चर) के लिए भारत की तैयारी के संदर्भ में, निम्नलिखित कथनों म पर विचार कीजिए :

  1. भारत में 'जलवायु-स्मार्ट ग्राम (क्लाइमेट-स्मार्ट विलेज) दृष्टिकोण, अंतर्राष्ट्रीय अनुसंधान कार्यक्रम-जलवायु परिवर्तन, कृषि एवं खाद्य सुरक्षा (सी० सी० ए० एफ० एस०) द्वारा संचालित परियोजना का एक भाग है।
  2. सी० सी० ए० एफ० एस० परियोजना, अंतर्राष्ट्रीय कृषि अनुसंधान हेतु परामर्शदात्री समूह (सी० जी० आइ० ए० आर०) के अधीन संचालित किया जाता है, जिसका मुख्यालय फ्रांस में है।
  3. भारत में स्थित अंतर्राष्ट्रीय अर्धशुष्क उष्णकटिबंधीय फसल अनुसंधान संस्थान (आइ०सी० आर० आइ० एस० ए० टी०), सी० जी० आइ० ए० आर० के अनुसंधान केन्द्रों में से एक है।

उपर्युक्त कथनों में से कौन-से सही हैं?

  1. केवल 1 और 2
  2. केवल 2 और 3
  3. केवल 1 और 3
  4. 1, 2 और 3

उत्तर (d). भारत में सीसीएएफएस की प्रमुख गतिविधियों में विभिन्न कृषि-पारिस्थितिक क्षेत्रों एवं कृषि प्रकारों के लिए जलवायु-स्मार्ट हस्तक्षेपों के पोर्टफोलियो का परीक्षण, मूल्यांकन एवं विकास, क्लाइमेट-स्मार्ट विलेज (सीएसवी) दृष्टिकोण के माध्यम से सीएसए को बढ़ावा देना, मौसम आधारित बीमा, जलवायु सूचना आधारित कृषि परामर्श के प्रसार के लिए आईसीटी का उपयोग, जर्मप्लाज्म संग्रह एवं संरक्षण के हॉटस्पॉट का मानचित्रण करना, तनाव प्रतिरोधी किस्मों के लिए क्राउडसोर्सिंग किसानों की प्राथमिकताएं, जलवायु परिवर्तन अनुकूलन में लिंग एवं सामाजिक समावेशन तथा अनुकूलन एवं शमन गतिविधियों में योजना एवं निवेश के लिए निर्णय-समर्थन उपकरणों का विकास शामिल है। सीसीएएफएस एवं सीजीआईएआर केंद्र भारत में जलवायु-स्मार्ट कृषि को बढ़ावा देने के लिए नीति निर्माताओं, गैर सरकारी संगठनों एवं नागरिक समाज समूहों, अनुसंधान संगठनों, किसान समूहों एवं निजी क्षेत्र सहित राष्ट्रीय एवं उप-राष्ट्रीय हितधारकों के साथ जुड़ रहे हैं। प्राथमिकता वाले क्षेत्रों में उत्तरी भारत की भारत-गंगा योजना एवं भारत के दक्षिणी एवं पश्चिमी भागों में शुष्क भूमि कृषि प्रणाली शामिल हैं।

सीजीआईएआर एक वैश्विक साझेदारी है जो खाद्य सुरक्षा के बारे में अनुसंधान में लगे अंतरराष्ट्रीय संगठनों को एकजुट करती है। इसका मुख्यालय मोंटपेलियर, फ्रांस में है।

आईसीआरआईएसएटी, एक सीजीआईएआर अनुसंधान केंद्र, एक गैर-लाभकारी, गैर-राजनीतिक सार्वजनिक अंतर्राष्ट्रीय अनुसंधान संगठन है जो एशिया एवं उप-सहारा अफ्रीका में विकास के लिए कृषि अनुसंधान करता है, जिसमें दुनिया भर में भागीदारों की एक विस्तृत श्रृंखला है, जिसका मुख्यालय कई क्षेत्रीय कार्यालयों के साथ पाटनचेरु (तेलंगाना) भारत में है।

अतः तीनों कथन सही हैं।


60.“पत्ती-कूड़ा (लीफ़ लिटर) किसी अन्य जीवोम (बायोम) की तुलना में तेजी से विघटित होता है और इसके परिणामस्वरूप मिट्टी की सतह प्रायः अनावृत होती है। पेड़ों के अतिरिक्त, वन में विविध प्रकार के पौधे होते हैं जो आरोहण के द्वारा या अधिपादप (एपिफाइट) के रूप में पनपकर पेड़ों के शीर्ष तक पहुँचकर प्रतिस्थ होते हैं और पेड़ों की ऊपरी शाखाओं में जड़ें जमाते हैं।" यह किसका सबसे अधिक सटीक विवरण है?

  1. शंकुधारी वन
  2. शुष्क पर्णपाती वन
  3. गरान (मैंग्रोव) वन
  4. उष्णकटिबंधीय वर्षावन

उत्तर (d). स्थलीय पारिस्थितिक तंत्र में पत्ती कूड़े के अपघटन की पर्यावरण में तत्वों के जैव-भू-रासायनिक चक्रण में एक प्रमुख भूमिका होती है। तापमान, वर्षा, आर्द्रता एवं मौसमी बदलाव जैसी जलवायु संबंधी विशेषताएं कूड़े के सड़ने की दर को प्रभावित करती हैं।

उष्णकटिबंधीय वर्षावन एक गर्म, नम जीवोम है जहां वर्ष भर वर्षा होती है। यह वनस्पति की घनी छतरियों के लिए जाना जाता है जो तीन अलग-अलग परतों का निर्माण करती हैं। शीर्ष परत या छत्र में विशाल पेड़ होते हैं जो 75 मीटर (लगभग 250 फीट) या उससे अधिक की ऊंचाई तक बढ़ते हैं। वनस्पति की यह परत सूर्य के अधिकांश भाग को जमीन तक पहुंचने से रोकती है। छत्र में मोटी, लकड़ी की बेलें भी पाई जाती हैं। वे धूप तक पहुँचने के लिए छत्रछाया में पेड़ों पर चढ़ती हैं। बीच की परत, लताओं, छोटे पेड़ों, फर्न एवं ताड़ से बनी होती है।

गर्म, आर्द्र जलवायु से चट्टानों का तेजी से रासायनिक अपक्षय होता है। इसके अलावा, जलवायु पत्ती कूड़े के तेजी से अपघटन के लिए आदर्श स्थिति प्रदान करती है। हालांकि, मिट्टी भंगुर होती है और पोषक तत्वों के लिए पत्ती कूड़े के तेजी से अपघटन पर निर्भर करती है। तेजी से अपघटन एक पतली धरण परत का निर्माण करता है। यदि वनस्पति हटा दी जाती है तो पोषक तत्व जल्दी से निकल जाते हैं और मिट्टी तेजी से उर्वरता खो देती है।


61.सवाना की वनस्पति में बिखरे हुए छोटे वृक्षों के साथ घास के मैदान होते हैं, किन्तु विस्तृत क्षेत्र में कोई वृक्ष नहीं होते हैं। ऐसे क्षेत्रों में वन विकास सामान्यतः एक या एकाधिक या कुछ परिस्थितियों के संयोजन के द्वारा नियंत्रित होता है। ऐसी परिस्थितियाँ निम्नलिखित में से कौन-सी हैं?

  1. बिलकारी प्राणी और दीमक
  2. अग्नि
  3. चरने वाले तृणभक्षी प्राणी (हर्बिवोर्स)
  4. मौसमी वर्षा
  5. मृदा के गुण

नीचे दिए गए कूट का प्रयोग कर सही उत्तर चुनिए।

  1. 1 और 2
  2. 4 और 5
  3. 2, 3 और 4
  4. 1, 3 और 5

उत्तर (c). सवाना को वनस्पति संरचना के आधार पर परिभाषित किया गया है, केंद्रीय विचार एक निरंतर घास के निचले हिस्से में एक असंतत वृक्ष का आवरण है। इस क्षेत्र में दुर्लभ वन विकास के पीछे आग, चरने वाले शाकाहारी एवं मौसमी वर्षा हैं।

सवाना वनस्पति में वृक्ष घटक आमतौर पर अधिक महत्वपूर्ण हो जाता है क्योंकि वर्षा बढ़ती है, लेकिन अन्य कारक जैसे स्थलाकृति, मिट्टी एवं चराई की तीव्रता सभी जटिल एवं परिवर्तनशील तरीकों से प्रभाव डालती हैं। शुष्क मौसम की आग, सूखी घास, कुछ पेड़ों, विशेष रूप से अधिक कमजोर युवा पौधों को जला सकती है, एवं इसलिए, उनकी गंभीरता भी सवाना वनस्पति की प्रकृति को बहुत प्रभावित करती है। अतः ‘आग’ सही है, एवं 2 उत्तर में होना चाहिए।

अतः विकल्प (b) एवं (d) खारिज किए जा सकते हैं।

स्पष्ट रूप से 3 एवं 4 महत्वपूर्ण हैं।

शुष्क मौसम में, सवाना में बहुत कम बारिश होती है - लगभग 4 इंच (100 मिमी) - एवं अक्सर कई महीनों तक बिल्कुल भी बारिश नहीं होती है। पौधों के लिए पानी के बिना रहना के लिए यह लंबा समय है। लेकिन नम मौसम में बहुत बारिश होती है।

(मृत कार्बनिक पदार्थों का एक असामान्य रूप से बड़ा अनुपात - लगभग 30 प्रतिशत - दीमक प्रस्फुटन गतिविधियों के माध्यम से विघटित होता है। इस प्रकार, जारी खनिज पोषक तत्वों का एक महत्वपूर्ण अनुपात दीमक के टीले में लंबी अवधि के लिए संग्रहीत किया जा सकता है जहां वे पौधों की जड़ों के लिए आसानी से उपलब्ध नहीं होते हैं। थाईलैंड में सवाना में यह पाया गया है कि मिट्टी की उर्वरता को यांत्रिक रूप से दीमक के टीले को तोड़कर एवं मिट्टी की सतह पर सामग्री को फैलाने से सुधार किया जा सकता है। केन्या में, पुराने दीमक के टीले, जो सामान्य मिट्टी की सतह से ऊपर उठ जाते हैं, अक्सर बाढ़-प्रूफ साइट भी प्रदान करते हैं जहां बीच में घास के मैदानों के साथ, पेड़ एवं झाड़ियां उग सकती हैं, जिन्हें तथाकथित दीमक सवाना कहते हैं।)


62.पृथ्वी ग्रह पर जल के संदर्भ में, निम्नलिखित कथनों पर asma विचार कीजिए: 1. नदियों और झीलों में जल की मात्रा, भू-जल की bagagimoots मात्रा से अधिक है। 2. ध्रुवीय हिमच्छद और हिमनदों में जल की मात्रा, भू-जल की मात्रा से अधिक है।

  1. नदियों और झीलों में जल की मात्रा, भू-जल की bagagimoots मात्रा से अधिक है।
  2. ध्रुवीय हिमच्छद और हिमनदों में जल की मात्रा, भू-जल की मात्रा से अधिक है।

उपर्युक्त कथनों में से कौन-सा/कौन-से सही है/हैं?

  1. केवल 1
  2. केवल 2
  3. 1 और 2 दोनों
  4. न तो 1 और न ही 2

उत्तर (b).


नीचे दी गई तालिका से स्पष्ट है कि केवल कथन 2 सही है।


63.निम्नलिखित कथनों पर विचार कीजिए:

  1. मोरिंगा (सहजन वृक्ष) एक फलीदार सदापर्णी वृक्ष
  2. इमली का पेड़ दक्षिण एशिया का स्थानिक वृक्ष
  3. भारत में अधिकांश इमली लघु वनोत्पाद के रूप में संगृहीत की जाती है।
  4. भारत इमली और मोरिंगा के बीज निर्यात करता
  5. मोरिंगा और इमली के बीजों का उपयोग जैव ईंधन के उत्पादन में किया जा सकता है।

उपर्युक्त कथनों में से कौन-से सही हैं?

  1. 1, 2, 4 और 5
  2. 3, 4 और 5
  3. 1, 3 और 4
  4. 1, 2, 3 और 5

उत्तर (b). मोरिंगा ओलीफेरा (सहजन वृक्ष) मोरिंगा ट्री परिवार मोरिंगासी की सबसे प्रसिद्ध प्रजाति है। यह तेजी से बढ़ने वाला, सूखा प्रतिरोधी वृक्ष है, जो उत्तर भारत, पाकिस्तान, बांग्लादेश एवं अफगानिस्तान के उप-हिमालयी क्षेत्र की मूल वनस्पति है। इसकी उत्पत्ति भारत के उत्तर-पश्चिम में आगरा एवं अवध में मानी जाती है। पेड़ फाइटोस्टेरॉल जैसे स्टिग्मास्टरोल, सिटोस्टेरॉल एवं कैंपेस्टरोल से भरपूर होता है जो हार्मोन के पूर्ववर्ती होते हैं। यह एक गैर फलीदार वृक्ष है। अतः, 1 सही नहीं है।

(ऐसी फसलें जिनकी जड़ों में गांठें होती हैं जिनमें नाइट्रोजन स्थिर करने वाले जीवाणु होते हैं, फलीदार फसले कहलाती हैं।)

यदि 1 सही नहीं है, तो केवल (b) ही हमारा उत्तर हो सकता है!

इमली, (इमली इंडिका), मटर परिवार (फैबेसी) का सदाबहार पेड़, जो उष्णकटिबंधीय अफ्रीका का मूल निवासी है। अतः कथन 2 भी गलत है।


64.भारत में काली कपास मृदा की रचना, निम्नलिखित में से किसके अपक्षयण से हुई है?

  1. भूरी वन मृदा
  2. (विदरी (फिशर) ज्वालामुखीय चट्टान
  3. ग्रेनाइट और शिस्ट
  4. शेल और चूना-पत्थर

उत्तर (b). भारत की इन-सीटू मिट्टी में लावा से ढके क्षेत्रों में पाई जाने वाली काली मिट्टी सबसे अधिक दिखाई देती है। उन्हें अक्सर ‘काली कपास मिट्टी’ के रूप में जाना जाता है, क्योंकि कपास इस मिट्टी के पाए जाने वाले क्षेत्रों की सबसे आम पारंपरिक फसल रही है।

काली मिट्टी ट्रैप लावा के व्युत्पन्न हैं एवं ज्यादातर आंतरिक गुजरात, महाराष्ट्र, कर्नाटक एवं मध्य प्रदेश में दक्कन लावा पठार एवं मालवा पठार पर फैली हुई हैं, जहाँ मध्यम वर्षा एवं अंतर्निहित बेसाल्टिक चट्टान दोनों हैं।

काली मिट्टी आग्नेय बेसाल्टिक चट्टानों - एक प्रकार की बहिर्मुखी चट्टान के अपक्षय का परिणाम है। काली मिट्टी आमतौर पर 6.6 करोड़ साल पहले बने दक्कन ट्रेप में पाई जाती है। यह आमतौर पर 2000 मीटर गहराई में होता है।


65.'पुनःसंयोजित (रीकॉम्बिनेंट) वेक्टर वैक्सीन' से संबंधित हाल के विकास के संदर्भ में, निम्नलिखित कथनों पर विचार कीजिए:

  1. केवल 1
  2. केवल 2
  3. 1 और 2 दोनों
  4. न तो 1 और न ही 2

उत्तर (c). जेनेटिक इंजीनियरिंग व्यापक शब्द है जो उस प्रक्रिया को संदर्भित करता है जिसका उपयोग किसी जीव की आनुवंशिक संरचना में हेरफेर करने के लिए किया जाता है। रिकॉम्बिनेंट तकनीक जेनेटिक इंजीनियरिंग की विधियों में से एक है। तो स्पष्ट रूप से, 1 सही है। तो विकल्प (b) एवं (d) गलत हैं।

हालांकि रीकॉम्बिनेंट वेक्टर टीके मुख्य रूप से वायरस का उपयोग करके विकसित किए जाते हैं, उन्हें बैक्टीरिया का उपयोग करके भी विकसित किया जा सकता है। अतः 2 भी सही है।


66.आनुवंशिक रोगों के संदर्भ में, निम्नलिखित कथनों पर - विचार कीजिए:

  1. अंडों के अंतःपात्र (इन विट्रो) निषेचन से या तो पहले या बाद में सूत्रकणिका प्रतिस्थापन (माइटोकॉन्ड्रिअल रिप्लेसमेंट) चिकित्सा द्वारा सूत्रकणिका रोगों (माइटोकॉन्ड्रिअल डिजीज) को माता-पिता से संतान में जाने से रोका जा सकता
  2. किसी संतान में सूत्रकणिका रोग आनुवंशिक रूप से पूर्णतः माता से जाता है न कि पिता से।

उपर्युक्त कथनों में से कौन-सा/कौन-से सही है/हैं?

  1. केवल 1
  2. केवल 2
  3. 1 और 2 दोनों
  4. न तो 1 और न ही 2

उत्तर (a). सूत्रकणिका (माइटोकॉन्ड्रियल) रोग क्रोनिक (दीर्घकालिक), आनुवंशिक, अक्सर विरासत में मिले विकार होते हैं जो तब होते हैं जब माइटोकॉन्ड्रिया शरीर को ठीक से काम करने के लिए पर्याप्त ऊर्जा का उत्पादन करने में विफल रहता है। (वंशानुगत का अर्थ है कि विकार माता-पिता से बच्चों में पारित हुआ है।) माइटोकॉन्ड्रियल रोग जन्म के समय मौजूद हो सकते हैं, लेकिन किसी भी उम्र में भी हो सकते हैं।

प्राथमिक सूत्रकणिका (माइटोकॉन्ड्रियल) रोग एक अनुवांशिक स्थिति है जिसे कई तरीकों से विरासत में प्राप्त किया जा सकता है (माता-पिता से उनके बच्चों को पारित किया जा सकता है)। सामान्य परिस्थितियों में, किसी बच्चे को जीन जोड़े में विरासत में मिलते हैं - एक जीन माँ से एवं एक पिता से। सूत्रकणिका (माइटोकॉन्ड्रियल) रोग वाले बच्चे को माता-पिता से जीन की एक सामान्य जोड़ी प्राप्त नहीं (किंतु उत्परिवर्तित जीन) होती है।

वंशानुक्रम कई प्रकार के होते हैं - (i) ऑटोसोमल रिसेसिव इनहेरिटेंस - इसमें बच्चे को माता-पिता प्रत्येक से एक जीन की उत्परिवर्तित प्रति प्राप्त होती है। 25% संभावना है कि परिवार के प्रत्येक बच्चे को माइटोकॉन्ड्रियल बीमारी विरासत में मिलेगी, (ii) ऑटोसोमल डोमिनेंट इनहेरिटेंस - इसमें बच्चे को किसी भी पालक या तो माता या पिता से जीन की एक उत्परिवर्तित प्रति प्राप्त होती है। 50% संभावना है कि परिवार के प्रत्येक बच्चे को माइटोकॉन्ड्रियल बीमारी विरासत में मिलेगी, (iii) माइटोकॉन्ड्रियल इनहेरिटेंस - इस अद्वितीय प्रकार की विरासत में, माइटोकॉन्ड्रिया का अपना स्वयं का डीएनए होता है। केवल माइटोकॉन्ड्रियल डीएनए में उत्परिवर्तन के कारण होने वाले माइटोकॉन्ड्रियल विकार विशेष रूप से माताओं से विरासत में मिलते हैं। यदि इस तरह से माइटोकॉन्ड्रियल बीमारी विरासत में मिली है, तो 100% संभावना है कि परिवार के प्रत्येक बच्चे को माइटोकॉन्ड्रियल बीमारी विरासत में मिलेगी एवं (iv) यादृच्छिक उत्परिवर्तन (रेंडम म्युटेशन) - कभी-कभी जीन स्वयं का एक उत्परिवर्तन विकसित करते हैं जो माता-पिता से विरासत में नहीं मिलता है।

इस प्रकार यह केवल ‘माइटोकॉन्ड्रियल वंशानुक्रम’ में है कि रोग माता से आता है, पिता से नहीं। अतः 2 सही नहीं है इसलिए (b) एवं (c) को खारिज किया जा सकता है।

अंतःपात्र (इन विट्रो) निषेचन के बाद एवं पहले सूत्रकणिका प्रतिस्थापन संभव है।

प्रोन्यूक्लियर ट्रांसफर तकनीक (पीटीटी) - यह निषेचन के बाद प्रशासित एमआरटी का एक महत्वपूर्ण दृष्टिकोण है, जिसमें दो युग्मज इन विट्रो में पाले जाते हैं। एक जाइगोट्स जैविक माता-पिता से संबंधित होता है जिसमें प्रोन्यूक्लियी एवं दोषपूर्ण माइटोकॉन्ड्रिया हैं एवं दूसरा प्रोन्यूक्लि एवं स्वस्थ माइटोकॉन्ड्रिया वाले दाता का है

मैटरनल स्पांइडल ट्रांसफर (एमएसटी) तकनीक - निषेचन से पहले निष्पादित तकनीक, प्रसवपूर्व निदान एवं पूर्व-प्रत्यारोपण आनुवंशिक निदान (पीजीडी) के समान चयनात्मक प्रजनन का एक रूप है।

अतः 1 सही है एवं सही उत्तर (a) है।


67.बॉलगार्ड-I और बॉलगार्ड-II प्रौद्योगिकियों का उल्लेख किसके संदर्भ में किया जाता है?

  1. फसली पादपों का क्लोनी प्रवर्धन
  2. आनुवंशिक रूप से रूपांतरित फसली पादपों का विकास
  3. पादप वृद्धिकर पदार्थों का उत्पादन
  4. जैव उर्वरकों का उत्पादन

उत्तर (b). बोलगार्ड I, मुख्य रूप से उगाई जाने वाली, कीट सहिष्णु कपास की फसल, जिसमें बैसिलस थुरिंगिनेसिस से केवल एक जीन, क्राई 1Ac होता है।

बोलगार्ड II में क्राई 1Ac के अलावा क्राई 2Ab जीन होता है। मायहको-मोनसेंटो बायोटेक लिमीटेड (एमएमबीएल) द्वारा विकसित बोलगार्ड II इवेंट को तकनीकी रूप से एमओएन 15985 के रूप में नामित किया गया है।


68.किसी प्रेशर कुकर में, जिस तापमान पर खाद्य पकाए जाते हैं, वह मुख्यतः निम्नलिखित में से किन पर निर्भर करता

  1. ढक्कन में स्थित छिद्र का क्षेत्रफल
  2. ज्वाला का तापमान
  3. ढक्कन का भार

नीचे दिए गए कूट का प्रयोग कर सही उत्तर चुनिए।

  1. केवल 1 और 2
  2. केवल 2 और 3
  3. केवल 1 और 3
  4. 1, 2 और 3

उत्तर (c). प्रेशर कुकिंग का सरल तर्क यह है कि यदि आप एक बंद जगह में दबाव बढ़ाते हैं, तो उस जगह में मौजूद तरल का क्वथनांक भी बढ़ जाएगा। 1 एटमोसफीयर पर पानी का क्वथनांक 100° सेल्सियस होता है। यदि दबाव बढ़ता है, तो पानी का क्वथनांक भी बढ़ जाता है। यह अधिक गर्मी वहन करता है, और चीजों को तेजी से पकाता है।

प्रेशर कुकर में एक संशोधित ढक्कन होता है जिसमे सील करने के लिए रबर गैसकेट लगा होता है। कुकर उबलते पानी का तापमान बढ़ाकर अपना कार्य करता है। प्रेशर कुकर का उपयोग करने के लिए, हम बर्तन में कुछ तरल के साथ खाना डालते हैं। एक बार जब ढक्कन बंद हो जाता है और कुकर तेज आंच पर सेट हो जाता है, तो बर्तन में भाप बन जाती है जो बाहर नही निकल सकती। फंसी हुई भाप कुकर के अंदर के वायुमंडलीय दबाव को सामान्य समुद्र-स्तर के दबाव से ऊपर बढ़ा देती है। उस दाब पर पानी का क्वथनांक बढ़ जाता है। यह उच्च तापमान वह है जो भोजन को तेजी से पकाता है। आमतौर पर ढक्कन पर एक गेज या पॉप-अप रॉड लगा होता है, एक बार जब कुकर पूर्ण दबाव पर पहुंच जाता है, तो यह रिलीज वाल्व खुलता है, जो बर्तन के अंदर एक स्थिर तापमान बनाए रखने के लिए एक विनियमित प्रवाह में भाप को बाहर निकालता रहता है।

ढक्कन का वजन जितना अधिक होगा, कुकर उतना ही अधिक दबाव और तापमान बनाए रख सकता है। ढक्कन में छेद का क्षेत्र उपयुक्त होना चाहिए, यदि यह बड़ा है तो उचित दबाव और तापमान बनाए नहीं रखा जा सकेगा।

लेकिन लौ का तापमान मायने नहीं रखेगा, क्योंकि यह खाना पकाने में लगने वाला ‘समय’ तय करेगा, और कुछ नहीं। एक उच्च तापमान भोजन को तेजी से पकाएगा, और इसके विपरीत। यह कुकर के अंदर के दबाव को प्रभावित नहीं करेगा क्योंकि कम तापमान की लौ, पर्याप्त समय दिए जाने पर, वैसे भी आवश्यक दबाव बनाएगी।


69.निम्नलिखित पर विचार कीजिए:

  1. जीवाणु
  2. कवक
  3. विषाणु

उपर्युक्त में से किन्हें कृत्रिम/संश्लेषित माध्यम में संवा किया जा सकता है?

  1. केवल 1 और 2
  2. केवल 2 और 3
  3. केवल 1 और 3
  4. 1, 2 और 3

उत्तर (a). विषाणु जानवरों, पौधों एवं यहां तक कि अन्य सूक्ष्मजीवों को भी संक्रमित कर सकते हैं। चूंकि विषाणुओं में स्वयं की उपापचयी मशीनरी का अभाव होता है एवं वे प्रतिकृति के लिए पूरी तरह से अपने मेजबान सेल पर निर्भर होते हैं, इसलिए उन्हें सिंथेटिक कल्चर मीडिया में नहीं उगाया जा सकता है। अतः उत्तर विकल्प (a) है।


70.निम्नलिखित कथनों पर विचार कीजिए:

  1. एडीनोवायरसों में एकल-तंतु डी० एन० संजीन (जीनोम) होते हैं, जबकि रेट्रोवायरसों द्वि-तंतु डी० एन० ए० संजीन (जीनोम) होते हैं
  2. कभी-कभी सामान्य जुकाम एडीनोवायरस के का होता है, जबकि एड्स (ए० आइ० डी० एस रेट्रोवायरस के कारण होता है।

उपर्युक्त कथनों में से कौन-सा/कौन-से सही है/हैं?

  1. केवल 1
  2. केवल 2
  3. 1 और 2 दोनों
  4. न तो 1 और न ही 2

उत्तर (b). एडेनोवायरस मध्यम आकार के, गैर-आवरणीय वायरस होते हैं जिनमें द्वितंतु डीएनए संजीन वाले इकोसाहेड्रल न्यूक्लियोकैप्सिड होते हैं, जबकि रेट्रोवायरस एकल-तंतु हुए आरएनए पशु वायरस होते हैं जो डबल-फंसे डीएनए मध्यवर्ती को नियोजित करते हैं।

एक एडेनोवायरस को गैर-आवरणीय वायरसों में सबसे बड़ा माना जाता है। यह शब्द इंगित करता है कि वायरस के अंदर संग्रहीत आनुवंशिक जानकारी को लपेटने के लिए वायरस के पास कोई सुरक्षात्मक प्रोटीन-आवरण नहीं है, जिसे कैप्सिड कहा जाता है। इस वायरस में एक द्वितुतु डीएनए होता है। यह साबित हो चुका है कि यह वायरस बच्चों एवं वयस्कों में लगभग 10% ऊपरी श्वसन संक्रमण का कारण है। एडेनोवायरस आम वायरस हैं जो कई तरह की बीमारी का कारण बनते हैं। वे सर्दी जैसे लक्षण, बुखार, गले में खराश, ब्रोंकाइटिस, निमोनिया, दस्त, एवं गुलाबी आंख (नेत्रश्लेषमलाशोथ) पैदा कर सकते हैं।

दूसरी ओर, एक रेट्रोवायरस एक आच्छादित वायरस का एक उदाहरण है। इस मामले में, इसमें एक सुरक्षात्मक प्रोटीन कोट होता है जो इसे अधिक लचीला बनाता है एवं इसमें बीमारियों का कारण बनने की प्रवृत्ति अधिक होती है। इसे एक आरएनए वायरस के रूप में माना जाता है एवं यह स्वयं को अपने मेजबान कोशिकाओं में एकीकृत कर सकता है, इस प्रकार, कोशिका को नुकसान पहुंचा सकता है। एड्स रोग एचआईवी नामक रेट्रोवायरस के कारण होता है।

अतः कथन 1 गलत है एवं कथन 2 सही है।


71.जल किसी अन्य द्रव की अपेक्षा अधिक पदार्थों को घोल सकता है, क्योंकि

  1. इसकी प्रकृति द्विध्रुवीय है
  2. यह ऊष्मा का सुचालक है
  3. इसकी विशिष्ट ऊष्मा का मान उच्च होता है
  4. यह हाइड्रोजन का एक ऑक्साइड है

उत्तर (a). जल को ‘सार्वभौमिक विलायक’ कहा जाता है क्योंकि यह किसी भी अन्य तरल की तुलना में अधिक पदार्थों को घोलने में सक्षम है। जल की रासायनिक संरचना एवं भौतिक गुण इसे इतना उत्कृष्ट विलायक बनाते हैं।

जल के अणुओं में ऑक्सीजन एवं हाइड्रोजन परमाणुओं की ध्रुवीय व्यवस्था होती है- एक तरफ (हाइड्रोजन) में सकारात्मक विद्युत चार्ज होता है एवं दूसरी तरफ (ऑक्सीजन) में नकारात्मक चार्ज होता है।

जल के इस गुण को उसके अणु की द्विध्रुवीय प्रकृति कहा जाता है।

यह प्रकृति जल के अणु को कई अन्य विभिन्न प्रकार के अणुओं की ओर आकर्षित करने में सक्षम बनाती है।


72.सड़क प्रकाश व्यवस्था के संदर्भ में, सोडियम बत्तियाँ, एल० ई० डी० बत्तियों से किस तरह भिन्न हैं?

  1. सोडियम बत्तियाँ प्रकाश को 360 डिग्री में उत्पन्न करती हैं, किन्तु एल० ई० डी० बत्तियों में ऐसा नहीं होता है।
  2. सड़क की बत्तियों के रूप में, एल० ई० डी० बत्तियों की तुलना में सोडियम बत्तियों की उपयोगिता अवधि अधिक होती है।
  3. सोडियम बत्ती के दृश्य प्रकाश का स्पेक्ट्रम लगभग एकवर्णी होता है, जबकि एल० ई० डी० बत्तियाँ सड़क प्रकाश व्यवस्था में सार्थक वर्ण सुविधाएँ (कलर अडवॅटेज) प्रदान करती हैं।

नीचे दिए गए कूट का प्रयोग कर सही उत्तर चुनिए।

  1. केवल 3
  2. केवल 2
  3. केवल 1 और 3
  4. 1, 2 और 3

उत्तर (c). एलईडी लैंप दिशात्मक प्रकाश (180 डिग्री से अधिक) प्रदान करते हैं, इसके विपरित सोडियम बत्तियाँ प्रकाश को 360 डिग्री में उत्पन्न करती हैं। अतः कथन 1 सत्य है। उत्तर (c) या (d) हो सकता है।

सड़क की बत्तियों के रूप में एलईडी लैंप का जीवनकाल अन्य स्ट्रीट लाइट प्रकारों (50,000 से 100,000 घंटे) की तुलना में सबसे लंबा होता है। अतः कथन 2 सत्य नहीं है। उत्तर विकल्प (c) है।

कलर रेंडरिंग इंडेक्स (सीआरआई) - सीआरआई प्रकाश स्रोत द्वारा वस्तु के रंग, आकार एवं प्रकार को दर्शाने की गुणवत्ता का मापक है। सड़क की बत्तियों के लिए मानदंड - 75 से 100 - उत्कृष्ट, 65 से 75 - अच्छा, 55 से 65 - ठीक, 0 से 55 - खराब। एलईडी स्ट्रीट लाइट में 65 से 95 का सीआरअई होता है जबकि सोडियम लैंप में सीआरआई 20 से 30 (लगभग एकवर्णी) होता है। अतः कथन 3 सही है।


73.'ACE2' पद का उल्लेख किस संदर्भ में किया जाता है?

  1. आनुवंशिक रूप से रूपांतरित पादपों में पुरःस्थापित (इंट्रोड्यूस्ड) जीन
  2. भारत के निजी उपग्रह संचालन प्रणाली का विकास
  3. वन्य प्राणियों पर निगाह रखने के लिए रेडियो कॉलर
  4. विषाणुजनित रोगों का प्रसार

उत्तर (d). एसीई2 का अर्थ एंजियोटेंसिन-परिवर्तित एंजाइम 2 है। यह कोविड -19 के कारण लगातार खबरों में रहा है।

एसीई2 कई प्रकार की कोशिकाओं की सतह पर पाया जाने वाला प्रोटीन है। यह एक एंजाइम है जो बड़े प्रोटीन एंजियोटेंसिनोजेन को काटकर छोटे प्रोटीन उत्पन्न करता है - जो तब कोशिका में कार्यों को विनियमित करते है। अपनी सतह पर स्पाइक जैसे प्रोटीन का उपयोग करते हुए, कोशिकाओं के प्रवेश एवं संक्रमण से पहले, सार्स-सीओवी-2 वायरस एसीई2 से ठीक वैसे ही बंध जाता है जैसे किसी चाबी को ताले में डाला जाता है। इस प्रकार, एसीई2 कोविड-19 कारक वायरस के लिए एक सेलुलर द्वार - रिसेप्टर - की तरह कार्य करता है।


74.बिस्फिनॉल A (BPA), जो चिन्ता का कारण है, निम्नलिखित में से किस प्रकार के प्लास्टिक के उत्पादन में एक संरचनात्मक/मुख्य घटक है?

  1. निम्न घनत्व वाले पॉलिएथिलीन
  2. पॉलिकार्बोनेट
  3. पॉलिएथिलीन टेरेफ्थेलेट
  4. पॉलिविनाइल क्लोराइड

उत्तर (b). बिस्फिनॉल ए (बीपीए) मुख्य रूप से पॉली कार्बोनेट प्लास्टिक एवं एपॉक्सी रेजिन के उत्पादन में उपयोग के लिए बड़ी मात्रा में उत्पादित एक रसायन है।

भ्रुण, शिशुओं एवं बच्चों के मस्तिष्क एवं प्रोस्टेट ग्रंथि पर संभावित स्वास्थ्य प्रभावों के कारण बीपीए का एक्सपोजर एक चिंता का विषय है। इसका असर बच्चों के व्यवहार पर भी पड़ सकता है। शोध बीपीए एवं बढ़े हुए रक्तचाप, टाइप 2 मधुमेह एवं हृदय रोग के बीच एक संभावित लिंक का सुझाव देते हैं।


75.निम्नलिखित में से किसमें 'ट्राइक्लोसन' के विद्यमान होने की सर्वाधिक संभावना है, जिसके लंबे समय तक उच्च स्तर के प्रभावन में रहने को हानिकारक माना जाता है?

  1. खाद्य परिरक्षक
  2. फल पकाने वाले पदार्थ
  3. पुनःप्रयुक्त प्लास्टिक के पात्र
  4. प्रसाधन सामग्री

उत्तर (d). ट्राईक्लोसन कई उपभोक्ता उत्पादों में पाया जाता है जिसका उद्देश्य जीवाणु संदूषण को कम करना या रोकना है। यह कुछ जीवाणुरोधी साबुन एवं बॉडी वॉश, टूथपेस्ट, एवं कुछ यू.एस. फूड एंड ड्रग एडमिनिस्ट्रेशन (एफडीए) द्वारा विनियमित सौंदर्य प्रसाधनों में पाया जाता है। यह कपड़ों, बरतन, फर्नीचर एवं खिलौनों अर्थात ऐसे उत्पादों जिन्हें एफडीए द्वारा विनियमित नही किया जाता मैं भी पाया जा सकता है।

कुछ जानवरों के अध्ययन से पता चला है कि ट्राइक्लोसन की उच्च खुराक थायराइड हार्मोन के स्तर में कमी सें जुड़ी है। ऐसे अन्य अध्ययन चल रहे हैं जिनमें ट्राईक्लोसन की स्वास्थ्य सुरक्षा संबंधी जाँच की जा रही है। जानवरों में ट्राइक्लोसन के लंबे समय तक संपर्क के बाद त्वचा कैंसर के विकास की संभावना की जांच करने वाला एक अध्ययन भी जारी है।


76.खगोलीय दूरियाँ प्रकाश-वर्ष में मापे जाने का कारण निम्नलिखित में से कौन-सा है?

  1. तारकीय पिंडों के बीच की दूरियाँ परिवर्तित नहीं होती हैं।
  2. तारकीय पिंडों का गुरुत्व परिवर्तित नहीं होता है।
  3. प्रकाश सदैव सीधी रेखा में यात्रा करता है।
  4. प्रकाश की गति (स्पीड) सदैव एकसमान होती

उत्तर (d). एक बेहतरीन प्रश्न।

प्रकाश वर्ष, खगोलविदों द्वारा अंतरिक्ष में दूरी नापने की ईकाई हैं। यह प्रकाश की किरण द्वारा एक वर्ष में तय की गई दूरी है जो - छह ट्रिलियन मील के बराबर होती है।

प्रकाश वर्ष में खगोलीय दूरियों को मापने के कारण हैं - (a) प्रकाश की गति स्थिर है, एवं (b) इतनी विस्तृत लंबाईयों के लिए यह प्रर्याप्त बड़ी है एवं संक्षिप्त भी है।

तारकीय-पिंडों की गति को प्रकाश-वर्ष में मापने से खगोलविदों को यह निर्धारित करने की भी अनुमति मिलती है कि वे कितने समय पहले देख रहे हैं। चूँकि प्रकाश को हमारी आँखों तक पहुँचने में समय लगता है, रात के आकाश में हम जो कुछ भी देखते हैं वह पहले ही हो चुका होता है। दूसरे शब्दों में, जब आप किसी 1 प्रकाश वर्ष दूर किसी वस्तु को देखते हैं, तो आप उसे ठीक उसी तरह देखते हैं जैसे वह एक वर्ष पहले दिखाई दी थी।


77.हमने ब्रिटिश मॉडल पर आधारित संसदीय लोकतंत्र को अपनाया है, किन्तु हमारा मॉडल उस मॉडल से किस प्रकार भिन्न है?

  1. जहाँ तक विधि-निर्माण का संबंध है, ब्रिटिश संसद सर्वोपरि अथवा संप्रभु है, किन्तु भारत में संसद की विधि-निर्माण की शक्ति परिसीमित है।
  2. भारत में, संसद के किसी अधिनियम के संशोधन की संवैधानिकता से संबंधित मामले उच्चतम न्यायालय द्वारा संविधान पीठ को भेजे जाते हैं।

नीचे दिए गए कूट का प्रयोग कर सही उत्तर चुनिए।

  1. केवल 1
  2. केवल 2
  3. 1 और 2 दोनों
  4. न तो 1 और न ही

उत्तर (c). भारत की संसदीय प्रणाली काफी हद तक ब्रिटिश संसदीय प्रणाली पर आधारित है लेकिन यह कभी भी ब्रिटिश प्रणाली की प्रतिकृति नहीं बनी एवं निम्नलिखित मामलों में भिन्न है -

  1. भारत में ब्रिटिश राजतंत्रीय व्यवस्था के स्थान पर एक गणतांत्रिक व्यवस्था है
  2. ब्रिटिश प्रणाली संसद की संप्रभुता के सिद्धांत पर आधारित है, जबकि संसद भारत में सर्वोच्च नहीं है एवं लिखित संविधान, संघीय प्रणाली, न्यायिक समीक्षा एवं मौलिक अधिकारों के कारण सीमित एवं प्रतिबंधित शक्तियों का आनंद लेती है।

अतः कथन 1 सत्य है।

कोई संविधान पीठ सर्वोच्च न्यायालय की एक पीठ है जिसमें पांच या अधिक न्यायाधीश होते हैं। ये कोई नियमित घटना नहीं हैं। सर्वोच्च न्यायालय के समक्ष अधिकांश मामलों की सुनवाई एवं निर्णय दो न्यायाधीशों (जिन्हें डिवीजन बेंच कहा जाता है) एवं कभी-कभी तीन की बेंच द्वारा किया जाता है। संविधान पीठ अपवाद हैं, केवल तभी स्थापित की जाती हैं जब निम्नलिखित में से एक या अधिक परिस्थितियां मौजूद हों -

  1. मामले में संविधान की व्याख्या से संबंधित कानून का एक महत्वपूर्ण प्रश्न शामिल है,
  2. भारत के राष्ट्रपति ने संविधान के अनुच्छेद 143 के तहत तथ्य या कानून के प्रश्न पर सर्वोच्च न्यायालय की राय मांगी है,
  3. सुप्रीम कोर्ट के दो या दो से अधिक तीन-न्यायाधीशों ने कानून के एक ही बिंदु पर परस्पर विरोधी निर्णय दिए हैं, इस प्रकार एक बड़ी बेंच द्वारा एक निश्चित घोषणा की आवश्यकता है,
  4. बाद में तीन-न्यायाधीशों की बेंच सुप्रीम कोर्ट की पिछली तीन-न्यायाधीशों की बेंच द्वारा दिए गए फैसले की शुद्धता पर संदेह करती है, एवं उस पहले के फैसले पर पुनर्विचार के लिए मामले को एक बड़ी बेंच को संदर्भित करने का फैसला करती है।
  5. अतः कथन 2 सत्य है भारत में कभी-कभी ऐसा होता है (लेकिन यूके में, ऐसा बिल्कुल नहीं हो सकता)।

    अतः उत्तर (c) है।


78.संघ सरकार के संदर्भ में, निम्नलिखित कथनों पर विचार कीजिए:

  1. एन० गोपालास्वामी आयंगर समिति ने सुझाव दिया था कि किसी मंत्री और किसी सचिव को प्रशासनिक सुधार करने और उसे बढ़ावा देने के लिए पूर्णतः नामित किया जाना चाहिए।
  2. प्रशासनिक सुधार आयोग, 1966 की संस्तुति के आधार पर वर्ष 1970 में कार्मिक विभाग का गठन किया गया और इसे प्रधानमंत्री के प्रभार के अधीन रखा गया।

उपर्युक्त कथनों में से कौन-सा/कौन-से सही है/हैं?

  1. केवल 1
  2. केवल 2
  3. 1 और 2 दोनों
  4. न तो 1 और न ही 2

उत्तर (d). एन. जी. आयंगर भारतीय संविधान की सात सदस्यीय मसौदा समिति का हिस्सा थे, जिसे अगस्त 29, 1947 को नियुक्त किया गया था। एन. गोपालस्वामी अय्यंगार ने 1950 में ‘केंद्र सरकार की मशीनरी का पुनर्गठन’ नामक अपनी रिपोर्ट में मंत्रालयों के समूहीकरण, कर्मियों की क्षमताओं में सुधार एवं ओ एंड एम डिवीजन के कामकाज में भी सिफारिश की थी।

अतः कथन 1 गलत है।

फरवरी 07, 1973 को प्रशासनिक सुधार विभाग से संबंधित कार्य को कैबिनेट सचिवालय के अंतर्गत सृजित कार्मिक विभाग को स्थानांतरित कर दिया गया एवं अगस्त 01, 1970 को इसे कार्मिक एवं प्रशासनिक सुधार विभाग के रूप में पुर्ननामित किया गया।

(संदर्भ https://darpg.gov.in/about-department-0

अतः कथन 2 सही नहीं है। इसलिए कथन 1 एवं 2 दोनों सही नहीं हैं।


79.भारत के संविधान के किस अनुच्छेद के अंतर 'निजता का अधिकार' संरक्षित है?

  1. अनुच्छेद 15
  2. अनुच्छेद 19
  3. अनुच्छेद 21
  4. अनुच्छेद 29

. उत्तर (c). पुट्टस्वामी मामले में अपने ऐतिहासिक फैसले में, निजता के अधिकार को भारत के सर्वोच्च न्यायालय द्वारा मौलिक अधिकार घोषित किया गया था। यह अधिकार अनुच्छेद 21 के तहत जीवन एवं व्यक्तिगत स्वतंत्रता के अधिकार के आंतरिक हिस्से के रूप में एवं संविधान के भाग III द्वारा गारंटीकृत स्वतंत्रता के एक भाग के रूप में संरक्षित है।

भारत के संविधान का अनुच्छेद 21 - जीवन एवं व्यक्तिगत स्वतंत्रता का संरक्षण प्रदान करता है। इसमें कहा गया है कि ‘कानून द्वारा स्थापित प्रक्रिया के अनुसार किसी भी व्यक्ति को उसके जीवन या व्यक्तिगत स्वतंत्रता से वंचित नहीं किया जाएगा।’ इस प्रकार, अनुच्छेद 21 दो अधिकारों को सुरक्षित करता है - जीवन का अधिकार एवं व्यक्तिगत स्वतंत्रता का अधिकार।

भारत के संविधान का अनुच्छेद 15 केवल धर्म, नस्ल, जाति, लिंग या जन्म स्थान के आधार पर भेदभाव को रोकता है। अतः विकल्प (b) सही नहीं है।

संविधान का अनुच्छेद 19 भाषण की स्वतंत्रता प्रदान करता है जो मौखिक/लिखित/इलेक्ट्रॉनिकध्/प्रसारण/प्रेस के माध्यम से बिना किसी डर के स्वतंत्र रूप से अपनी राय व्यक्त करने का अधिकार है।

भारतीय संविधान का अनुच्छेद 29 अल्पसंख्यकों के हितों के संरक्षण से संबंधित है।


80.निम्नलिखित कथनों पर विचार कीजिए:

  1. भारत में ऐसा कोई कानून नहीं है जो प्रत्याशिय को किसी एक लोक सभा चुनाव में ती निर्वाचन-क्षेत्रों से लड़ने से रोकता है।
  2. 1991 के लोक सभा चुनाव में श्री देवी लाल तीन लोक सभा निर्वाचन-क्षेत्रों से चुनाव लड़ था।
  3. वर्तमान नियमों के अनुसार, यदि कोई प्रत्याश किसी एक लोक सभा चुनाव में कई निर्वाचन क्षेत्रों से चुनाव लड़ता है, तो उसकी पार्टी को उनिर्वाचन-क्षेत्रों के उप-चुनावों का खर्च उठा. चाहिए, जिन्हें उसने खाली किया है बशर्ते व सभी निर्वाचन-क्षेत्रों से विजयी हुआ हो।

उपर्युक्त कथनों में से कौन-सा/कौन-से सही है/हैं?

  1. केवल 1
  2. केवल 2
  3. 1 और 3
  4. 2 और 3

उत्तर (b). भारत में चुनाव जनप्रतिनिधित्व अधिनियम, 1951 (आरपीए) द्वारा शासित होते हैं, एवं भारत के चुनाव आयोग (ईसीआई) द्वारा आयोजित किए जाते हैं।

आरपीए की धारा 33 (7) के अनुसार, एक उम्मीदवार अधिकतम दो निर्वाचन क्षेत्रों से चुनाव लड़ सकता है (अधिक निर्वाचन क्षेत्रों को 1996 तक अनुमति दी गई थी जब आरपीए में दो निर्वाचन क्षेत्रों की सीमा निर्धारित करने के लिए संशोधन किया गया था)।

अतः, कथन 1 गलत है। उत्तर विकल्प (b) या (d) हो सकता है।

चुनाव आयोग ने 2004, 2010, 2016 एवं फिर 2018 में कार्यपालिका एवं सर्वोच्च न्यायालय को दी गई अपनी सिफारिशों में धारा 33 (7) में संशोधन करने की सिफारिश की ताकि एक उम्मीदवार को केवल एक सीट से चुनाव लड़ने की अनुमति मिल सके। (यदि कोई संशोधन नहीं होता है, तो चुनाव आयोग का मत था कि एक ऐसी प्रणाली तैयार की जानी चाहिए जिसमें यदि कोई उम्मीदवार दो निर्वाचन क्षेत्रों से चुनाव लड़ता है एवं दोनों में जीत हासिल करता है, तो वह एक निर्वाचन क्षेत्र में बाद के उप-चुनाव कराने का वित्तीय भार वहन करेगा। यह राशि विधानसभा चुनाव के लिए 5 लाख रुपये एवं लोकसभा चुनाव के लिए 10 लाख रुपये होगी।)

अतः कथन 3 सही नहीं है।

अतः उत्तर (b) होना चाहिए। ‘लेकिन हम इस बात की पुष्टि नहीं कर सके कि श्री देवीलाल ने 1991 के लोकसभा चुनावों में किन 3 सीटों से चुनाव लड़ा था।


81.निम्नलिखित कथनों पर विचार कीजिए:

  1. 'शहर का अधिकार' एक सम्मत मानव अधिकार है तथा इस संबंध में, संयुक्त राष्ट्र हैबिटेट (यू० एन० हैबिटेट) प्रत्येक देश द्वारा की गई प्रतिबद्धताओं को मॉनिटर करता है।
  2. शहर का अधिकार' शहर के प्रत्येक निवासी को शहर में सार्वजनिक स्थानों को वापस लेने (रीक्लेम) एवं सार्वजनिक सहभागिता का अधिकार देता है।
  3. 'शहर का अधिकार' का आशय यह है कि राज्य, For शहर की अनधिकृत बस्तियों को किसी भी लोक सेवा अथवा सुविधा से वंचित नहीं कर सकता।

उपर्युक्त कथनों में से कौन-सा/कौन-से सही है/हैं?

  1. केवल 1
  2. केवल 3
  3. 1 और 2
  4. 2 और 3

उत्तर (c). यह साइट देखें https://www.right2city.org/the-right-to-the-city/

शहर का अधिकार सभी निवासियों, वर्तमान एवं भविष्य, स्थायी एवं अस्थायी, निवास करने, उपयोग करने, आधिपत्य रखने, उत्पादन करने, शासन करने एवं न्यायपूर्ण, समावेशी, सुरक्षित एवं टिकाऊ शहरों, गांवों तथा मानव बस्तियों का आनंद लेने का अधिकार है, जिसे एक पूर्ण एवं सभ्य जीवन के लिए सामान्य आवश्यकता के रूप में परिभाषित किया गया है।

अतः कथन 2 सही है। इसलिए विकल्प (a) एवं (b) को खारिज कर दिया गया है।

शहर का अधिकार एक सामूहिक अधिकार है जो अंतरराष्ट्रीय स्तर पर मान्यता प्राप्त सभी नागरिक, राजनीतिक, आर्थिक, सामाजिक, सांस्कृतिक एवं पर्यावरणीय अधिकारों की क्षेत्रीय अखंडता एवं अन्योन्याश्रयता पर प्रकाश डालता है, जैसा कि अंतरराष्ट्रीय मानवाधिकार संधियों में विनियमित है, उनके क्षेत्रीय आयाम एवं पर्याप्त जीवन मानक पर ध्यान केंद्रित करता है।

अब देखें https://uploads.habitat3.org/hb3/Habitat%20III%20Policy%20Paper%201.pdf

अतः कथन 1 सही प्रतीत होता है। अतः उत्तर (c) है।


82.भारत के संदर्भ में, निम्नलिखित कथनों पर विचार कीजिए:

  1. न्यायिक हिरासत का अर्थ है कि अभियुक्त संबंधित मजिस्ट्रेट की हिरासत में है और ऐसे अभियुक्त को पुलिस स्टेशन के हवालात में रखा जाता है न कि जेल में।
  2. न्यायिक हिरासत के दौरान, मामले के प्रभारी पुलिस अधिकारी, न्यायालय की अनुमति के बिना संदिग्ध व्यक्ति से पूछताछ नहीं कर सकते।

उपर्युक्त कथनों में से कौन-सा/कौन-से सही है/हैं?

  1. केवल 1
  2. केवल 2
  3. 1 और 2 दोनों
  4. न तो 1 और न ही

उत्तर (b). ‘हिरासत’ शब्द का अर्थ है सुरक्षात्मक देखभाल के लिए किसी को पकड़ना।

पुलिस हिरासत (पीसी) का अर्थ है कि पुलिस के पास आरोपी की शारीरिक हिरासत है जबकि न्यायिक हिरासत (जेसी) का अर्थ है कि आरोपी संबंधित मजिस्ट्रेट की हिरासत में है।

पूर्व में, आरोपी को थाने के लॉकअप में बंद रखा जाता है, जबकि बाद में वाले मामलें में जेल में।

न्यायिक हिरासत के दौरान, मामले के प्रभारी पुलिस अधिकारी को संदिग्ध से पूछताछ करने की अनुमति नहीं है। हालांकि, अदालत के सामने पेश किए गए तथ्यों के तहत पूछताछ आवश्यक होने पर अदालत पूछताछ करने की अनुमति दे सकती है।

अतः केवल कथन 2 सही है।


83.भारत के संदर्भ में, निम्नलिखित कथनों पर कीजिए:

  1. जब एक कैदी पर्याप्त आधार प्रस्तुत करता है, तो ऐसे कैदी को पैरोल मना नहीं किया जा सकता क्योंकि वह उसके अधिकार का मामला बन जाता है|
  2. कैदी को पैरोल पर छोड़ने के लिए राज्य सरकारों के अपने नियम हैं।

उपर्युक्त कथनों में से कौन-सा/कौन-से सही है/हैं?

  1. केवल 1
  2. केवल 2
  3. 1 और 2 दोनों
  4. न तो 1 और न ही 2

उत्तर (b). आपराधिक कानून में ‘पैरोल’ एक अपराधी को उसके द्वारा पालन की जाने वाली कुछ शर्तों पर कारावास से रिहा करना है। यह सजा के कुछ हिस्से के पूर्ण होने के बाद जेल से कुछ दिनों के लिए रिहाई है।

‘पैरोल’ देने की शक्ति अनिवार्य रूप से कार्यपालिका की है। कैदी द्वारा पैरोल की अनुमति माँगे जाने पर उसके अनुरोध पर विचार करना केवल कार्यपालिका का विशेषाधिकार है।

प्रत्येक राज्य ने पैरोल के अनुदान को विनियमित करने के अलग नियम बनाए हैं। अतः कथन 2 सत्य है।

पैरोल पर रिहा होने के हकदार होने से पहले प्रत्येक कैदी को कुछ पात्रता मानदंडों को पूरा करना होता है।

इसके अलावा, अत्यंत विशेष परिस्थितीयों के अलावा, निम्नलिखित मामलों में, पैरोल को सामान्यतः तब तक नहीं दिया जाता,

  1. यदि कैदी को बलात्कार के बाद हत्या का दोषी ठहराया जाता है,
  2. यदि कैदी को बच्चों की हत्या एवं बलात्कार के लिए दोषी ठहराया जाता है,
  3. यदि कैदी को कई हत्याओं के लिए दोषी ठहराया जाता है।

अतः कथन 1 सत्य नहीं है।


84.राष्ट्रीय स्तर पर, अनुसूचित जनजाति और अन्य पार वन निवासी (वन अधिकारों की मान्यता) अधिनि 2006 के प्रभावी कार्यान्वयन को सुनिश्चित कर लिए कौन-सा मंत्रालय केन्द्रक अभिकरण (न एजेंसी) है?

  1. पर्यावरण, वन और जलवायु परिवर्तन मंत्रालय
  2. पंचायती राज मंत्रालय
  3. ग्रामीण विकास मंत्रालय
  4. जनजातीय कार्य मंत्रालय

उत्तर (d). अनुसूचित जनजाति एवं अन्य पारंपरिक वन निवासी (वन अधिकारों की मान्यता) अधिनियम, 2006 - वन अधिकार अधिनियम (एफआरए), 2006 - वन में रहने वाले आदिवासी समुदायों एवं अन्य पारंपरिक वनवासियों के वन संसाधनों के अधिकारों को मान्यता देता है, जिस पर ये समुदाय आजीविका, आवास एवं अन्य सामाजिक-सांस्कृतिक जरूरतों सहित विभिन्न जरूरतों के लिए निर्भर थे।

अधिनियम के अध्याय टप् में यह स्पष्ट रूप से उल्लेख किया गया है कि ‘नोडल एजेंसी - जनजातीय मामलों से संबंधित केंद्र सरकार का मंत्रालय या इस संबंध में केंद्र सरकार द्वारा अधिकृत कोई अधिकारी या प्राधिकरण, इस अधिनियम के प्रावधान के कार्यान्वयन के लिए नोडल एजेंसी होगी।”


85.कानून को लागू करने के मामले में कोई विधान, जो किसी कार्यपालक अथवा प्रशासनिक प्राधिकारी को अनिर्देशित एवं अनियंत्रित विवेकाधिकार देता है, भारत ' के संविधान के निम्नलिखित अनुच्छेदों में से किसका उल्लंघन करता है?

  1. अनुच्छेद 14
  2. अनुच्छेद 28
  3. अनुच्छेद 32
  4. अनुच्छेद 44

उत्तर (a). ‘प्रस्तावना’ के अलावा भारतीय संविधान का ‘अनुच्छेद 14’ कानून के समक्ष समानता एवं भारत के नागरिक को कानून की समान सुरक्षा सुनिश्चित करता है।

‘अनुच्छेद 14’ जो ‘अनियंत्रित एवं अनिर्देशित अधिकार’ के मूल में है एवं अनुच्छेद 21 जीवन एवं स्वतंत्रता के अधिकार की गारंटी देता है जो स्वतंत्रता की रक्षा एवं गरिमा के साथ जीवन सुनिश्चित करने का मौलिक प्रावधान है। यह भेदभाव को रोकता है एवं भेदभावपूर्ण कानूनों एवं प्रशासनिक कार्रवाई दोनों को प्रतिबंधित करता है।

अनुच्छेद 22 प्राकृतिक न्याय की गारंटी देता है एवं गिरफ्तार व्यक्ति को निष्पक्ष सुनवाई का प्रावधान करता है।

अनुच्छेद 28 कुछ शैक्षणिक संस्थानों में धार्मिक शिक्षा या धार्मिक पूजा में उपस्थिति के बारे में स्वतंत्रता से संबंधित है।

अनुच्छेद 32 ‘संवैधानिक उपचार के अधिकार’ से संबंधित है, या संविधान के भाग III में प्रदत्त अधिकारों के प्रवर्तन के लिए उपयुक्त कार्यवाही द्वारा सर्वोच्च न्यायालय को स्थानांतरित करने के अधिकार की पुष्टि करता है।

भारतीय संविधान का अनुच्छेद 44 समान नागरिक संहिता से संबंधित है।


86.भारतीय राज्य-व्यवस्था में, निम्नलिखित में से कौन-सी अनिवार्य विशेषता है, जो यह दर्शाती है कि उसका स्वरूप संघीय है?

  1. न्यायपालिका की स्वतंत्रता सुरक्षित है।
  2. संघ की विधायिका में संघटक इकाइयों के निर्वाचित प्रतिनिधि होते हैं।
  3. केन्द्रीय मंत्रिमंडल में क्षेत्रीय पार्टियों के निर्वाचित प्रतिनिधि हो सकते हैं।
  4. मूल अधिकार न्यायालयों द्वारा प्रवर्तनीय हैं।

उत्तर (b). संविधान या तो एकात्मक या संघीय होते हैं। एकात्मक सरकार में, सरकार की शक्तियाँ केंद्र सरकार में केंद्रीकृत होती हैं एवं राज्य केंद्र के अधीन होते हैं।

एक संघीय संविधान में, राज्यों एवं केंद्र सरकार के बीच सत्ता का विभाजन होता है एवं दोनों अपने-अपने क्षेत्रों में स्वतंत्र होते हैं।

भारतीय संविधान न तो विशुद्ध रूप से संघीय है एवं न ही पूर्णतः एकात्म। यह दोनों का सामंजस्यपूर्ण मिश्रण है।

संघवाद की प्रमुख विशेषताओं को इस प्रकार सूचीबद्ध किया जा सकता है -

  1. सरकार के दो या दो से अधिक स्तर होते हैं।
  2. सरकार के विभिन्न स्तर समान नागरिकों पर शासन करते हैं, लेकिन कानून, कराधान एवं प्रशासन के विशिष्ट मामलों में प्रत्येक स्तर का अपना अधिकार क्षेत्र होता है।
  3. सरकार के संबंधित स्तरों के क्षेत्राधिकार संविधान में निर्दिष्ट हैं। इसलिए सरकार के प्रत्येक स्तर के अस्तित्व एवं अधिकार की संवैधानिक गारंटी है।
  4. संविधान के मौलिक प्रावधानों को एक स्तर की सरकार द्वारा एकतरफा नहीं बदला जा सकता है। इस तरह के परिवर्तनों के लिए सरकार के दोनों स्तरों की सहमति की आवश्यकता होती है।
  5. न्यायालयों के पास संविधान एवं सरकार के विभिन्न स्तरों की शक्तियों की व्याख्या करने की शक्ति है। सर्वोच्च न्यायालय अंपायर के रूप में कार्य करता है यदि सरकार के विभिन्न स्तरों के बीच उनकी संबंधित शक्तियों के प्रयोग में विवाद उत्पन्न होता है।
  6. भारतीय संविधान द्वारा अपनाई गई संघीय व्यवस्था की सबसे महत्वपूर्ण विशेषता यह सिद्धांत है कि राज्यों एवं केंद्र के बीच संबंध सहयोग पर आधारित होंगे। इसके लिए स्वतंत्र न्यायपालिका का होना जरूरी है।
  7. इसलिए, विकल्प (a) सही है।


87.निम्नलिखित में से कौन-सा ‘राज्य' शब्द को सर्वोत्तम रूप से परिभाषित करता है?

  1. व्यक्तियों का एक समुदाय, जो बिना किसी बाह्य नियंत्रण के एक निश्चित भूभाग में स्थायी रूप से निवास करता है और जिसकी एक संगठित सरकार है
  2. एक निश्चित भूभाग के राजनैतिक रूप से संगठित लोग, जो स्वयं पर शासन करने, कानून एवं व्यवस्था को बनाए रखने, अपने नैसर्गिक अधिकारों की रक्षा करने तथा अपनी जीविका के साधनों को सुरक्षित रखने का अधिकार रखते हैं
  3. बहुत से व्यक्ति, जो एक निश्चित भूभाग में बहुत लंबे समय से अपनी संस्कृति, परंपरा और शासन-व्यवस्था के साथ रहते आए हैं।
  4. एक निश्चित भूभाग में स्थायी रूप से रह रहा समाज, जिसकी एक केन्द्रीय प्राधिकारी तथा केन्द्रीय प्राधिकारी के प्रति उत्तरदायी कार्यपालिका और एक स्वतंत्र न्यायपालिका है

उत्तर (a). राजनीति विज्ञान के अध्ययन के लिए ‘राज्य’ शब्द केंद्रीय है। लेकिन इसे गलत तरीके से राष्ट्र, समाज, सरकार आदि के पर्याय के रूप में इस्तेमाल किया जाता है।

राज्य की अवधारणा की कुछ परिभाषाएँ इस प्रकार हैं -

‘राज्य एक निश्चित क्षेत्र के राजनीतिक रूप से संगठित लोग हैं’ - ब्लंटश्ली

राज्य ‘व्यक्तियों, कमोबेश असंख्य, का एक समुदाय है, जिसका स्थायी रूप से किसी क्षेत्र के एक निश्चित हिस्से पर आधिपत्य रहा होता है, स्वतंत्र, या लगभग इतना ही, बाहरी नियंत्रण का, एवं एक संगठित सरकार रखता है जिसके लिए निवासियों का बड़ा समूह आदतन आज्ञाकारिता रखता है।’ - गार्नेर

राज्य ‘एक क्षेत्रीय समाज है जो सरकारों एवं विषयों में विभाजित है, चाहे वे व्यक्ति हों या व्यक्तियों के संघ, जिनके संबंध इस सर्वोच्च जबरदस्त शक्ति के प्रयोग से निर्धारित होते हैं।’ - लास्की

राज्य ‘एक निश्चित क्षेत्र के भीतर कानून के लिए संगठित लोग हैं’। - वुडरो विल्सन

‘राज्य राजनीति विज्ञान की एक अवधारणा है, एवं एक नैतिक वास्तविकता है जो वहां मौजूद है जहां एक निश्चित क्षेत्र में रहने वाले कई लोग एक सरकार के तहत एकीकृत होते हैं जो आंतरिक मामलों में अपनी संप्रभुता व्यक्त करती है एवं बाहरी मामलों में अन्य सरकारों से स्वतंत्र है। - गिलक्रिस्ट

एक ‘राज्य’ ऐसा होता है कि उसके पास अकेले कानून बनाने का विशेषाधिकार होता है। कानून बनाने की शक्ति संप्रभुता से प्राप्त होती है, जो राज्य की सबसे विशिष्ट विशेषता है।

केवल विकल्प (a) परिभाषाओं के अनुकूल है।


88.भारतीय न्यायपालिका के संदर्भ में, निम्नलिखित कथन borge पर विचार कीजिए:

  1. भारत के राष्ट्रपति की पूर्वानुमति से भारत के मुख्य न्यायमूर्ति द्वारा उच्चतम न्यायालय से सेवानिवृत्त किसी न्यायाधीश को उच्चतम न्यायालय वे न्यायाधीश के पद पर बैठने और कार्य करने हेतु बुलाया जा सकता है।
  2. भारत में किसी भी उच्च न्यायालय को अपने निर्णय के पुनर्विलोकन की शक्ति प्राप्त है, जैस कि उच्चतम न्यायालय के पास है।

उपर्युक्त कथनों में से कौन-सा/कौन-से सही है/हैं?

  1. केवल 1
  2. केवल 2
  3. 1 और 2 दोनों
  4. न तो 1 और न ही 2

उत्तर (c). कथन 1 सही है - संविधान के अनुच्छेद 128 के तहत, भारत के मुख्य न्यायाधीश, किसी भी समय, राष्ट्रपति की पूर्व सहमति से, किसी भी व्यक्ति से, जिसने सर्वोच्च न्यायालय के न्यायाधीश का पद धारण किया था, पुनः सर्वोच्च न्यायालय के न्यायाधीश के रूप में नियूक्त करने एवं कार्य करने का अनुरोध कर सकता है।

कथन 2 सही है - ‘एक न्यायालय’ होने के नाते, एक उच्च न्यायालय संविधान के अनुच्छेद 226 के तहत अपने स्वयं के निर्णयों की समीक्षा कर सकता है। केरल उच्च न्यायालय ने कानूनी प्रस्ताव को दोहराया है कि उच्च न्यायालय अभिलेख न्यायालय के रूप में अपने स्वयं के आदेशों की समीक्षा कर सकते हैं। मुख्य न्यायाधीश एस मणिकुमार एवं शाजी पी चाली की खंडपीठ को एक समीक्षा याचिका के खिलाफ अपील के साथ सामना करना पड़ा।


89.भारत के संदर्भ में, निम्नलिखित कथनों पर विचार कीजिए:

  1. भारत में केवल एक ही नागरिकता और एक ही अधिवास है।
  2. जो व्यक्ति जन्म से नागरिक हो, केवल वही राष्ट्राध्यक्ष बन सकता है।
  3. जिस विदेशी को एक बार नागरिकता दे दी गई है, किसी भी परिस्थिति में उसे इससे वंचित नहीं किया जा सकता।

उपर्युक्त कथनों में से कौन-सा/कौन-से सही है/हैं?

  1. केवल 1
  2. केवल 2
  3. 1 और 3
  4. 2 और 3

उत्तर (a). भारत के संविधान का भाग प्प् (अनुच्छेद 5-11) भारत की नागरिकता से संबंधित है।

अनुच्छेद 5 - संविधान के प्रारंभ में नागरिकता

इस संविधान के प्रारंभ में, प्रत्येक व्यक्ति जिसका भारत के राज्यक्षेत्र में अधिवास है एवं -

  1. जो भारत के क्षेत्र में पैदा हुआ था या
  2. जिनके माता-पिता में से कोई भी भारत के क्षेत्र में पैदा हुआ था या
  3. जो भारत के राज्य क्षेत्र में सामान्य रूप से इस तरह के प्रारंभ से कम से कम पांच साल पहले से निवास कर रहा है, वह भारत का नागरिक होगा।

अनुच्छेद 9 - किसी विदेशी राज्य की नागरिकता स्वेच्छा से प्राप्त करने वाले व्यक्तियों का नागरिक नहीं होना

कोई भी व्यक्ति अनुच्छेद 5 के आधार पर भारत का नागरिक नहीं होगा या अनुच्छेद 6 या अनुच्छेद 8 के आधार पर भारत का नागरिक नहीं माना जाएगा यदि उसने स्वेच्छा से किसी विदेशी राज्य की नागरिकता प्राप्त की है। अतः कथन 1 सही है।

अनुच्छेद 10 - नागरिकता के अधिकारों की निरंतरता

प्रत्येक व्यक्ति जो इस भाग के पूर्वगामी प्रावधानों में से किसी के तहत भारत का नागरिक है या माना जाता है, संसद द्वारा बनाए गए किसी भी कानून के प्रावधानों के अधीन, ऐसा नागरिक बना रहेगा।

नागरिकता से वंचित (धारा 10 या भारतीय नागरिकता अधिनियम, 1955)

वंचन भारत की नागरिकता की अनिवार्य समाप्ति है। भारत के एक नागरिक को देशीयकरण, पंजीकरण, अधिवास एवं निवास द्वारा केंद्र सरकार के एक आदेश द्वारा उसकी नागरिकता से वंचित किया जा सकता है यदि वह संतुष्ट है कि -

नागरिक ने धोखाधड़ी, झूठे प्रतिनिधित्व या किसी भौतिक तथ्य को छिपाने के माध्यम से नागरिकता प्राप्त की है,

नागरिक भारत के संविधान के प्रति निष्ठाहीन रहा है,

नागरिक ने युद्ध के दौरान दुश्मन के साथ अवैध रूप से व्यापार या संचार किया है,

नागरिक, पंजीकरण या निष्प्रभावीकरण के बाद पांच वर्ष के भीतर, किसी भी देश में दो साल के लिए कैद हो गया है,

नागरिक सामान्य रूप से लगातार सात वर्षों से भारत से बाहर का निवासी है।

अतः कथन 3 सही नहीं है।

संविधान का अनुच्छेद 58 राष्ट्रपति के पद के लिए पात्र होने के लिए प्रमुख योग्यताएं निर्धारित करता है। एक राष्ट्रपति होना चाहिएरू भारत का नागरिक, 35 वर्ष या उससे अधिक आयु का होना चाहिए।

राष्ट्रपति के रूप में विदेशी मूल के भारतीय नागरिक की व्याख्या करने वाला कोई लिखित प्रावधान नहीं है।

अतः कथन 2 सही नहीं है।


90.निम्नलिखित में से कौन-सा कारक किसी उदार लोकतंत्र में स्वतंत्रता की सर्वोत्तम सुरक्षा को नियत करता है?

  1. एक प्रतिबद्ध न्यायपालिका
  2. शक्तियों का केन्द्रीकरण
  3. निर्वाचित सरकार
  4. शक्तियों का पृथक्करण

उत्तर (d). उदार लोकतंत्र के दृष्टिकोण में निहित है, सत्ता के सभी रूप लोगों की इच्छा में निहित हैं।

यह कानून के शासन को लोकतंत्र की बुनियादी नींव में से एक के रूप में बढ़ाता है, यह उनके संयम के लिए एक वाहन के रूप में शक्तियों के पृथक्करण की पुष्टि करता है, एवं यह व्यक्तियों के अधिकारों एवं स्वतंत्रता को उनकी गरिमा के लिए एक शर्त के रूप में बढ़ावा देता है।

एक ‘प्रतिबद्ध न्यायपालिका’ एक अपमानजनक शब्द है जिसका अर्थ है तत्कालीन प्रचलित सरकार के लिए प्रतिबद्ध। इसलिए (a) गलत है। विकल्प (b) स्पष्ट रूप से गलत है।


91.भारतीय संविधान के अंतर्गत धन का केन्द्रीकरण किसका उल्लंघन करता है?

  1. समता का अधिकार
  2. राज्य की नीति के निदेशक तत्त्व
  3. स्वातंत्र्य का अधिकार
  4. कल्याण की अवधारणा ।

उत्तर (b). भारतीय संविधान के राज्य नीति के निर्देशक सिद्धांतों में, अनुच्छेद 38 कहता है कि ‘राज्य सामाजिक, आर्थिक एवं राजनीतिक न्याय एवं आय, स्थिति, सुविधाओं एवं अवसर में असमानताओं को कम करना सुनिश्चित करके एक सामाजिक व्यवस्था को सुरक्षित एवं संरक्षित करके लोगों के कल्याण को बढ़ावा देने का प्रयास करें’।

अनुच्छेद 39 - राज्य विशेष रूप से अपनी नीतियों को सुरक्षित करने की दिशा में निर्देशित करेगा -

सभी नागरिकों को आजीविका के पर्याप्त साधन का अधिकार।

भौतिक संसाधनों के स्वामित्व एवं नियंत्रण को सामान्य भलाई की सेवा के लिए व्यवस्थित किया जाएगा।

राज्य कुछ ही हाथों में धन की एकाग्रता से बचना चाहिए।

पुरुषों एवं महिलाओं दोनों के लिए समान काम के लिए समान वेतन।

श्रमिकों की शक्ति एवं स्वास्थ्य की सुरक्षा।

बालशोषण नहीं किया जाएगा।


92.भारत में संपत्ति के अधिकार की क्या स्थिति है?

  1. यह विधिक अधिकार है, जो केवल नागरिकों को प्राप्त है
  2. यह विधिक अधिकार है, जो किसी भी व्यक्ति को प्राप्त है
  3. यह मूल अधिकार है, जो केवल नागरिकों को प्राप्त है
  4. यह न तो मूल अधिकार है, न ही विधिक अधिकार

उत्तर (b). संविधान में ‘संपत्ति के अधिकार’ का प्रावधान मूल रूप से अनुच्छेद 19 एवं 31 (मौलिक अधिकार) के तहत किया गया था। अनुच्छेद 19 सभी नागरिकों को संपत्ति अर्जित करने, धारण करने एवं बेचने का अधिकार प्रदान करता है। अनुच्छेद 31 के अनुसार ‘कानून के अधिकार के बिना किसी भी व्यक्ति को उसकी संपत्ति से वंचित नहीं किया जाएगा।’ यह भी प्रदान किया गया है कि जिस व्यक्ति की संपत्ति सार्वजनिक उद्देश्यों के लिए ली गई है उसे मुआवजा दिया जाए।

संपत्ति के अधिकार से संबंधित प्रावधानों को कई बार बदला गया है। 1978 के 44वें संशोधन ने संपत्ति के अधिकार को मौलिक अधिकारों की सूची से हटा दिया। संविधान में एक नया प्रावधान, अनुच्छेद 300-ए जोड़ा गया, जिसमें यह प्रावधान था कि ‘कानून के अधिकार के बिना किसी भी व्यक्ति को उसकी संपत्ति से वंचित नहीं किया जाएगा’।

विदेशियों एवं भारतीय नागरिकों के संबंध में - भारत के बाहर का निवासी गैर-भारतीय मूल का कोई विदेशी नागरिक भारत में कोई अचल संपत्ति नहीं खरीद सकता है। विदेशी नागरिकों के लिए भारत में संपत्ति का स्वामित्व तब तक अवैध है जब तक कि वे एक वित्तीय वर्ष में 183 दिनों की निवास आवश्यकता को पूरा नहीं करते (एक पर्यटक वीजा 180 दिनों तक रहता है)। पर्यटक वीजा पर संपत्ति खरीदना भी अवैध है।

संपत्ति एक अपात्र व्यक्ति के साथ एक पात्र व्यक्ति के नाम पर संयुक्त रूप से नहीं खरीदी जा सकती है। इसका अर्थ है कि एक अनिवासी भारतीय (एनआरआई) या भारतीय मूल का विदेशी नागरिक (पीआईओ) किसी विदेशी के साथ संयुक्त रूप से संपत्ति नहीं खरीद सकता है। हालांकि, भारत में एक विदेशी नागरिक को भारत में कोई अचल संपत्ति खरीदने के लिए आरबीआई के अनुमोदन की आवश्यकता नहीं है। ऐसा इसलिए है क्योंकि एक बार जब वह भारत का निवासी हो जाता है, तो उसे किसी अन्य निवासी की तरह अधिकार मिल जाते हैं। हालाँकि यह स्वतंत्रता पाकिस्तान, बांग्लादेश, श्रीलंका, अफगानिस्तान, चीन, ईरान, नेपाल एवं भूटान के नागरिकों के लिए उपलब्ध नहीं है।


93.26 जनवरी, 1950 को भारत की वास्तविक सांविधानिक स्थिति क्या थी?

  1. लोकतंत्रात्मक गणराज्य
  2. संपूर्ण प्रभुत्व-संपन्न लोकतंत्रात्मक गणराज्य
  3. संपूर्ण प्रभुत्व-संपन्न पंथनिरपेक्ष लोकतंत्रात्मक गणराज्य
  4. संपूर्ण प्रभुत्व-संपन्न समाजवादी पंथनिरपेक्ष लोकतंत्रात्मक गणराज्य

उत्तर (b). जनवरी 26, 1950 को इसके प्रारंभ होने के समय संविधान के अनुसार, भारत एक संप्रभु, लोकतांत्रिक, गणराज्य था। इसे 42वें संशोधन अधिनियम 1976 के बाद समाजवादी एवं धर्मनिरपेक्ष होने का संवैधानिक दर्जा मिला।


94.सांविधानिक सरकार का आशय क्या है?

  1. किसी राष्ट्र की परिसंघीय संरचना वाली एक प्रतिनिधि सरकार
  2. कोई सरकार, जिसके प्रमुख के पास नाममात्र की शक्तियाँ हों
  3. कोई सरकार, जिसके प्रमुख के पास वास्तवि शक्तियाँ हों
  4. कोई सरकार, जो संविधान की सीमाओं से परिबद्ध हो

उत्तर (d). संविधानिक सरकार को एक संविधान के अस्तित्व से परिभाषित किया जाता है - जो एक कानूनी साधन हो सकता है या केवल निश्चित मानदंडों या सिद्धांतों का एक सेट जिसे आम तौर पर मौलिक कानून के रूप में स्वीकार किया जाता है।

राजव्यवस्था - जो राजनीतिक शक्ति के प्रयोग को प्रभावी ढंग से नियंत्रित करती है।

प्रतिनिधि लोकतंत्र, जिसे अप्रत्यक्ष लोकतंत्र के रूप में भी जाना जाता है, एक प्रकार का लोकतंत्र है जिसकी स्थापना प्रत्यक्ष लोकतंत्र के विपरीत, लोगों के समूह का प्रतिनिधित्व करने वाले निर्वाचित व्यक्तियों का सिद्धांत होता है।

एक संघीय गणराज्य सरकार के गणतांत्रिक रूप वाले राज्यों का एक संघ है।

इसलिए केवल विकल्प (d) सत्य है।


95.भारत के संदर्भ में 'हल्बी, हो और कुई' पद किससे संबंधित हैं?

  1. पश्चिमोत्तर भारत का नृत्यरूप
  2. वाद्ययंत्र
  3. प्रागैतिहासिक गुफा चित्रकला
  4. जनजातीय भाषा

उत्तर (d). हल्बी, हो एवं कुई भारत की जनजातीय भाषाएँ हैं।

हल्बी एक पूर्वी इंडो-आर्यन भाषा है, जो उड़िया एवं मराठी के बीच संक्रमणकालीन है। यह भारत के मध्य भाग में 5,00,000 लोगों द्वारा बोली जाती है।

हो ऑस्ट्रोएशियाटिक भाषा परिवार की एक मुंडा भाषा है जो 2001 की जनगणना के अनुसार, मुख्य रूप से भारत में लगभग 1.04 मिलीयन लोगों (भारत की जनसंख्या का 0.103%) लोगो द्वारा बोली जाती है। यह भाषा ओडिशा, झारखंड, बिहार, छत्तीसगढ़, पश्चिम बंगाल, असम के हो, मुंडा कोल्हा एवं कोल आदिवासी समुदाय द्वारा बोली जाती है एवं वारंग सिटी लिपि में लिखी जाती है।

कुई एक दक्षिण-पूर्वी द्रविड़ भाषा है जो कांधों द्वारा बोली जाती है। यह ज्यादातर ओडिशा में बोली जाती है, एवं उड़िया लिपि में लिखी जाती है।


96.भारतरत्न और पद्म पुरस्कारों के संबंध में, निम्नलिखित कथनों पर विचार कीजिए:

  1. भारतरत्न और पद्म पुरस्कार, भारत के संविधान के अनुच्छेद 18 (1) के अंतर्गत उपाधियाँ हैं।
  2. वर्ष 1954 में प्रारंभ किए गए पद्म पुरस्कारों को केवल एक बार निलंबित किया गया था।
  3. किसी वर्ष-विशेष में भारतरत्न पुरस्कारों की अधिकतम संख्या पाँच तक सीमित है।

उपर्युक्त कथनों में से कौन-से सही नहीं हैं?

  1. केवल 1 और 2
  2. केवल 2 और 3
  3. केवल 1 और 3
  4. 1, 2 और 3

उत्तर (d). भारत रत्न -

  1. देश के सर्वोच्च नागरिक सम्मान ‘भारत रत्न’ की स्थापना वर्ष 1954 में की गई थी।
  2. जाति, व्यवसाय, पद या लिंग के भेद के बिना कोई भी व्यक्ति इन पुरस्कारों के लिए पात्र है।
  3. यह मानव प्रयास के किसी भी क्षेत्र में असाधारण सेवा/उच्चतम क्रम के प्रदर्शन की मान्यता में प्रदान किया जाता है।
  4. भारत रत्न की सिफारिशें स्वयं प्रधानमंत्री द्वारा राष्ट्रपति को की जाती हैं। इसके लिए किसी औपचारिक सिफारिश की जरूरत नहीं है।
  5. वार्षिक पुरस्कारों की संख्या किसी विशेष वर्ष में अधिकतम तीन तक सीमित है।
  6. पुरस्कार प्रदान करने पर, प्राप्तकर्ता को राष्ट्रपति द्वारा हस्ताक्षरित एक सनद (प्रमाण पत्र) एवं एक पदक प्राप्त होता है।
  7. पुरस्कार में कोई मौद्रिक अनुदान नहीं होता है।

पद्म पुरस्कार -

  1. पद्म पुरस्कार गणतंत्र दिवस की पूर्व संध्या पर प्रतिवर्ष घोषित भारत के सर्वोच्च नागरिक सम्मानों में से एक है।
  2. पुरस्कार तीन श्रेणियों में दिए जाते हैं - पद्म विभूषण (असाधारण एवं विशिष्ट सेवा के लिए), पद्म भूषण (उच्च क्रम की विशिष्ट सेवा) एवं पद्म श्री (प्रतिष्ठित सेवा)।
  3. पुरस्कार गतिविधियों या विषयों सार्वजनिक सेवा का एक तत्व वाले सभी क्षेत्रों में उपलब्धियों को मान्यता दी जाती है।
  4. पद्म पुरस्कार - पद्म पुरस्कार समिति द्वारा की गई सिफारिशों पर प्रदान किए जाते हैं, जिसका गठन प्रतिवर्ष प्रधानमंत्री द्वारा किया जाता है। नामांकन प्रक्रिया जनता के लिए खुली है। यहां तक कि स्व-नामांकन भी किया जा सकता है।
  5. पद्म पुरस्कार, जो वर्ष 1954 में स्थापित किए गए थे, प्रतिवर्ष गणतंत्र दिवस के अवसर पर 1978 एवं 1979 तथा 1993 से 1997 के दौरान संक्षिप्त रुकावटों को छोड़कर घोषित किए जाते हैं।

भारतीय संविधान का अनुच्छेद 18(1) सभी उपाधियों को समाप्त कर देता है। यह राज्य को किसी भी नागरिक या गैर-नागरिक को उपाधि प्रदान करने से रोकता है। हालाँकि, सैन्य एवं शैक्षणिक भेदों को निषेध से छूट दी गई है। इस प्रकार, एक विश्वविद्यालय योग्यता के आधार पर उपाधि या सम्मान दे सकता है।

अतः कथन 1, 2 एवं 3 सही नहीं हैं।


97.निम्नलिखित कथनों पर विचार कीजिए:

कथन 1: संयुक्त राष्ट्र पूँजी विकास निधि (यू० एन० सी० डी० एफ०) और आर्बर डे फाउंडेशन ने हाल ही में हैदराबाद को विश्व के 2020 वृक्ष नगर की मान्यता प्रदान की है।

कथन 2: शहरी वनों को बढ़ाने और संपोषित करने के प्रति प्रतिबद्धता को देखते हुए हैदराबाद का एक वर्ष के लिए इस मान्यता हेतु चयन किया गया है।

उपर्युक्त कथनों के संदर्भ में, निम्नलिखित में से कौन-सा सही है?

  1. कथन 1 और कथन 2 दोनों सही हैं और कथन 2, कथन 1 की सही व्याख्या है
  2. कथन 1 और कथन 2 दोनों सही हैं, किन्तु कथन 2, कथन 1 की सही व्याख्या नहीं है
  3. कथन 1 सही है, किन्तु कथन 2 सही नहीं है
  4. कथन 1 सही नहीं है, किन्तु कथन 2 सही है

उत्तर (d). आर्बर डे फाउंडेशन एवं संयुक्त राष्ट्र के खाद्य एवं कृषि संगठन (यूएनसीडीएफ नहीं जैसा कि प्रश्न में दिया गया है) ने हैदराबाद को ‘विश्व के 2020 वृक्ष नगर’ के रूप में मान्यता दी है। अतः कथन 1 सही नहीं है।

हरित हरम कार्यक्रम एवं शहरी वन पार्क योजना जैसी पहलों के कारण हैदराबाद को ट्री सिटी के रूप में चुना गया था। हरित हरम, हरित आवरण को बढ़ाने के लिए राज्य भर में बड़े पैमाने पर वृक्षारोपण के लिए राज्य सरकार का एक प्रमुख कार्यक्रम है। अतः कथन 2 सही है।


98.वर्ष 2000 में प्रारंभ किए गए लॉरियस विश्व खेल पुरस्कार (लॉरियस वर्ल्ड स्पोर्ट्स अवार्ड) के संबंध में निम्नलिखित कथनों पर विचार कीजिए:

  1. अमरीकी गोल्फ खिलाड़ी टाइगर वुड्स इस पुरस्कार का सर्वप्रथम विजेता थे।
  2. अब तक यह पुरस्कार अधिकतर ‘फॉर्मूला वन' के खिलाड़ियों को मिला है।
  3. अन्य खिलाड़ियों की तुलना में रॉजर फेडरर को यह पुरस्कार सर्वाधिक बार मिला है।

उपर्युक्त कथनों में से कौन-से सही हैं?

  1. केवल 1 और 2
  2. केवल 2 और 3
  3. केवल 1 और 3
  4. 1, 2 और 3

उत्तर (c). लॉरियस वल्र्ड स्पोर्ट्स अवार्ड्स एक वार्षिक पुरस्कार समारोह है जिसमें खेल की दुनिया के व्यक्तियों एवं टीमों को पूरे वर्ष खेल उपलब्धियों के साथ सम्मानित किया जाता है। यह 1999 में लॉरियस स्पोर्ट फॉर गुड फाउंडेशन के संस्थापक संरक्षक डेमलर एवं रिचमोंट द्वारा स्थापित किया गया था।

पहला समारोह 25 मई 2000 को मोंटे कार्लो में आयोजित किया गया था जिसमें अमेरिकी गोल्फर टाइगर वुड्स पुरस्कार के (प्रथम) विजेता थे।

स्विस टेनिस खिलाड़ी रोजर फेडरर के नाम सबसे ज्यादा छह पुरस्कार, पांच स्पोर्ट्समैन ऑफ द ईयर एवं एक कमबैक ऑफ द ईयर का रिकॉर्ड है।

कुल 21 बार पुरस्कार से सम्मानित किया गया है, फिर भी ‘फॉर्मूला वन’ खिलाड़ियों ने इसे केवल चार बार जीता है, यानी 2020 में - लुईस हैमिल्टन, 2014- सेबेस्टियन वेटेल एवं 2002 एवं 2004 - माइकल शूमाकर।

अतः केवल कथन 1 एवं 3 सत्य हैं।


99.32वें ग्रीष्मकालीन ओलंपिक के संदर्भ में, निम्नलिखित कथनों पर विचार कीजिए:

  1. इस ओलंपिक का आधिकारिक आदर्श वाक्य एक नई दुनिया (ए न्यू वर्ल्ड)' है।
  2. इस ओलंपिक में स्पोर्ट क्लाइंबिंग, सर्फिंग स्केटबोर्डिंग, कराटे तथा बेसबॉल को शामि किया गया है।

उपर्युक्त कथनों में से कौन-सा/कौन-से सही है/हैं?

  1. केवल 1
  2. केवल 2
  3. 1 और 2 दोनों
  4. न तो 1 और न ही 2

उत्तर (b). 2021 में टोक्यो (जापान) में आयोजित 32वें ग्रीष्मकालीन ओलंपिक (2020 ग्रीष्मकालीन ओलंपिक) के लिए आधिकारिक आदर्श वाक्य (कोविड-19 बीमारी से विलंबित) ‘यूनाइटेड बाय विज़न’ था।

ओलंपिक खेलों के इतिहास में किसी न किसी समय कुल 42 खेलों में 55 प्रतियोगिताओं में खेले जा चुके हैं। 32वें ग्रीष्मकालीन ओलंपिक के लिए कार्यक्रम में 33 खेल जो 50 प्रतियोगिताओं में फैले थे, शामिल किए गए।

2016 में रियो डी जनेरियो (ब्राजील) में आयोजित ग्रीष्मकालीन ओलंपिक में, 28 खेल (41 प्रतियोगिताएँ) थे। जापानी ओलंपिक समिति द्वारा 2020 के ग्रीष्मकालीन ओलंपिक में बेसबॉल एवं सॉफ्टबॉल, कराटे, स्पोर्टस क्लाईबिंग, सर्फिंग एवं स्केटबोर्डिंग शामिल किया गया था, जिनमें से ओलंपिक में पहली बार खेले गए थे, एवं जिनमें से अंतिम तीन ओलंपिक कार्यक्रम पर बने रहेंगे।

अतः केवल कथन 2 सत्य है।


100.आइ० सी० सी० वर्ल्ड टेस्ट चैंपियनशिप के संदर्भ निम्नलिखित कथनों पर विचार कीजिए:

  1. अंतिम दौर में पहुंचने वाली टीमों का निर्धा रण उनके द्वारा जीते गए मैचों की संख्या के आधार पर किया गया।
  2. न्यूजीलैंड का स्थान इंग्लैंड से ऊपर था, क्यों कि उसने इंग्लैंड की तुलना में अधिक मैच जीते।

उपर्युक्त कथनों में से कौन-सा/कौन-से सही है/हैं?

  1. केवल 1
  2. केवल 2
  3. 1 और 2 दोनों
  4. न तो 1 और न ही 2

उत्तर (d). icc-cricket.com पर उपलब्ध जानकारी के अनुसार, न्यूजीलैंड 2,764 अंकों के साथ इंग्लैंड (अंक 3753) से आगे है।

आईसीसी विश्व टेस्ट चैम्पियनशिप अंक प्रणाली में 2021-23 चक्र में लड़े गए प्रत्येक टेस्ट मैच के लिए, विजेता टीम को बारह अंक दिए जाएंगे, ड्रॉ के लिए चार अंक एवं एक टाई गेम के लिए छह अंक दिए जाएंगे। डब्ल्युटीसी स्टैंडिंग निर्धारित करने के लिए जीते गए अंकों के प्रतिशत उपयोग किया जाता है।

अतः 1 एवं 2 दोनों सही नहीं हैं।



Exam Analysis HOME       2021 - Paper I - English       Paper II - E       Paper I - Hindi       Paper II - H      


Questions and Detailed Solutions are being continuously updated ... refresh and check. Comment and let us know your experience, answers and solutions too!